Slot System
Featured Buckets
Featured Buckets Admin

An alternative to warfarin for patients with PE

Article Type
Changed
Thu, 07/16/2020 - 14:06
Display Headline
An alternative to warfarin for patients with PE
PRACTICE CHANGER

Consider treating patients with acute pulmonary embolism (PE) with rivaroxaban, a factor Xa inhibitor; it works as well as low-molecular-weight heparin (LMWH) followed by warfarin, but may cause fewer major bleeds.1

STRENGTH OF RECOMMENDATION

B: Based on a single, nonblinded randomized controlled trial.

EINSTEIN-PE Investigators; Buller HR, Prins MH, Lensin AW, et al. Oral rivaroxaban for the treatment of symptomatic pulmonary embolism. N Engl J Med. 2012;366:1287-1297.

 

ILLUSTRATIVE CASE

A 55-year-old man is brought to the emergency department with shortness of breath, pleuritic chest pain, and hypoxia shortly after returning from an overseas business trip. High-resolution spiral computed tomography (CT) reveals a PE.

How should he be treated?

Pulmonary embolism (PE) is fairly common—with an annual incidence estimated at 69 per 100,0002—and the cause of significant morbidity and mortality. Up to 30% of patients with venous thromboembolism (VTE) die within a month of diagnosis, mostly from PE, and in about 25% of cases, PE presents as sudden death.3

Warfarin has a significant downside
Standard therapy consists of either unfractionated heparin or LMWH followed by warfarin, a vitamin K antagonist (VKA), for ≥3 months.4 In addition to requiring frequent laboratory monitoring, warfarin has potentially significant interactions with many prescription drugs. Numerous trials have investigated novel anticoagulants for treatment of VTE in recent years. In one randomized controlled trial (RCT), rivaroxaban (Xarelto)was found to be noninferior to a VKA for treating acute deep vein thrombosis (DVT).5

STUDY SUMMARY: Major bleeding is less likely with rivaroxaban

The EINSTEIN PE investigators conducted a randomized, unblinded noninferiority trial to determine whether rivaroxaban was at least as effective as the standard therapy—enoxaparin, followed by a dose-adjusted VKA (warfarin [for US patients] or acenocoumarol) for acute symptomatic PE.1 To be included, participants had to have PE confirmed by CT, ventilation perfusion scan, or pulmonary angiography, with or without accompanying DVT. Exclusion criteria included active bleeding, significant renal impairment (creatinine clearance <30 mL/min), >48 hours of heparin treatment, or more than one dose of a VKA.

Participants (N=4832 in 38 countries) were randomized to receive either rivaroxaban (15 mg twice daily for 3 weeks, then 20 mg once a day thereafter) or standard therapy. The intervention and control groups were similar. Just over half were male, with an average age of 58 years; three-quarters of the patients had an intermediate to extensive PE burden; and 90% were hospitalized for initial treatment. The researchers listed the etiology as unprovoked in 64% of the cases, followed by recent surgery or trauma and immobilization (17% and 16%, respectively).

After VKA initiation, the international normalized ratio (INR) was checked at least monthly. Patients in the control groups were within the target range (INR 1-2) 62% of the time, which is similar to other studies of anticoagulation in patients with VTE. Adherence to rivaroxaban was at least 80% in more than 94% of patients. Treatment lasted 3, 6, or 12 months, with the duration determined before randomization by the treating physician.

There was no difference in dropout rates (10.7% of rivaroxaban patients withdrew for any reason, vs 12.3% of the controls). Fewer than 0.5% of participants were lost to follow-up.

Symptomatic recurrent VTE, the primary outcome, occurred in 50 patients receiving rivaroxaban vs 44 of those on standard therapy (2.1% vs 1.8%; P=.003 for noninferiority using an intention-to-treat analysis). Major bleeding, defined as overt bleeding causing death, a drop in hemoglobin of ≥2 points, needing a transfusion, or bleeding in a critical site, occurred less often in the rivaroxaban group (1.1% vs 2.2%, P=.003, NNT=91). There was no significant difference in overall bleeding rates between the 2 groups.1

WHAT’S NEW: Rivaroxaban is easier to use—and on label

This trial found rivaroxaban to be at least as effective as enoxaparin followed by a dose-adjusted VKA for acute symptomatic PE, with fewer major bleeding events. What’s more, rivaroxaban—which now has US Food and Drug Administration approval for the prevention and treatment of PE and DVT6—does not require laboratory monitoring.

 

 

 

CAVEATS: Questions about study population, duration remain

This was an open-label study—neither patients nor investigators were blinded to the group assignments after randomization. The investigators suspected more recurrent VTE in those receiving rivaroxaban, which could have biased their findings in favor of the standard treatment. However, actual rates of recurrence were similar.

Study participants were <60 years old, on average, which may limit extrapolation to an older population. This trial lasted 12 months; the effects of longer treatment with rivaroxaban are unknown. Bayer HealthCare and Janssen Pharmaceuticals, who jointly manufacture rivaroxaban, funded the study.

CHALLENGES TO IMPLEMENTATION: Cost and lack of antidote may limit use

Rivaroxaban is more expensive than warfarin: A one-month supply costs approximately $260, while a month’s supply of warfarin plus lab monitoring runs less than $100.7 What’s more, factor Xa inhibitors, unlike VKAs, do not have a readily available pharmacologic antidote.

Acknowledgement

The PURLs Surveillance System is supported in part by Grant Number UL1RR024999 from the National Center for Research Resources, a Clinical Translational Science Award to the University of Chicago. The content is solely the responsibility of the authors and does not necessarily represent the official views of the National Center for Research Resources or the National Institutes of Health.

Files
References

1. Buller HR, Prins MH, Lensin AW, et al. EINSTEIN-PE Investigators; Oral rivaroxaban for the treatment of symptomatic pulmonary embolism. N Engl J Med. 2012;366:1287-1297.

2. Silverstein MD, Heit JA, Mohr DN, et al. Trends in the incidence of deep vein thrombosis and pulmonary embolism: a 25-year population-based study. Arch Intern Med. 1998;158:585-593.

3. Beckman MG, Hooper WC, Critchley SE, et al. Venous thromboembolism: a public health concern. Am J Prev Med. 2010;38(suppl):S495-S501.

4. Kearon C, Akl EA, Comerota AJ, et al. Antithrombotic therapy for VTE disease: antithrombotic therapy and prevention of thrombosis, 9th ed: American College of Chest Physicians Evidence-Based Clinical Practice Guidelines. Chest. 2012;141(suppl):e419S-e494S.

5. Bauersachs R, Berkowitz SD, Brenner B, et al. Oral rivaroxaban for symptomatic venous thromboembolism. N Engl J Med. 2010;363:2499-2510.

6. US Food and Drug Administration. FDA expands use of Xarelto to treat, reduce recurrence of blood clots [press release]. November 2, 2012. Available at: http://www.fda.gov/NewsEvents/Newsroom/PressAnnouncements/ucm326654.htm. Accessed November 12, 2012.

7. PL detail-document. Comparison of oral antithrombotics. Prescriber’s Letter. 2011;18:271020.-

Article PDF
Author and Disclosure Information

Laura Morris, MD, MSPH
Department of Family and Community Medicine, University of Missouri-Columbia

James J. Stevermer, MD, MSPH
Department of Family and Community Medicine, University of Missouri-Columbia

PURLs EDITOR
Anne Mounsey, MD
Department of Family, Medicine, University of North Carolina at, Chapel Hill

Issue
The Journal of Family Practice - 61(12)
Publications
Topics
Page Number
751-752
Legacy Keywords
Laura Morris;MD;MSPH; James J. Stevermer;MD;MSPH; alternative to warfarin; pulmonary embolism; PE; rivaroxaban; factor Xa inhibitor; low-molecular-weight heparin; vitamin K antagonist; VKA; laboratory monitoring; VTE; venous thromboembolism
Sections
Files
Files
Author and Disclosure Information

Laura Morris, MD, MSPH
Department of Family and Community Medicine, University of Missouri-Columbia

James J. Stevermer, MD, MSPH
Department of Family and Community Medicine, University of Missouri-Columbia

PURLs EDITOR
Anne Mounsey, MD
Department of Family, Medicine, University of North Carolina at, Chapel Hill

Author and Disclosure Information

Laura Morris, MD, MSPH
Department of Family and Community Medicine, University of Missouri-Columbia

James J. Stevermer, MD, MSPH
Department of Family and Community Medicine, University of Missouri-Columbia

PURLs EDITOR
Anne Mounsey, MD
Department of Family, Medicine, University of North Carolina at, Chapel Hill

Article PDF
Article PDF
PRACTICE CHANGER

Consider treating patients with acute pulmonary embolism (PE) with rivaroxaban, a factor Xa inhibitor; it works as well as low-molecular-weight heparin (LMWH) followed by warfarin, but may cause fewer major bleeds.1

STRENGTH OF RECOMMENDATION

B: Based on a single, nonblinded randomized controlled trial.

EINSTEIN-PE Investigators; Buller HR, Prins MH, Lensin AW, et al. Oral rivaroxaban for the treatment of symptomatic pulmonary embolism. N Engl J Med. 2012;366:1287-1297.

 

ILLUSTRATIVE CASE

A 55-year-old man is brought to the emergency department with shortness of breath, pleuritic chest pain, and hypoxia shortly after returning from an overseas business trip. High-resolution spiral computed tomography (CT) reveals a PE.

How should he be treated?

Pulmonary embolism (PE) is fairly common—with an annual incidence estimated at 69 per 100,0002—and the cause of significant morbidity and mortality. Up to 30% of patients with venous thromboembolism (VTE) die within a month of diagnosis, mostly from PE, and in about 25% of cases, PE presents as sudden death.3

Warfarin has a significant downside
Standard therapy consists of either unfractionated heparin or LMWH followed by warfarin, a vitamin K antagonist (VKA), for ≥3 months.4 In addition to requiring frequent laboratory monitoring, warfarin has potentially significant interactions with many prescription drugs. Numerous trials have investigated novel anticoagulants for treatment of VTE in recent years. In one randomized controlled trial (RCT), rivaroxaban (Xarelto)was found to be noninferior to a VKA for treating acute deep vein thrombosis (DVT).5

STUDY SUMMARY: Major bleeding is less likely with rivaroxaban

The EINSTEIN PE investigators conducted a randomized, unblinded noninferiority trial to determine whether rivaroxaban was at least as effective as the standard therapy—enoxaparin, followed by a dose-adjusted VKA (warfarin [for US patients] or acenocoumarol) for acute symptomatic PE.1 To be included, participants had to have PE confirmed by CT, ventilation perfusion scan, or pulmonary angiography, with or without accompanying DVT. Exclusion criteria included active bleeding, significant renal impairment (creatinine clearance <30 mL/min), >48 hours of heparin treatment, or more than one dose of a VKA.

Participants (N=4832 in 38 countries) were randomized to receive either rivaroxaban (15 mg twice daily for 3 weeks, then 20 mg once a day thereafter) or standard therapy. The intervention and control groups were similar. Just over half were male, with an average age of 58 years; three-quarters of the patients had an intermediate to extensive PE burden; and 90% were hospitalized for initial treatment. The researchers listed the etiology as unprovoked in 64% of the cases, followed by recent surgery or trauma and immobilization (17% and 16%, respectively).

After VKA initiation, the international normalized ratio (INR) was checked at least monthly. Patients in the control groups were within the target range (INR 1-2) 62% of the time, which is similar to other studies of anticoagulation in patients with VTE. Adherence to rivaroxaban was at least 80% in more than 94% of patients. Treatment lasted 3, 6, or 12 months, with the duration determined before randomization by the treating physician.

There was no difference in dropout rates (10.7% of rivaroxaban patients withdrew for any reason, vs 12.3% of the controls). Fewer than 0.5% of participants were lost to follow-up.

Symptomatic recurrent VTE, the primary outcome, occurred in 50 patients receiving rivaroxaban vs 44 of those on standard therapy (2.1% vs 1.8%; P=.003 for noninferiority using an intention-to-treat analysis). Major bleeding, defined as overt bleeding causing death, a drop in hemoglobin of ≥2 points, needing a transfusion, or bleeding in a critical site, occurred less often in the rivaroxaban group (1.1% vs 2.2%, P=.003, NNT=91). There was no significant difference in overall bleeding rates between the 2 groups.1

WHAT’S NEW: Rivaroxaban is easier to use—and on label

This trial found rivaroxaban to be at least as effective as enoxaparin followed by a dose-adjusted VKA for acute symptomatic PE, with fewer major bleeding events. What’s more, rivaroxaban—which now has US Food and Drug Administration approval for the prevention and treatment of PE and DVT6—does not require laboratory monitoring.

 

 

 

CAVEATS: Questions about study population, duration remain

This was an open-label study—neither patients nor investigators were blinded to the group assignments after randomization. The investigators suspected more recurrent VTE in those receiving rivaroxaban, which could have biased their findings in favor of the standard treatment. However, actual rates of recurrence were similar.

Study participants were <60 years old, on average, which may limit extrapolation to an older population. This trial lasted 12 months; the effects of longer treatment with rivaroxaban are unknown. Bayer HealthCare and Janssen Pharmaceuticals, who jointly manufacture rivaroxaban, funded the study.

CHALLENGES TO IMPLEMENTATION: Cost and lack of antidote may limit use

Rivaroxaban is more expensive than warfarin: A one-month supply costs approximately $260, while a month’s supply of warfarin plus lab monitoring runs less than $100.7 What’s more, factor Xa inhibitors, unlike VKAs, do not have a readily available pharmacologic antidote.

Acknowledgement

The PURLs Surveillance System is supported in part by Grant Number UL1RR024999 from the National Center for Research Resources, a Clinical Translational Science Award to the University of Chicago. The content is solely the responsibility of the authors and does not necessarily represent the official views of the National Center for Research Resources or the National Institutes of Health.

PRACTICE CHANGER

Consider treating patients with acute pulmonary embolism (PE) with rivaroxaban, a factor Xa inhibitor; it works as well as low-molecular-weight heparin (LMWH) followed by warfarin, but may cause fewer major bleeds.1

STRENGTH OF RECOMMENDATION

B: Based on a single, nonblinded randomized controlled trial.

EINSTEIN-PE Investigators; Buller HR, Prins MH, Lensin AW, et al. Oral rivaroxaban for the treatment of symptomatic pulmonary embolism. N Engl J Med. 2012;366:1287-1297.

 

ILLUSTRATIVE CASE

A 55-year-old man is brought to the emergency department with shortness of breath, pleuritic chest pain, and hypoxia shortly after returning from an overseas business trip. High-resolution spiral computed tomography (CT) reveals a PE.

How should he be treated?

Pulmonary embolism (PE) is fairly common—with an annual incidence estimated at 69 per 100,0002—and the cause of significant morbidity and mortality. Up to 30% of patients with venous thromboembolism (VTE) die within a month of diagnosis, mostly from PE, and in about 25% of cases, PE presents as sudden death.3

Warfarin has a significant downside
Standard therapy consists of either unfractionated heparin or LMWH followed by warfarin, a vitamin K antagonist (VKA), for ≥3 months.4 In addition to requiring frequent laboratory monitoring, warfarin has potentially significant interactions with many prescription drugs. Numerous trials have investigated novel anticoagulants for treatment of VTE in recent years. In one randomized controlled trial (RCT), rivaroxaban (Xarelto)was found to be noninferior to a VKA for treating acute deep vein thrombosis (DVT).5

STUDY SUMMARY: Major bleeding is less likely with rivaroxaban

The EINSTEIN PE investigators conducted a randomized, unblinded noninferiority trial to determine whether rivaroxaban was at least as effective as the standard therapy—enoxaparin, followed by a dose-adjusted VKA (warfarin [for US patients] or acenocoumarol) for acute symptomatic PE.1 To be included, participants had to have PE confirmed by CT, ventilation perfusion scan, or pulmonary angiography, with or without accompanying DVT. Exclusion criteria included active bleeding, significant renal impairment (creatinine clearance <30 mL/min), >48 hours of heparin treatment, or more than one dose of a VKA.

Participants (N=4832 in 38 countries) were randomized to receive either rivaroxaban (15 mg twice daily for 3 weeks, then 20 mg once a day thereafter) or standard therapy. The intervention and control groups were similar. Just over half were male, with an average age of 58 years; three-quarters of the patients had an intermediate to extensive PE burden; and 90% were hospitalized for initial treatment. The researchers listed the etiology as unprovoked in 64% of the cases, followed by recent surgery or trauma and immobilization (17% and 16%, respectively).

After VKA initiation, the international normalized ratio (INR) was checked at least monthly. Patients in the control groups were within the target range (INR 1-2) 62% of the time, which is similar to other studies of anticoagulation in patients with VTE. Adherence to rivaroxaban was at least 80% in more than 94% of patients. Treatment lasted 3, 6, or 12 months, with the duration determined before randomization by the treating physician.

There was no difference in dropout rates (10.7% of rivaroxaban patients withdrew for any reason, vs 12.3% of the controls). Fewer than 0.5% of participants were lost to follow-up.

Symptomatic recurrent VTE, the primary outcome, occurred in 50 patients receiving rivaroxaban vs 44 of those on standard therapy (2.1% vs 1.8%; P=.003 for noninferiority using an intention-to-treat analysis). Major bleeding, defined as overt bleeding causing death, a drop in hemoglobin of ≥2 points, needing a transfusion, or bleeding in a critical site, occurred less often in the rivaroxaban group (1.1% vs 2.2%, P=.003, NNT=91). There was no significant difference in overall bleeding rates between the 2 groups.1

WHAT’S NEW: Rivaroxaban is easier to use—and on label

This trial found rivaroxaban to be at least as effective as enoxaparin followed by a dose-adjusted VKA for acute symptomatic PE, with fewer major bleeding events. What’s more, rivaroxaban—which now has US Food and Drug Administration approval for the prevention and treatment of PE and DVT6—does not require laboratory monitoring.

 

 

 

CAVEATS: Questions about study population, duration remain

This was an open-label study—neither patients nor investigators were blinded to the group assignments after randomization. The investigators suspected more recurrent VTE in those receiving rivaroxaban, which could have biased their findings in favor of the standard treatment. However, actual rates of recurrence were similar.

Study participants were <60 years old, on average, which may limit extrapolation to an older population. This trial lasted 12 months; the effects of longer treatment with rivaroxaban are unknown. Bayer HealthCare and Janssen Pharmaceuticals, who jointly manufacture rivaroxaban, funded the study.

CHALLENGES TO IMPLEMENTATION: Cost and lack of antidote may limit use

Rivaroxaban is more expensive than warfarin: A one-month supply costs approximately $260, while a month’s supply of warfarin plus lab monitoring runs less than $100.7 What’s more, factor Xa inhibitors, unlike VKAs, do not have a readily available pharmacologic antidote.

Acknowledgement

The PURLs Surveillance System is supported in part by Grant Number UL1RR024999 from the National Center for Research Resources, a Clinical Translational Science Award to the University of Chicago. The content is solely the responsibility of the authors and does not necessarily represent the official views of the National Center for Research Resources or the National Institutes of Health.

References

1. Buller HR, Prins MH, Lensin AW, et al. EINSTEIN-PE Investigators; Oral rivaroxaban for the treatment of symptomatic pulmonary embolism. N Engl J Med. 2012;366:1287-1297.

2. Silverstein MD, Heit JA, Mohr DN, et al. Trends in the incidence of deep vein thrombosis and pulmonary embolism: a 25-year population-based study. Arch Intern Med. 1998;158:585-593.

3. Beckman MG, Hooper WC, Critchley SE, et al. Venous thromboembolism: a public health concern. Am J Prev Med. 2010;38(suppl):S495-S501.

4. Kearon C, Akl EA, Comerota AJ, et al. Antithrombotic therapy for VTE disease: antithrombotic therapy and prevention of thrombosis, 9th ed: American College of Chest Physicians Evidence-Based Clinical Practice Guidelines. Chest. 2012;141(suppl):e419S-e494S.

5. Bauersachs R, Berkowitz SD, Brenner B, et al. Oral rivaroxaban for symptomatic venous thromboembolism. N Engl J Med. 2010;363:2499-2510.

6. US Food and Drug Administration. FDA expands use of Xarelto to treat, reduce recurrence of blood clots [press release]. November 2, 2012. Available at: http://www.fda.gov/NewsEvents/Newsroom/PressAnnouncements/ucm326654.htm. Accessed November 12, 2012.

7. PL detail-document. Comparison of oral antithrombotics. Prescriber’s Letter. 2011;18:271020.-

References

1. Buller HR, Prins MH, Lensin AW, et al. EINSTEIN-PE Investigators; Oral rivaroxaban for the treatment of symptomatic pulmonary embolism. N Engl J Med. 2012;366:1287-1297.

2. Silverstein MD, Heit JA, Mohr DN, et al. Trends in the incidence of deep vein thrombosis and pulmonary embolism: a 25-year population-based study. Arch Intern Med. 1998;158:585-593.

3. Beckman MG, Hooper WC, Critchley SE, et al. Venous thromboembolism: a public health concern. Am J Prev Med. 2010;38(suppl):S495-S501.

4. Kearon C, Akl EA, Comerota AJ, et al. Antithrombotic therapy for VTE disease: antithrombotic therapy and prevention of thrombosis, 9th ed: American College of Chest Physicians Evidence-Based Clinical Practice Guidelines. Chest. 2012;141(suppl):e419S-e494S.

5. Bauersachs R, Berkowitz SD, Brenner B, et al. Oral rivaroxaban for symptomatic venous thromboembolism. N Engl J Med. 2010;363:2499-2510.

6. US Food and Drug Administration. FDA expands use of Xarelto to treat, reduce recurrence of blood clots [press release]. November 2, 2012. Available at: http://www.fda.gov/NewsEvents/Newsroom/PressAnnouncements/ucm326654.htm. Accessed November 12, 2012.

7. PL detail-document. Comparison of oral antithrombotics. Prescriber’s Letter. 2011;18:271020.-

Issue
The Journal of Family Practice - 61(12)
Issue
The Journal of Family Practice - 61(12)
Page Number
751-752
Page Number
751-752
Publications
Publications
Topics
Article Type
Display Headline
An alternative to warfarin for patients with PE
Display Headline
An alternative to warfarin for patients with PE
Legacy Keywords
Laura Morris;MD;MSPH; James J. Stevermer;MD;MSPH; alternative to warfarin; pulmonary embolism; PE; rivaroxaban; factor Xa inhibitor; low-molecular-weight heparin; vitamin K antagonist; VKA; laboratory monitoring; VTE; venous thromboembolism
Legacy Keywords
Laura Morris;MD;MSPH; James J. Stevermer;MD;MSPH; alternative to warfarin; pulmonary embolism; PE; rivaroxaban; factor Xa inhibitor; low-molecular-weight heparin; vitamin K antagonist; VKA; laboratory monitoring; VTE; venous thromboembolism
Sections
PURLs Copyright

Copyright © 2012 The Family Physicians Inquiries Network. All rights reserved.

Disallow All Ads
Alternative CME
Use ProPublica
Hide sidebar & use full width
render the right sidebar.
Conference Recap Checkbox
Not Conference Recap
Clinical Edge
Display the Slideshow in this Article
Article PDF Media
Media Files

A safer way to prevent VTE recurrence

Article Type
Changed
Thu, 07/16/2020 - 13:57
Display Headline
A safer way to prevent VTE recurrence
PRACTICE CHANGER

After patients with unprovoked venous thromboembolism (VTE) complete a 6- to 18-month course of oral anticoagulation therapy, consider a switch to aspirin.1

STRENGTH OF RECOMMENDATION

A: Based on one well-designed, randomized controlled trial (RCT).

Becattini C, Agnelli G, Schenone A, et al. Aspirin for preventing the recurrence of venous thromboembolism. N Engl J Med. 2012;366:1959-1967.

 

ILLUSTRATIVE CASE

A 62-year-old patient comes to your office for follow-up of a primary unprovoked venous thromboembolus. He has been on an oral anticoagulant for 12 months. Should he continue anticoagulation therapy despite the increased risk for major bleeding?

Patients who survive VTE—defined as either deep venous thrombosis or pulmonary embolism—are put on anticoagulant therapy to prevent a recurrence, typically for 6 to 18 months. But about 20% of patients with unprovoked VTE have a recurrence within 2 years of anticoagulation withdrawal.2 Extending anticoagulation prevents recurrences but increases the risk of bleeding.3

Is aspirin a viable alternative?
Until recently, the efficacy of aspirin for the prevention of recurrent VTE was unknown. Becattini et al investigated it in the multicenter RCT detailed in this PURL.

STUDY SUMMARY: Aspirin can prevent recurrence with minimal risk

To determine whether aspirin was a viable alternative to oral anticoagulation, the researchers compared aspirin with placebo in patients with primary unprovoked VTE who had completed a course of oral anticoagulation treatment. To be considered for the study, patients had to be >18 years and have had their first-ever objectively confirmed, symptomatic unprovoked proximal deep vein thrombosis, pulmonary embolism, or both. They also had to have completed 6 to 18 months of anticoagulant therapy, with a target international normalized ratio (INR) of 2.0 to 3.0. Exclusion criteria included a history of cancer, clinically significant thrombophilia, atrial fibrillation, and a bleeding event that occurred during the course of anticoagulation therapy.

Becattini et al identified 403 eligible patients. Two weeks after stopping anticoagulation, patients were randomly assigned to receive either aspirin 100 mg once daily (n=205) or placebo (n=198) for 2 years. (One patient in the placebo group never received treatment.) At baseline, there were no significant differences in patient characteristics. All were evaluated every 3 months in the first year and every 6 months in the second year.

The primary efficacy outcome was objectively confirmed recurrent VTE. The primary safety outcome was major bleeding, defined as bleeding that occurred in a critical location (eg, intracranial bleeding), was associated with a decrease of hemoglobin of at least 2 g/dL, required a transfusion of 2 units of whole blood or red blood cells, or was fatal. Overt bleeding, which required medical intervention but did not meet the criteria for major bleeding, was a secondary safety outcome.

Twenty-eight of the 205 patients in the aspirin group experienced a recurrence, compared with 43 of the 197 patients on placebo (6.6% vs. 11.2% per year; hazard ratio [HR]=0.58; 95% confidence interval [CI], 0.36-0.93; P=.02). Adverse events were reported by 7 patients in the aspirin therapy group and 6 in the placebo group. One patient in each group experienced major bleeding, and 3 in each group experienced clinically relevant but nonmajor bleeding. Withdrawal rates were similar (10 in the treatment group vs 9 controls), as were the number of patients who developed new indications for aspirin or anticoagulation therapy or were lost to follow-up.

 

 

 

An analysis adjusted for age, sex, index event (deep vein thrombosis or pulmonary embolism), and duration of initial anticoagulation treatment confirmed that aspirin reduced the risk of recurrence (adjusted HR=0.53; 95% CI, 0.32-0.85; P=.009). No association was found between recurrent VTE and duration of anticoagulation therapy (6 months vs longer). Nor was there a difference in recurrence rates based on the index event.

WHAT’S NEW: Aspirin has a key role in preventing recurrence

This study found that for patients with unprovoked VTE who completed a course of oral anticoagulation, aspirin was effective in preventing a recurrence, with no apparent increase in the risk of major bleeding. Protection in Year 2 was nearly as great as in Year one.1

CAVEAT: Patients were followed for just 2 years

It is unclear whether continuing aspirin therapy beyond 2 years would continue to confer protection against a VTE recurrence without an increase in adverse effects.

CHALLENGE TO IMPLEMENTATION: Some patients can’t tolerate chronic aspirin therapy

Although this study investigated aspirin in a dosage of 100 mg/d, this strength is not readily available in the United States.4 There is no evidence to suggest that the 81-mg strength that is available in this country would provide a diminished antiplatelet effect. And, as is already customary, patients undergoing chronic aspirin therapy must be monitored for major bleeding, GI irritation, and renal compromise. A few patients will be ineligible for prophylaxis due to a history of intolerance to aspirin or nonsteroidal anti-inflammatory drugs.

Acknowledgement

The PURLs Surveillance System was supported in part by Grant Number UL1RR024999 from the National Center for Research Resources, a Clinical Translational Science Award to the University of Chicago. The content is solely the responsibility of the authors and does not necessarily represent the official views of the National Center for Research Resources or the National Institutes of Health.

Files
References

1. Becattini C, Agnelli G, Schenone A, et al. Aspirin for preventing the recurrence of venous thromboembolism. N Engl J Med. 2012;366:1959-1967.

2. Prandoni P, Lensing AW, Cogo A, et al. The long-term clinical course of acute deep venous thrombosis. Ann Intern Med. 1996;125:1-7.

3. Kearon C, Akl EA, Comerota AJ, et al. Antithrombotic therapy for VTE disease: antithrombotic therapy and prevention of thrombosis, 9th ed: American College of Chest Physicians evidence-based clinical practice guidelines. Chest. 2012;141(2 suppl):e419S-494S.

4. Daily Med. Aspirin. Available at: http://dailymed.nlm.nih.gov/dailymed/search.cfm?startswith=aspirin. Accessed September 6, 2012.

Article PDF
Author and Disclosure Information

Altaf Kaiseruddin, MD
The University of Chicago

Sonia Oyola, MD
The University of Chicago

Goutham Rao, MD
The University of Chicago

PURLs EDITOR
Bernard Ewigman, MD, MSPH
The University of Chicago

Issue
The Journal of Family Practice - 61(11)
Publications
Topics
Page Number
673-674
Legacy Keywords
ltaf Kaiseruddin;MD; Sonia Oyola;MD; Goutham Rao;MD; oral anticoagulation; venous thromboembolism; aspirin; major bleeding; deep vein thrombosis; international normalized ratio; INR; atrial fibrillation
Sections
Files
Files
Author and Disclosure Information

Altaf Kaiseruddin, MD
The University of Chicago

Sonia Oyola, MD
The University of Chicago

Goutham Rao, MD
The University of Chicago

PURLs EDITOR
Bernard Ewigman, MD, MSPH
The University of Chicago

Author and Disclosure Information

Altaf Kaiseruddin, MD
The University of Chicago

Sonia Oyola, MD
The University of Chicago

Goutham Rao, MD
The University of Chicago

PURLs EDITOR
Bernard Ewigman, MD, MSPH
The University of Chicago

Article PDF
Article PDF
PRACTICE CHANGER

After patients with unprovoked venous thromboembolism (VTE) complete a 6- to 18-month course of oral anticoagulation therapy, consider a switch to aspirin.1

STRENGTH OF RECOMMENDATION

A: Based on one well-designed, randomized controlled trial (RCT).

Becattini C, Agnelli G, Schenone A, et al. Aspirin for preventing the recurrence of venous thromboembolism. N Engl J Med. 2012;366:1959-1967.

 

ILLUSTRATIVE CASE

A 62-year-old patient comes to your office for follow-up of a primary unprovoked venous thromboembolus. He has been on an oral anticoagulant for 12 months. Should he continue anticoagulation therapy despite the increased risk for major bleeding?

Patients who survive VTE—defined as either deep venous thrombosis or pulmonary embolism—are put on anticoagulant therapy to prevent a recurrence, typically for 6 to 18 months. But about 20% of patients with unprovoked VTE have a recurrence within 2 years of anticoagulation withdrawal.2 Extending anticoagulation prevents recurrences but increases the risk of bleeding.3

Is aspirin a viable alternative?
Until recently, the efficacy of aspirin for the prevention of recurrent VTE was unknown. Becattini et al investigated it in the multicenter RCT detailed in this PURL.

STUDY SUMMARY: Aspirin can prevent recurrence with minimal risk

To determine whether aspirin was a viable alternative to oral anticoagulation, the researchers compared aspirin with placebo in patients with primary unprovoked VTE who had completed a course of oral anticoagulation treatment. To be considered for the study, patients had to be >18 years and have had their first-ever objectively confirmed, symptomatic unprovoked proximal deep vein thrombosis, pulmonary embolism, or both. They also had to have completed 6 to 18 months of anticoagulant therapy, with a target international normalized ratio (INR) of 2.0 to 3.0. Exclusion criteria included a history of cancer, clinically significant thrombophilia, atrial fibrillation, and a bleeding event that occurred during the course of anticoagulation therapy.

Becattini et al identified 403 eligible patients. Two weeks after stopping anticoagulation, patients were randomly assigned to receive either aspirin 100 mg once daily (n=205) or placebo (n=198) for 2 years. (One patient in the placebo group never received treatment.) At baseline, there were no significant differences in patient characteristics. All were evaluated every 3 months in the first year and every 6 months in the second year.

The primary efficacy outcome was objectively confirmed recurrent VTE. The primary safety outcome was major bleeding, defined as bleeding that occurred in a critical location (eg, intracranial bleeding), was associated with a decrease of hemoglobin of at least 2 g/dL, required a transfusion of 2 units of whole blood or red blood cells, or was fatal. Overt bleeding, which required medical intervention but did not meet the criteria for major bleeding, was a secondary safety outcome.

Twenty-eight of the 205 patients in the aspirin group experienced a recurrence, compared with 43 of the 197 patients on placebo (6.6% vs. 11.2% per year; hazard ratio [HR]=0.58; 95% confidence interval [CI], 0.36-0.93; P=.02). Adverse events were reported by 7 patients in the aspirin therapy group and 6 in the placebo group. One patient in each group experienced major bleeding, and 3 in each group experienced clinically relevant but nonmajor bleeding. Withdrawal rates were similar (10 in the treatment group vs 9 controls), as were the number of patients who developed new indications for aspirin or anticoagulation therapy or were lost to follow-up.

 

 

 

An analysis adjusted for age, sex, index event (deep vein thrombosis or pulmonary embolism), and duration of initial anticoagulation treatment confirmed that aspirin reduced the risk of recurrence (adjusted HR=0.53; 95% CI, 0.32-0.85; P=.009). No association was found between recurrent VTE and duration of anticoagulation therapy (6 months vs longer). Nor was there a difference in recurrence rates based on the index event.

WHAT’S NEW: Aspirin has a key role in preventing recurrence

This study found that for patients with unprovoked VTE who completed a course of oral anticoagulation, aspirin was effective in preventing a recurrence, with no apparent increase in the risk of major bleeding. Protection in Year 2 was nearly as great as in Year one.1

CAVEAT: Patients were followed for just 2 years

It is unclear whether continuing aspirin therapy beyond 2 years would continue to confer protection against a VTE recurrence without an increase in adverse effects.

CHALLENGE TO IMPLEMENTATION: Some patients can’t tolerate chronic aspirin therapy

Although this study investigated aspirin in a dosage of 100 mg/d, this strength is not readily available in the United States.4 There is no evidence to suggest that the 81-mg strength that is available in this country would provide a diminished antiplatelet effect. And, as is already customary, patients undergoing chronic aspirin therapy must be monitored for major bleeding, GI irritation, and renal compromise. A few patients will be ineligible for prophylaxis due to a history of intolerance to aspirin or nonsteroidal anti-inflammatory drugs.

Acknowledgement

The PURLs Surveillance System was supported in part by Grant Number UL1RR024999 from the National Center for Research Resources, a Clinical Translational Science Award to the University of Chicago. The content is solely the responsibility of the authors and does not necessarily represent the official views of the National Center for Research Resources or the National Institutes of Health.

PRACTICE CHANGER

After patients with unprovoked venous thromboembolism (VTE) complete a 6- to 18-month course of oral anticoagulation therapy, consider a switch to aspirin.1

STRENGTH OF RECOMMENDATION

A: Based on one well-designed, randomized controlled trial (RCT).

Becattini C, Agnelli G, Schenone A, et al. Aspirin for preventing the recurrence of venous thromboembolism. N Engl J Med. 2012;366:1959-1967.

 

ILLUSTRATIVE CASE

A 62-year-old patient comes to your office for follow-up of a primary unprovoked venous thromboembolus. He has been on an oral anticoagulant for 12 months. Should he continue anticoagulation therapy despite the increased risk for major bleeding?

Patients who survive VTE—defined as either deep venous thrombosis or pulmonary embolism—are put on anticoagulant therapy to prevent a recurrence, typically for 6 to 18 months. But about 20% of patients with unprovoked VTE have a recurrence within 2 years of anticoagulation withdrawal.2 Extending anticoagulation prevents recurrences but increases the risk of bleeding.3

Is aspirin a viable alternative?
Until recently, the efficacy of aspirin for the prevention of recurrent VTE was unknown. Becattini et al investigated it in the multicenter RCT detailed in this PURL.

STUDY SUMMARY: Aspirin can prevent recurrence with minimal risk

To determine whether aspirin was a viable alternative to oral anticoagulation, the researchers compared aspirin with placebo in patients with primary unprovoked VTE who had completed a course of oral anticoagulation treatment. To be considered for the study, patients had to be >18 years and have had their first-ever objectively confirmed, symptomatic unprovoked proximal deep vein thrombosis, pulmonary embolism, or both. They also had to have completed 6 to 18 months of anticoagulant therapy, with a target international normalized ratio (INR) of 2.0 to 3.0. Exclusion criteria included a history of cancer, clinically significant thrombophilia, atrial fibrillation, and a bleeding event that occurred during the course of anticoagulation therapy.

Becattini et al identified 403 eligible patients. Two weeks after stopping anticoagulation, patients were randomly assigned to receive either aspirin 100 mg once daily (n=205) or placebo (n=198) for 2 years. (One patient in the placebo group never received treatment.) At baseline, there were no significant differences in patient characteristics. All were evaluated every 3 months in the first year and every 6 months in the second year.

The primary efficacy outcome was objectively confirmed recurrent VTE. The primary safety outcome was major bleeding, defined as bleeding that occurred in a critical location (eg, intracranial bleeding), was associated with a decrease of hemoglobin of at least 2 g/dL, required a transfusion of 2 units of whole blood or red blood cells, or was fatal. Overt bleeding, which required medical intervention but did not meet the criteria for major bleeding, was a secondary safety outcome.

Twenty-eight of the 205 patients in the aspirin group experienced a recurrence, compared with 43 of the 197 patients on placebo (6.6% vs. 11.2% per year; hazard ratio [HR]=0.58; 95% confidence interval [CI], 0.36-0.93; P=.02). Adverse events were reported by 7 patients in the aspirin therapy group and 6 in the placebo group. One patient in each group experienced major bleeding, and 3 in each group experienced clinically relevant but nonmajor bleeding. Withdrawal rates were similar (10 in the treatment group vs 9 controls), as were the number of patients who developed new indications for aspirin or anticoagulation therapy or were lost to follow-up.

 

 

 

An analysis adjusted for age, sex, index event (deep vein thrombosis or pulmonary embolism), and duration of initial anticoagulation treatment confirmed that aspirin reduced the risk of recurrence (adjusted HR=0.53; 95% CI, 0.32-0.85; P=.009). No association was found between recurrent VTE and duration of anticoagulation therapy (6 months vs longer). Nor was there a difference in recurrence rates based on the index event.

WHAT’S NEW: Aspirin has a key role in preventing recurrence

This study found that for patients with unprovoked VTE who completed a course of oral anticoagulation, aspirin was effective in preventing a recurrence, with no apparent increase in the risk of major bleeding. Protection in Year 2 was nearly as great as in Year one.1

CAVEAT: Patients were followed for just 2 years

It is unclear whether continuing aspirin therapy beyond 2 years would continue to confer protection against a VTE recurrence without an increase in adverse effects.

CHALLENGE TO IMPLEMENTATION: Some patients can’t tolerate chronic aspirin therapy

Although this study investigated aspirin in a dosage of 100 mg/d, this strength is not readily available in the United States.4 There is no evidence to suggest that the 81-mg strength that is available in this country would provide a diminished antiplatelet effect. And, as is already customary, patients undergoing chronic aspirin therapy must be monitored for major bleeding, GI irritation, and renal compromise. A few patients will be ineligible for prophylaxis due to a history of intolerance to aspirin or nonsteroidal anti-inflammatory drugs.

Acknowledgement

The PURLs Surveillance System was supported in part by Grant Number UL1RR024999 from the National Center for Research Resources, a Clinical Translational Science Award to the University of Chicago. The content is solely the responsibility of the authors and does not necessarily represent the official views of the National Center for Research Resources or the National Institutes of Health.

References

1. Becattini C, Agnelli G, Schenone A, et al. Aspirin for preventing the recurrence of venous thromboembolism. N Engl J Med. 2012;366:1959-1967.

2. Prandoni P, Lensing AW, Cogo A, et al. The long-term clinical course of acute deep venous thrombosis. Ann Intern Med. 1996;125:1-7.

3. Kearon C, Akl EA, Comerota AJ, et al. Antithrombotic therapy for VTE disease: antithrombotic therapy and prevention of thrombosis, 9th ed: American College of Chest Physicians evidence-based clinical practice guidelines. Chest. 2012;141(2 suppl):e419S-494S.

4. Daily Med. Aspirin. Available at: http://dailymed.nlm.nih.gov/dailymed/search.cfm?startswith=aspirin. Accessed September 6, 2012.

References

1. Becattini C, Agnelli G, Schenone A, et al. Aspirin for preventing the recurrence of venous thromboembolism. N Engl J Med. 2012;366:1959-1967.

2. Prandoni P, Lensing AW, Cogo A, et al. The long-term clinical course of acute deep venous thrombosis. Ann Intern Med. 1996;125:1-7.

3. Kearon C, Akl EA, Comerota AJ, et al. Antithrombotic therapy for VTE disease: antithrombotic therapy and prevention of thrombosis, 9th ed: American College of Chest Physicians evidence-based clinical practice guidelines. Chest. 2012;141(2 suppl):e419S-494S.

4. Daily Med. Aspirin. Available at: http://dailymed.nlm.nih.gov/dailymed/search.cfm?startswith=aspirin. Accessed September 6, 2012.

Issue
The Journal of Family Practice - 61(11)
Issue
The Journal of Family Practice - 61(11)
Page Number
673-674
Page Number
673-674
Publications
Publications
Topics
Article Type
Display Headline
A safer way to prevent VTE recurrence
Display Headline
A safer way to prevent VTE recurrence
Legacy Keywords
ltaf Kaiseruddin;MD; Sonia Oyola;MD; Goutham Rao;MD; oral anticoagulation; venous thromboembolism; aspirin; major bleeding; deep vein thrombosis; international normalized ratio; INR; atrial fibrillation
Legacy Keywords
ltaf Kaiseruddin;MD; Sonia Oyola;MD; Goutham Rao;MD; oral anticoagulation; venous thromboembolism; aspirin; major bleeding; deep vein thrombosis; international normalized ratio; INR; atrial fibrillation
Sections
PURLs Copyright

Copyright © 2012 The Family Physicians Inquiries Network. All rights reserved.

Disallow All Ads
Alternative CME
Use ProPublica
Hide sidebar & use full width
render the right sidebar.
Conference Recap Checkbox
Not Conference Recap
Clinical Edge
Display the Slideshow in this Article
Article PDF Media
Media Files

Patient overusing antianxiety meds? Say so (in a letter)

Article Type
Changed
Mon, 01/14/2019 - 11:35
Display Headline
Patient overusing antianxiety meds? Say so (in a letter)

PRACTICE CHANGER

Express your concern about long-term use of benzodiazepines in a letter—a simple intervention that patients often respond to by reducing or eliminating their use of the drug.1

STRENGTH OF RECOMMENDATION

A: Based on a well-done meta-analysis with few clinical trials.

Mugunthan K, McGuire T, Glasziou P. Minimal interventions to decrease long-term use of benzodiazepines in primary care: a systematic review and meta-analysis. Br J Gen Pract. 2011;61:e573-e578.

ILLUSTRATIVE CASE

A 65-year-old patient has been taking lorazepam for insomnia for more than a year. You are concerned about her ongoing use of the benzodiazepine and want to wean her from the medication. What strategies can you use to decrease, or eliminate, her use of the drug?

Benzodiazepines are commonly used medications, with an estimated 12-month prevalence of use of 8.6% in the United States.2 While short-term use of these antianxiety medications can be effective, long-term use (defined as regular use for >3 months) is associated with significant risk.

Abuse linked to chronic use
Prescription drug abuse has recently become the nation’s leading cause of accidental death, overtaking motor vehicle accidents.3 And tranquilizers, including benzodiazepines, are the second most abused prescription medication, after pain relievers.4 In addition to dependence and withdrawal, chronic use of benzodiazepines is associated with daytime somnolence, blunted reflexes, memory loss, cognitive impairment, and an increased risk of falls and fractures—particularly in older patients.5

Reducing long-term use of benzodiazepines in a primary care setting is important but challenging. Until recently, most of the successful strategies reported were resource intensive and required multiple office visits.6

STUDY SUMMARY: Brief interventions are often effective

This study was a meta-analysis of randomized controlled trials in which “minimal interventions” were compared with usual care for their effectiveness in reducing or eliminating benzodiazepine use in primary care patients. A minimal intervention was defined as a letter, self-help information, or short consultation with a primary care provider. In each case, the message to the patient included (a) an expression of concern about the patient’s long-term use of the medication, (b) information about the potential adverse effects of the medication, and (c) advice on how to gradually reduce or stop using it.

Three studies met the inclusion criteria for randomization, blinding, and analysis by intention-to-treat.7-9 All 3 (n=615) had a 6-month follow-up period, a higher proportion of women (>60%), and participants with a mean age >60 years. Few patients were lost to follow-up; withdrawal rates were low and similar in all 3 studies. Each study compared a letter with usual care; 2 of the 3 had a third arm that included both a letter and a short consultation.

Pooled results from the studies showed twice the reduction in benzodiazepine use in the intervention groups compared with the control groups (risk ratio [RR]=2.04; 95% confidence interval [CI], 1.5-2.8; P< .001). The RR for cessation of benzodiazepine use was 2.3 (95% CI, 1.3-4.2; P= .003). The number needed to treat for a reduction or cessation of use was 12. The studies reported benzodiazepine reduction rates of 20% to 35% in the intervention groups vs 6% to 15% in the usual care groups.7-9 There appeared to be no additional benefit to adding the brief consultation compared with the letter alone.

WHAT’S NEW?: This strategy is easy to implement

While many methods to reduce benzodiazepine use have been studied, most involved levels of skill and resources that are not feasible for widespread use. This study found that a letter, stating the risks of continued use of the medication and providing a weaning schedule and tips for handling withdrawal, can be effective in reducing chronic use in a small but significant part of the population.

CAVEATS: Effects of withdrawal went unaddressed

The study did not adequately address the adverse effects of withdrawal from benzodiazepines, with one of the studies reporting significantly worse qualitative (but not quantitative) withdrawal symptoms at 6 months.7 This is of particular concern, as withdrawal symptoms are associated with the potential for relapse and concomitant abuse of other drugs and alcohol. We recommend that primary care physicians screen for substance abuse prior to the intervention and arrange for adequate follow-up.

All 3 studies in the meta-analysis lasted 6 months; no longer-term outcomes were reported. In addition, the study did not yield enough information to identify patients who would be most likely to respond to this brief intervention.

CHALLENGES TO IMPLEMENTATION: Determining which patients to target

 

 

Identifying patients who are appropriate candidates for this brief intervention and providing adequate monitoring for adverse effects of withdrawal are the main challenges of this practice changer. Nonetheless, chronic benzodiazepine use is of considerable concern, and we believe that this is a useful, and manageable intervention.

Acknowledgement

The PURLs Surveillance System is supported in part by Grant Number UL1RR024999 from the National Center for Research Resources, a Clinical Translational Science Award to the University of Chicago. The content is solely the responsibility of the authors and does not necessarily represent the official views of the National Center for Research Resources or the National Institutes of Health.

References

1. Mugunthan K, McGuire T, Glasziou P. Minimal interventions to decrease long-term use of benzodiazepines in primary care: a systematic review and meta-analysis. Br J Gen Pract. 2011;61:e573-e578.

2. Tyrer PJ. Benzodiazepines on trial. Br Med J. 1984;288:1101-1102.

3. Centers for Disease Control and Prevention. Deaths: Leading causes for 2008. June 6, 2012. Available at: http://www.cdc.gov/nchs/data/nvsr/nvsr60/nvsr60_06.pdf. Accessed October 10, 2012.

4. National Institute on Drug Abuse. Topics in brief: Prescription drug abuse. Available at: http://www.drugabuse.gov/publications/topics-in-brief/prescription-drug-abuse. Accessed October 11, 2012.

5. Morin CM, Bastien C, Guay B, et al. Randomized clinical trail of supervised tapering and cognitive behavior therapy to facilitate benzodiazepine discontinuation in older adults with chronic insomnia. Am J Psychiatry. 2004;161:332-342.

6. Oude Voshaar RC, Couvee JE, van Balkorn AJ, et al. Strategies for discontinuing long-term benzodiazepine use-meta-analysis. Br J Psychiatr. 2006;189:213-220.

7. Bashir K, King M, Ashworth M. Controlled evaluation of brief intervention by general practitioners to reduce chronic use of benzodiazepines. Br J Gen Pract. 1994;44:408-412.

8. Cormack MA, Sweeney KG, Hughes-Jones H, et al. Evaluation of an easy, cost-effective strategy to cut benzodiazepine use in general practice. Br J Gen Pract. 1994;44:5-8

9. Heather NA, Bowie A, Ashton H, et al. Randomized controlled trial of two brief interventions against long-term benzodiazepine use: outcome of intervention. Addict Res Theory. 2004;12:141-145.

Article PDF
Author and Disclosure Information

Robert Levy, MD
North Memorial, Family Medicine Residency, University of Minnesota, Minneapolis

Shailendra Prasad, MBBS, MPH
North Memorial, Family Medicine Residency, University of Minnesota, Minneapolis

PURLs EDITOR
James J. Stevermer, MD, MSPH

University of Missouri-Columbia

Issue
The Journal of Family Practice - 61(11)
Publications
Topics
Page Number
671-672
Legacy Keywords
Robert Levy;MD; Shailendra Prasad;MBBS;MPH; benzodiazepines; minimal interventions; inclusion criteria; primary care patients; short consultation; letter; withdrawal rates
Sections
Author and Disclosure Information

Robert Levy, MD
North Memorial, Family Medicine Residency, University of Minnesota, Minneapolis

Shailendra Prasad, MBBS, MPH
North Memorial, Family Medicine Residency, University of Minnesota, Minneapolis

PURLs EDITOR
James J. Stevermer, MD, MSPH

University of Missouri-Columbia

Author and Disclosure Information

Robert Levy, MD
North Memorial, Family Medicine Residency, University of Minnesota, Minneapolis

Shailendra Prasad, MBBS, MPH
North Memorial, Family Medicine Residency, University of Minnesota, Minneapolis

PURLs EDITOR
James J. Stevermer, MD, MSPH

University of Missouri-Columbia

Article PDF
Article PDF

PRACTICE CHANGER

Express your concern about long-term use of benzodiazepines in a letter—a simple intervention that patients often respond to by reducing or eliminating their use of the drug.1

STRENGTH OF RECOMMENDATION

A: Based on a well-done meta-analysis with few clinical trials.

Mugunthan K, McGuire T, Glasziou P. Minimal interventions to decrease long-term use of benzodiazepines in primary care: a systematic review and meta-analysis. Br J Gen Pract. 2011;61:e573-e578.

ILLUSTRATIVE CASE

A 65-year-old patient has been taking lorazepam for insomnia for more than a year. You are concerned about her ongoing use of the benzodiazepine and want to wean her from the medication. What strategies can you use to decrease, or eliminate, her use of the drug?

Benzodiazepines are commonly used medications, with an estimated 12-month prevalence of use of 8.6% in the United States.2 While short-term use of these antianxiety medications can be effective, long-term use (defined as regular use for >3 months) is associated with significant risk.

Abuse linked to chronic use
Prescription drug abuse has recently become the nation’s leading cause of accidental death, overtaking motor vehicle accidents.3 And tranquilizers, including benzodiazepines, are the second most abused prescription medication, after pain relievers.4 In addition to dependence and withdrawal, chronic use of benzodiazepines is associated with daytime somnolence, blunted reflexes, memory loss, cognitive impairment, and an increased risk of falls and fractures—particularly in older patients.5

Reducing long-term use of benzodiazepines in a primary care setting is important but challenging. Until recently, most of the successful strategies reported were resource intensive and required multiple office visits.6

STUDY SUMMARY: Brief interventions are often effective

This study was a meta-analysis of randomized controlled trials in which “minimal interventions” were compared with usual care for their effectiveness in reducing or eliminating benzodiazepine use in primary care patients. A minimal intervention was defined as a letter, self-help information, or short consultation with a primary care provider. In each case, the message to the patient included (a) an expression of concern about the patient’s long-term use of the medication, (b) information about the potential adverse effects of the medication, and (c) advice on how to gradually reduce or stop using it.

Three studies met the inclusion criteria for randomization, blinding, and analysis by intention-to-treat.7-9 All 3 (n=615) had a 6-month follow-up period, a higher proportion of women (>60%), and participants with a mean age >60 years. Few patients were lost to follow-up; withdrawal rates were low and similar in all 3 studies. Each study compared a letter with usual care; 2 of the 3 had a third arm that included both a letter and a short consultation.

Pooled results from the studies showed twice the reduction in benzodiazepine use in the intervention groups compared with the control groups (risk ratio [RR]=2.04; 95% confidence interval [CI], 1.5-2.8; P< .001). The RR for cessation of benzodiazepine use was 2.3 (95% CI, 1.3-4.2; P= .003). The number needed to treat for a reduction or cessation of use was 12. The studies reported benzodiazepine reduction rates of 20% to 35% in the intervention groups vs 6% to 15% in the usual care groups.7-9 There appeared to be no additional benefit to adding the brief consultation compared with the letter alone.

WHAT’S NEW?: This strategy is easy to implement

While many methods to reduce benzodiazepine use have been studied, most involved levels of skill and resources that are not feasible for widespread use. This study found that a letter, stating the risks of continued use of the medication and providing a weaning schedule and tips for handling withdrawal, can be effective in reducing chronic use in a small but significant part of the population.

CAVEATS: Effects of withdrawal went unaddressed

The study did not adequately address the adverse effects of withdrawal from benzodiazepines, with one of the studies reporting significantly worse qualitative (but not quantitative) withdrawal symptoms at 6 months.7 This is of particular concern, as withdrawal symptoms are associated with the potential for relapse and concomitant abuse of other drugs and alcohol. We recommend that primary care physicians screen for substance abuse prior to the intervention and arrange for adequate follow-up.

All 3 studies in the meta-analysis lasted 6 months; no longer-term outcomes were reported. In addition, the study did not yield enough information to identify patients who would be most likely to respond to this brief intervention.

CHALLENGES TO IMPLEMENTATION: Determining which patients to target

 

 

Identifying patients who are appropriate candidates for this brief intervention and providing adequate monitoring for adverse effects of withdrawal are the main challenges of this practice changer. Nonetheless, chronic benzodiazepine use is of considerable concern, and we believe that this is a useful, and manageable intervention.

Acknowledgement

The PURLs Surveillance System is supported in part by Grant Number UL1RR024999 from the National Center for Research Resources, a Clinical Translational Science Award to the University of Chicago. The content is solely the responsibility of the authors and does not necessarily represent the official views of the National Center for Research Resources or the National Institutes of Health.

PRACTICE CHANGER

Express your concern about long-term use of benzodiazepines in a letter—a simple intervention that patients often respond to by reducing or eliminating their use of the drug.1

STRENGTH OF RECOMMENDATION

A: Based on a well-done meta-analysis with few clinical trials.

Mugunthan K, McGuire T, Glasziou P. Minimal interventions to decrease long-term use of benzodiazepines in primary care: a systematic review and meta-analysis. Br J Gen Pract. 2011;61:e573-e578.

ILLUSTRATIVE CASE

A 65-year-old patient has been taking lorazepam for insomnia for more than a year. You are concerned about her ongoing use of the benzodiazepine and want to wean her from the medication. What strategies can you use to decrease, or eliminate, her use of the drug?

Benzodiazepines are commonly used medications, with an estimated 12-month prevalence of use of 8.6% in the United States.2 While short-term use of these antianxiety medications can be effective, long-term use (defined as regular use for >3 months) is associated with significant risk.

Abuse linked to chronic use
Prescription drug abuse has recently become the nation’s leading cause of accidental death, overtaking motor vehicle accidents.3 And tranquilizers, including benzodiazepines, are the second most abused prescription medication, after pain relievers.4 In addition to dependence and withdrawal, chronic use of benzodiazepines is associated with daytime somnolence, blunted reflexes, memory loss, cognitive impairment, and an increased risk of falls and fractures—particularly in older patients.5

Reducing long-term use of benzodiazepines in a primary care setting is important but challenging. Until recently, most of the successful strategies reported were resource intensive and required multiple office visits.6

STUDY SUMMARY: Brief interventions are often effective

This study was a meta-analysis of randomized controlled trials in which “minimal interventions” were compared with usual care for their effectiveness in reducing or eliminating benzodiazepine use in primary care patients. A minimal intervention was defined as a letter, self-help information, or short consultation with a primary care provider. In each case, the message to the patient included (a) an expression of concern about the patient’s long-term use of the medication, (b) information about the potential adverse effects of the medication, and (c) advice on how to gradually reduce or stop using it.

Three studies met the inclusion criteria for randomization, blinding, and analysis by intention-to-treat.7-9 All 3 (n=615) had a 6-month follow-up period, a higher proportion of women (>60%), and participants with a mean age >60 years. Few patients were lost to follow-up; withdrawal rates were low and similar in all 3 studies. Each study compared a letter with usual care; 2 of the 3 had a third arm that included both a letter and a short consultation.

Pooled results from the studies showed twice the reduction in benzodiazepine use in the intervention groups compared with the control groups (risk ratio [RR]=2.04; 95% confidence interval [CI], 1.5-2.8; P< .001). The RR for cessation of benzodiazepine use was 2.3 (95% CI, 1.3-4.2; P= .003). The number needed to treat for a reduction or cessation of use was 12. The studies reported benzodiazepine reduction rates of 20% to 35% in the intervention groups vs 6% to 15% in the usual care groups.7-9 There appeared to be no additional benefit to adding the brief consultation compared with the letter alone.

WHAT’S NEW?: This strategy is easy to implement

While many methods to reduce benzodiazepine use have been studied, most involved levels of skill and resources that are not feasible for widespread use. This study found that a letter, stating the risks of continued use of the medication and providing a weaning schedule and tips for handling withdrawal, can be effective in reducing chronic use in a small but significant part of the population.

CAVEATS: Effects of withdrawal went unaddressed

The study did not adequately address the adverse effects of withdrawal from benzodiazepines, with one of the studies reporting significantly worse qualitative (but not quantitative) withdrawal symptoms at 6 months.7 This is of particular concern, as withdrawal symptoms are associated with the potential for relapse and concomitant abuse of other drugs and alcohol. We recommend that primary care physicians screen for substance abuse prior to the intervention and arrange for adequate follow-up.

All 3 studies in the meta-analysis lasted 6 months; no longer-term outcomes were reported. In addition, the study did not yield enough information to identify patients who would be most likely to respond to this brief intervention.

CHALLENGES TO IMPLEMENTATION: Determining which patients to target

 

 

Identifying patients who are appropriate candidates for this brief intervention and providing adequate monitoring for adverse effects of withdrawal are the main challenges of this practice changer. Nonetheless, chronic benzodiazepine use is of considerable concern, and we believe that this is a useful, and manageable intervention.

Acknowledgement

The PURLs Surveillance System is supported in part by Grant Number UL1RR024999 from the National Center for Research Resources, a Clinical Translational Science Award to the University of Chicago. The content is solely the responsibility of the authors and does not necessarily represent the official views of the National Center for Research Resources or the National Institutes of Health.

References

1. Mugunthan K, McGuire T, Glasziou P. Minimal interventions to decrease long-term use of benzodiazepines in primary care: a systematic review and meta-analysis. Br J Gen Pract. 2011;61:e573-e578.

2. Tyrer PJ. Benzodiazepines on trial. Br Med J. 1984;288:1101-1102.

3. Centers for Disease Control and Prevention. Deaths: Leading causes for 2008. June 6, 2012. Available at: http://www.cdc.gov/nchs/data/nvsr/nvsr60/nvsr60_06.pdf. Accessed October 10, 2012.

4. National Institute on Drug Abuse. Topics in brief: Prescription drug abuse. Available at: http://www.drugabuse.gov/publications/topics-in-brief/prescription-drug-abuse. Accessed October 11, 2012.

5. Morin CM, Bastien C, Guay B, et al. Randomized clinical trail of supervised tapering and cognitive behavior therapy to facilitate benzodiazepine discontinuation in older adults with chronic insomnia. Am J Psychiatry. 2004;161:332-342.

6. Oude Voshaar RC, Couvee JE, van Balkorn AJ, et al. Strategies for discontinuing long-term benzodiazepine use-meta-analysis. Br J Psychiatr. 2006;189:213-220.

7. Bashir K, King M, Ashworth M. Controlled evaluation of brief intervention by general practitioners to reduce chronic use of benzodiazepines. Br J Gen Pract. 1994;44:408-412.

8. Cormack MA, Sweeney KG, Hughes-Jones H, et al. Evaluation of an easy, cost-effective strategy to cut benzodiazepine use in general practice. Br J Gen Pract. 1994;44:5-8

9. Heather NA, Bowie A, Ashton H, et al. Randomized controlled trial of two brief interventions against long-term benzodiazepine use: outcome of intervention. Addict Res Theory. 2004;12:141-145.

References

1. Mugunthan K, McGuire T, Glasziou P. Minimal interventions to decrease long-term use of benzodiazepines in primary care: a systematic review and meta-analysis. Br J Gen Pract. 2011;61:e573-e578.

2. Tyrer PJ. Benzodiazepines on trial. Br Med J. 1984;288:1101-1102.

3. Centers for Disease Control and Prevention. Deaths: Leading causes for 2008. June 6, 2012. Available at: http://www.cdc.gov/nchs/data/nvsr/nvsr60/nvsr60_06.pdf. Accessed October 10, 2012.

4. National Institute on Drug Abuse. Topics in brief: Prescription drug abuse. Available at: http://www.drugabuse.gov/publications/topics-in-brief/prescription-drug-abuse. Accessed October 11, 2012.

5. Morin CM, Bastien C, Guay B, et al. Randomized clinical trail of supervised tapering and cognitive behavior therapy to facilitate benzodiazepine discontinuation in older adults with chronic insomnia. Am J Psychiatry. 2004;161:332-342.

6. Oude Voshaar RC, Couvee JE, van Balkorn AJ, et al. Strategies for discontinuing long-term benzodiazepine use-meta-analysis. Br J Psychiatr. 2006;189:213-220.

7. Bashir K, King M, Ashworth M. Controlled evaluation of brief intervention by general practitioners to reduce chronic use of benzodiazepines. Br J Gen Pract. 1994;44:408-412.

8. Cormack MA, Sweeney KG, Hughes-Jones H, et al. Evaluation of an easy, cost-effective strategy to cut benzodiazepine use in general practice. Br J Gen Pract. 1994;44:5-8

9. Heather NA, Bowie A, Ashton H, et al. Randomized controlled trial of two brief interventions against long-term benzodiazepine use: outcome of intervention. Addict Res Theory. 2004;12:141-145.

Issue
The Journal of Family Practice - 61(11)
Issue
The Journal of Family Practice - 61(11)
Page Number
671-672
Page Number
671-672
Publications
Publications
Topics
Article Type
Display Headline
Patient overusing antianxiety meds? Say so (in a letter)
Display Headline
Patient overusing antianxiety meds? Say so (in a letter)
Legacy Keywords
Robert Levy;MD; Shailendra Prasad;MBBS;MPH; benzodiazepines; minimal interventions; inclusion criteria; primary care patients; short consultation; letter; withdrawal rates
Legacy Keywords
Robert Levy;MD; Shailendra Prasad;MBBS;MPH; benzodiazepines; minimal interventions; inclusion criteria; primary care patients; short consultation; letter; withdrawal rates
Sections
Article Source

PURLs Copyright

Copyright © 2012 The Family Physicians Inquiries Network. All rights reserved.

Inside the Article

Article PDF Media

Rethinking antibiotics for sinusitis—again

Article Type
Changed
Thu, 07/16/2020 - 13:45
Display Headline
Rethinking antibiotics for sinusitis—again
PRACTICE CHANGER

Stop prescribing amoxicillin for acute rhinosinusitis. It’s unlikely to provide a speedier recovery than over-the-counter (OTC) remedies alone.1

STRENGTH OF RECOMMENDATION

B: Based on a single high-quality randomized controlled trial.

Garbutt J, Banister C, Spitznagel E, et al. Amoxicillin for acute rhinosinusitis: a randomized controlled trial. JAMA. 2012 ;307:685-692.

 

ILLUSTRATIVE CASE

A 28-year-old man comes to your clinic after suffering from fatigue, purulent nasal discharge, and unilateral facial pain for nearly 10 days. Overall, he appears healthy, and you diagnose acute rhinosinusitis. You suggest OTC remedies for supportive care and wonder if a course of amoxicillin would speed his recovery.

Each year, more than 30 million Americans—about one in 7 adults—are diagnosed with sinusitis.2 No more than 2% of these cases are thought to be bacterial.3

Centers for Disease Control and Prevention (CDC) guidelines for the diagnosis of acute bacterial rhinosinusitis include symptoms that last 7 days or more, with maxillary pain or tenderness in the face or teeth and purulent nasal secretions.4 Patients with symptoms lasting less than 7 days are unlikely to have a bacterial infection. But the non-specific signs and symptoms included in the CDC guidelines limit their usefulness in determining whether the cause of the sinusitis is bacterial or viral on clinical grounds alone.

Most cases of sinusitis spontaneously resolve
In patients with acute bacterial sinusitis, the American Academy of Otolaryngology-Head and Neck Surgery (AAO-HNS) guidelines advocate watchful waiting and symptom relief with nasal oxymetazoline, pseudoephedrine, and saline nasal irrigation.3 The rate of spontaneous resolution is high: 80% of patients with clinically diagnosed sinusitis improve without treatment within 2 weeks.1,5

Traditional decongestants and mucolytics have not demonstrated efficacy in resolving sinusitis, although rigorous evaluation is lacking. Other treatments, such as saline irrigation and intranasal corticosteroids, are of unclear benefit and need further study.6-8

Lack of evidence has done little to curtail antibiotic use
An earlier PURL based on a meta-analysis of antibiotic treatment trials for sinusitis recommended that we stop prescribing antibiotics for adults with acute sinusitis unless their symptoms are severe.9,10 Yet antibiotics remain the mainstay of treatment.

Despite the AAO-HNS guidelines, evidence of spontaneous resolution, and accumulating data on the lack of efficacy of antimicrobials for sinusitis, 81% of patients diagnosed with acute sinusitis were given prescriptions for antibiotics, a study of primary care practices showed.11 Frequent use of antibiotics contributes to high rates of drug resistance, and adverse events related to antibiotic use account for an estimated 142,500 emergency department visits annually.12

STUDY SUMMARY: Little benefit from amoxicillin, even for severe cases

Garbutt and colleagues revisited the issue, randomizing 166 patients from 10 primary care practices to amoxicillin plus symptomatic treatment or placebo plus symptomatic treatment for acute rhinosinusitis.1 To be eligible for the study, patients had to be between the ages of 18 and 70 years, meet CDC diagnostic criteria for acute rhinosinusitis, and have moderate to very severe symptoms that were of 7- to-28-day duration and worsening or not improving or of <7-day duration but had worsened after an initial improvement. Exclusion criteria included complications from sinusitis, a history of allergy to penicillin or amoxicillin, antibiotic use in the past 4 weeks, comorbidities that impair immune function, cystic fibrosis, pregnancy, and mild symptoms.

Both groups had similar baseline characteristics, with participants who were predominantly white (79%) and female (64%). All the participants received a supply of symptomatic treatments: acetaminophen, guaifenesin, dextromethorphan, and sustained-release pseudoephedrine. The treatment group also received amoxicillin 1500 mg/d, divided into 3 doses; the placebo group received identical-looking placebo pills.

Patients were assessed with the Sino-nasal Outcome Test-16 (SNOT-16), a validated measure that asks patients to assess both the severity and frequency of 16 sinus symptoms. SNOT-16 uses a 0-to-3 rating scale (0=no problem; 3=severe problem), with a clinically important difference of ≥0.5 on the mean score. The test was administered at enrollment and at Days 3, 7, and 10. The disease-specific quality of life at Day 3 was the primary outcome.

 

 

 

There was no statistically significantly difference in SNOT-16 scores between the amoxicillin and placebo groups on Days 3 and 10. On Day 7, there was a small statistically significant improvement in the amoxicillin group, but it did not reach the level of clinical importance (≥0.5) based on SNOT-16’s mean score.

The authors also asked participants to retrospectively assess symptom change since enrollment on a 6-point scale. Those who reported that their symptoms were “a lot better” or “absent” were characterized as significantly improved. The results correlated with the data from the SNOT-16, showing no difference between the amoxicillin and control group at Days 3 and 10. On Day 7, 74% of patients treated with amoxicillin self- reported significant improvement in symptoms since the start of the study, vs 56% in the control group. The number needed to treat was 6 (95% confidence interval, 3-34; P= .02) for a reduction in symptoms at Day 7.

Patients in both groups had similar rates of absenteeism, inability to perform usual activities, relapse and recurrence, and use of additional health care. Satisfaction with treatment was similar, as well.

No serious adverse effects occurred. Both groups reported similar frequencies (<10%) of nausea, diarrhea, abdominal pain, or vaginitis.

WHAT’S NEW: Even severe sinusitis resolves without antibiotics

Previous studies recommended foregoing antibiotics for acute sinusitis, except when symptoms are severe. This study—in which more than half (52%) of patients in each group had symptoms rated severe or very severe—found no benefit to adding amoxicillin to supportive treatments.1 Antibiotics did not shorten the duration of illness, prevent relapse and recurrence, or improve satisfaction with treatment. The researchers found a statistically significant difference between groups on Day 7 of 0.19 points, but no clinically meaningful difference (≥0.5) based on the SNOT-16 mean score.

CAVEATS: Guidelines, risk of complications may give reason to pause

The 2012 Infectious Diseases Society of America guidelines recommend amoxicillin with clavulanic acid as empiric therapy for acute bacterial rhinosinusitis.7 The findings of the study by Garbutt et al—conducted at a time when the incidence of beta-lactamase-producing organisms was low and amoxicillin was the treatment of choice—suggest otherwise.

Serious complications of sinusitis, such as brain abscess, periorbital cellulitis, and meningitis, can occur, however. Patients who deteriorate clinically or develop high fever or severe headache require close follow-up, which may include further diagnostic evaluation or consultation with an otolaryngologist. Evidence is lacking as to whether antibiotics prevent such complications.5

CHALLENGES TO IMPLEMENTATION: Managing patient expectations

Many patients with symptoms of acute rhinosinusitis think they need an antibiotic. Managing their expectations and providing instructions about supportive treatments are time consuming and may be difficult.

Nonetheless, we’re optimistic: We think that most patients today are aware of the problems associated with antibiotic resistance and wary of “superbugs,” and will therefore be receptive to this practice change. Physicians can help by reminding patients of the adverse effects of antibiotics and the natural course of rhino-sinusitis, as well as by offering symptomatic treatments.

Acknowledgement

The PURLs Surveillance System is supported in part by Grant Number UL1RR024999 from the National Center for Research Resources, a Clinical Translational Science Award to the University of Chicago. The content is solely the responsibility of the authors and does not necessarily represent the official views of the National Center for Research Resources or the National Institutes of Health.

Files
References

1. Garbutt J, Banister C, Spitznagel E, et al. Amoxicillin for acute rhinosinusitis: a randomized controlled trial. JAMA. 2012;307:685-692.

2. Centers for Disease Control and Prevention. Summary health statistics for US adults: National Health Interview Survey 2010. January 2012. Available at: http://www.cdc.gov/nchs/data/series/sr_10/sr10_252.pdf. Accessed July 9, 2012.

3. Rosenfeld RM, Andes D, Bhattacharyya N, et al. Clinical practice guideline: adult sinusitis. Otolaryngol Head Neck Surg. 2007;137(3 suppl):S1-S31.

4. Hickner JM, Bartlett JG, Besser RE, et al. American Academy of Family Physians; American College of Physicians; American Society of Internal Medicine; Centers for Disease Control; Infectious Disease Society of America. Principles of appropriate antibiotic use for acute rhinosinusitis in adults; background. Ann Intern Med. 2001;134:498-505.

5. Ahovuo-Saloranta A, Borisenk OV, Kovanen N, et al. Antibiotics for acute maxillary sinusitis. Cochrane Database Syst Rev. 2008;(2):CD000243.-

6. Allen G, Kelsberg G, Jankowski TA. Do nasal decongestants relieve symptoms? J Fam Pract. 2003;52:714-724.

7. Chow AW, Benninger MS, Brook I, et al. IDSA clinical practice guideline for acute bacterial rhinosinusitis in children and adults. Clin Infect Dis. 2012;54:e72-e112.

8. Zalmanovici A, Yaphe J. Intranasal steroids for acute sinusitis. Cochrane Database Syst Rev. 2009;(4):CD005149.-

9. Schumann A, Hickner J. Patients insist on antibiotics for sinusitis? Here is a good reason to say “no”. J Fam Pract. Jul:2008;57:464-468.

10. Young J, De Sutter A, Merenstein D, et al. Antibiotics for adults with clinically diagnosed acute rhinosinusitis: a meta-analysis of individual patient data. Lancet. 2008;371:908-914.

11. Gill JM, Fleischut P, Haas S. Use of antibiotics for adult upper respiratory infections in outpatient settings: a national ambulatory network study. Fam Med. 2006;38:349-354.

12. Shehab N, Patel PR, Srinivasan A, et al. Emergency department visits for antibiotic-associated adverse events. Clin Infect Dis. 2008;47:735-743.

Article PDF
Author and Disclosure Information

Christopher Boisselle, MD
UIC/Advocate Illinois Masonic

Kate Rowland, MD, MS
UIC/Advocate Illinois Masonic, University of Chicago

PURLs EDITOR
Anne Mounsey, MD
University of North Carolina, Chapel Hill

Issue
The Journal of Family Practice - 61(10)
Publications
Topics
Page Number
610-612
Legacy Keywords
Christopher Boisselle;MD; Kate Rowland;MD; acute rhinosinusitis; amoxicillin; OTC remedies; spontaneously resolve; decongestants; patient expectations; mucolytics; complications
Sections
Files
Files
Author and Disclosure Information

Christopher Boisselle, MD
UIC/Advocate Illinois Masonic

Kate Rowland, MD, MS
UIC/Advocate Illinois Masonic, University of Chicago

PURLs EDITOR
Anne Mounsey, MD
University of North Carolina, Chapel Hill

Author and Disclosure Information

Christopher Boisselle, MD
UIC/Advocate Illinois Masonic

Kate Rowland, MD, MS
UIC/Advocate Illinois Masonic, University of Chicago

PURLs EDITOR
Anne Mounsey, MD
University of North Carolina, Chapel Hill

Article PDF
Article PDF
PRACTICE CHANGER

Stop prescribing amoxicillin for acute rhinosinusitis. It’s unlikely to provide a speedier recovery than over-the-counter (OTC) remedies alone.1

STRENGTH OF RECOMMENDATION

B: Based on a single high-quality randomized controlled trial.

Garbutt J, Banister C, Spitznagel E, et al. Amoxicillin for acute rhinosinusitis: a randomized controlled trial. JAMA. 2012 ;307:685-692.

 

ILLUSTRATIVE CASE

A 28-year-old man comes to your clinic after suffering from fatigue, purulent nasal discharge, and unilateral facial pain for nearly 10 days. Overall, he appears healthy, and you diagnose acute rhinosinusitis. You suggest OTC remedies for supportive care and wonder if a course of amoxicillin would speed his recovery.

Each year, more than 30 million Americans—about one in 7 adults—are diagnosed with sinusitis.2 No more than 2% of these cases are thought to be bacterial.3

Centers for Disease Control and Prevention (CDC) guidelines for the diagnosis of acute bacterial rhinosinusitis include symptoms that last 7 days or more, with maxillary pain or tenderness in the face or teeth and purulent nasal secretions.4 Patients with symptoms lasting less than 7 days are unlikely to have a bacterial infection. But the non-specific signs and symptoms included in the CDC guidelines limit their usefulness in determining whether the cause of the sinusitis is bacterial or viral on clinical grounds alone.

Most cases of sinusitis spontaneously resolve
In patients with acute bacterial sinusitis, the American Academy of Otolaryngology-Head and Neck Surgery (AAO-HNS) guidelines advocate watchful waiting and symptom relief with nasal oxymetazoline, pseudoephedrine, and saline nasal irrigation.3 The rate of spontaneous resolution is high: 80% of patients with clinically diagnosed sinusitis improve without treatment within 2 weeks.1,5

Traditional decongestants and mucolytics have not demonstrated efficacy in resolving sinusitis, although rigorous evaluation is lacking. Other treatments, such as saline irrigation and intranasal corticosteroids, are of unclear benefit and need further study.6-8

Lack of evidence has done little to curtail antibiotic use
An earlier PURL based on a meta-analysis of antibiotic treatment trials for sinusitis recommended that we stop prescribing antibiotics for adults with acute sinusitis unless their symptoms are severe.9,10 Yet antibiotics remain the mainstay of treatment.

Despite the AAO-HNS guidelines, evidence of spontaneous resolution, and accumulating data on the lack of efficacy of antimicrobials for sinusitis, 81% of patients diagnosed with acute sinusitis were given prescriptions for antibiotics, a study of primary care practices showed.11 Frequent use of antibiotics contributes to high rates of drug resistance, and adverse events related to antibiotic use account for an estimated 142,500 emergency department visits annually.12

STUDY SUMMARY: Little benefit from amoxicillin, even for severe cases

Garbutt and colleagues revisited the issue, randomizing 166 patients from 10 primary care practices to amoxicillin plus symptomatic treatment or placebo plus symptomatic treatment for acute rhinosinusitis.1 To be eligible for the study, patients had to be between the ages of 18 and 70 years, meet CDC diagnostic criteria for acute rhinosinusitis, and have moderate to very severe symptoms that were of 7- to-28-day duration and worsening or not improving or of <7-day duration but had worsened after an initial improvement. Exclusion criteria included complications from sinusitis, a history of allergy to penicillin or amoxicillin, antibiotic use in the past 4 weeks, comorbidities that impair immune function, cystic fibrosis, pregnancy, and mild symptoms.

Both groups had similar baseline characteristics, with participants who were predominantly white (79%) and female (64%). All the participants received a supply of symptomatic treatments: acetaminophen, guaifenesin, dextromethorphan, and sustained-release pseudoephedrine. The treatment group also received amoxicillin 1500 mg/d, divided into 3 doses; the placebo group received identical-looking placebo pills.

Patients were assessed with the Sino-nasal Outcome Test-16 (SNOT-16), a validated measure that asks patients to assess both the severity and frequency of 16 sinus symptoms. SNOT-16 uses a 0-to-3 rating scale (0=no problem; 3=severe problem), with a clinically important difference of ≥0.5 on the mean score. The test was administered at enrollment and at Days 3, 7, and 10. The disease-specific quality of life at Day 3 was the primary outcome.

 

 

 

There was no statistically significantly difference in SNOT-16 scores between the amoxicillin and placebo groups on Days 3 and 10. On Day 7, there was a small statistically significant improvement in the amoxicillin group, but it did not reach the level of clinical importance (≥0.5) based on SNOT-16’s mean score.

The authors also asked participants to retrospectively assess symptom change since enrollment on a 6-point scale. Those who reported that their symptoms were “a lot better” or “absent” were characterized as significantly improved. The results correlated with the data from the SNOT-16, showing no difference between the amoxicillin and control group at Days 3 and 10. On Day 7, 74% of patients treated with amoxicillin self- reported significant improvement in symptoms since the start of the study, vs 56% in the control group. The number needed to treat was 6 (95% confidence interval, 3-34; P= .02) for a reduction in symptoms at Day 7.

Patients in both groups had similar rates of absenteeism, inability to perform usual activities, relapse and recurrence, and use of additional health care. Satisfaction with treatment was similar, as well.

No serious adverse effects occurred. Both groups reported similar frequencies (<10%) of nausea, diarrhea, abdominal pain, or vaginitis.

WHAT’S NEW: Even severe sinusitis resolves without antibiotics

Previous studies recommended foregoing antibiotics for acute sinusitis, except when symptoms are severe. This study—in which more than half (52%) of patients in each group had symptoms rated severe or very severe—found no benefit to adding amoxicillin to supportive treatments.1 Antibiotics did not shorten the duration of illness, prevent relapse and recurrence, or improve satisfaction with treatment. The researchers found a statistically significant difference between groups on Day 7 of 0.19 points, but no clinically meaningful difference (≥0.5) based on the SNOT-16 mean score.

CAVEATS: Guidelines, risk of complications may give reason to pause

The 2012 Infectious Diseases Society of America guidelines recommend amoxicillin with clavulanic acid as empiric therapy for acute bacterial rhinosinusitis.7 The findings of the study by Garbutt et al—conducted at a time when the incidence of beta-lactamase-producing organisms was low and amoxicillin was the treatment of choice—suggest otherwise.

Serious complications of sinusitis, such as brain abscess, periorbital cellulitis, and meningitis, can occur, however. Patients who deteriorate clinically or develop high fever or severe headache require close follow-up, which may include further diagnostic evaluation or consultation with an otolaryngologist. Evidence is lacking as to whether antibiotics prevent such complications.5

CHALLENGES TO IMPLEMENTATION: Managing patient expectations

Many patients with symptoms of acute rhinosinusitis think they need an antibiotic. Managing their expectations and providing instructions about supportive treatments are time consuming and may be difficult.

Nonetheless, we’re optimistic: We think that most patients today are aware of the problems associated with antibiotic resistance and wary of “superbugs,” and will therefore be receptive to this practice change. Physicians can help by reminding patients of the adverse effects of antibiotics and the natural course of rhino-sinusitis, as well as by offering symptomatic treatments.

Acknowledgement

The PURLs Surveillance System is supported in part by Grant Number UL1RR024999 from the National Center for Research Resources, a Clinical Translational Science Award to the University of Chicago. The content is solely the responsibility of the authors and does not necessarily represent the official views of the National Center for Research Resources or the National Institutes of Health.

PRACTICE CHANGER

Stop prescribing amoxicillin for acute rhinosinusitis. It’s unlikely to provide a speedier recovery than over-the-counter (OTC) remedies alone.1

STRENGTH OF RECOMMENDATION

B: Based on a single high-quality randomized controlled trial.

Garbutt J, Banister C, Spitznagel E, et al. Amoxicillin for acute rhinosinusitis: a randomized controlled trial. JAMA. 2012 ;307:685-692.

 

ILLUSTRATIVE CASE

A 28-year-old man comes to your clinic after suffering from fatigue, purulent nasal discharge, and unilateral facial pain for nearly 10 days. Overall, he appears healthy, and you diagnose acute rhinosinusitis. You suggest OTC remedies for supportive care and wonder if a course of amoxicillin would speed his recovery.

Each year, more than 30 million Americans—about one in 7 adults—are diagnosed with sinusitis.2 No more than 2% of these cases are thought to be bacterial.3

Centers for Disease Control and Prevention (CDC) guidelines for the diagnosis of acute bacterial rhinosinusitis include symptoms that last 7 days or more, with maxillary pain or tenderness in the face or teeth and purulent nasal secretions.4 Patients with symptoms lasting less than 7 days are unlikely to have a bacterial infection. But the non-specific signs and symptoms included in the CDC guidelines limit their usefulness in determining whether the cause of the sinusitis is bacterial or viral on clinical grounds alone.

Most cases of sinusitis spontaneously resolve
In patients with acute bacterial sinusitis, the American Academy of Otolaryngology-Head and Neck Surgery (AAO-HNS) guidelines advocate watchful waiting and symptom relief with nasal oxymetazoline, pseudoephedrine, and saline nasal irrigation.3 The rate of spontaneous resolution is high: 80% of patients with clinically diagnosed sinusitis improve without treatment within 2 weeks.1,5

Traditional decongestants and mucolytics have not demonstrated efficacy in resolving sinusitis, although rigorous evaluation is lacking. Other treatments, such as saline irrigation and intranasal corticosteroids, are of unclear benefit and need further study.6-8

Lack of evidence has done little to curtail antibiotic use
An earlier PURL based on a meta-analysis of antibiotic treatment trials for sinusitis recommended that we stop prescribing antibiotics for adults with acute sinusitis unless their symptoms are severe.9,10 Yet antibiotics remain the mainstay of treatment.

Despite the AAO-HNS guidelines, evidence of spontaneous resolution, and accumulating data on the lack of efficacy of antimicrobials for sinusitis, 81% of patients diagnosed with acute sinusitis were given prescriptions for antibiotics, a study of primary care practices showed.11 Frequent use of antibiotics contributes to high rates of drug resistance, and adverse events related to antibiotic use account for an estimated 142,500 emergency department visits annually.12

STUDY SUMMARY: Little benefit from amoxicillin, even for severe cases

Garbutt and colleagues revisited the issue, randomizing 166 patients from 10 primary care practices to amoxicillin plus symptomatic treatment or placebo plus symptomatic treatment for acute rhinosinusitis.1 To be eligible for the study, patients had to be between the ages of 18 and 70 years, meet CDC diagnostic criteria for acute rhinosinusitis, and have moderate to very severe symptoms that were of 7- to-28-day duration and worsening or not improving or of <7-day duration but had worsened after an initial improvement. Exclusion criteria included complications from sinusitis, a history of allergy to penicillin or amoxicillin, antibiotic use in the past 4 weeks, comorbidities that impair immune function, cystic fibrosis, pregnancy, and mild symptoms.

Both groups had similar baseline characteristics, with participants who were predominantly white (79%) and female (64%). All the participants received a supply of symptomatic treatments: acetaminophen, guaifenesin, dextromethorphan, and sustained-release pseudoephedrine. The treatment group also received amoxicillin 1500 mg/d, divided into 3 doses; the placebo group received identical-looking placebo pills.

Patients were assessed with the Sino-nasal Outcome Test-16 (SNOT-16), a validated measure that asks patients to assess both the severity and frequency of 16 sinus symptoms. SNOT-16 uses a 0-to-3 rating scale (0=no problem; 3=severe problem), with a clinically important difference of ≥0.5 on the mean score. The test was administered at enrollment and at Days 3, 7, and 10. The disease-specific quality of life at Day 3 was the primary outcome.

 

 

 

There was no statistically significantly difference in SNOT-16 scores between the amoxicillin and placebo groups on Days 3 and 10. On Day 7, there was a small statistically significant improvement in the amoxicillin group, but it did not reach the level of clinical importance (≥0.5) based on SNOT-16’s mean score.

The authors also asked participants to retrospectively assess symptom change since enrollment on a 6-point scale. Those who reported that their symptoms were “a lot better” or “absent” were characterized as significantly improved. The results correlated with the data from the SNOT-16, showing no difference between the amoxicillin and control group at Days 3 and 10. On Day 7, 74% of patients treated with amoxicillin self- reported significant improvement in symptoms since the start of the study, vs 56% in the control group. The number needed to treat was 6 (95% confidence interval, 3-34; P= .02) for a reduction in symptoms at Day 7.

Patients in both groups had similar rates of absenteeism, inability to perform usual activities, relapse and recurrence, and use of additional health care. Satisfaction with treatment was similar, as well.

No serious adverse effects occurred. Both groups reported similar frequencies (<10%) of nausea, diarrhea, abdominal pain, or vaginitis.

WHAT’S NEW: Even severe sinusitis resolves without antibiotics

Previous studies recommended foregoing antibiotics for acute sinusitis, except when symptoms are severe. This study—in which more than half (52%) of patients in each group had symptoms rated severe or very severe—found no benefit to adding amoxicillin to supportive treatments.1 Antibiotics did not shorten the duration of illness, prevent relapse and recurrence, or improve satisfaction with treatment. The researchers found a statistically significant difference between groups on Day 7 of 0.19 points, but no clinically meaningful difference (≥0.5) based on the SNOT-16 mean score.

CAVEATS: Guidelines, risk of complications may give reason to pause

The 2012 Infectious Diseases Society of America guidelines recommend amoxicillin with clavulanic acid as empiric therapy for acute bacterial rhinosinusitis.7 The findings of the study by Garbutt et al—conducted at a time when the incidence of beta-lactamase-producing organisms was low and amoxicillin was the treatment of choice—suggest otherwise.

Serious complications of sinusitis, such as brain abscess, periorbital cellulitis, and meningitis, can occur, however. Patients who deteriorate clinically or develop high fever or severe headache require close follow-up, which may include further diagnostic evaluation or consultation with an otolaryngologist. Evidence is lacking as to whether antibiotics prevent such complications.5

CHALLENGES TO IMPLEMENTATION: Managing patient expectations

Many patients with symptoms of acute rhinosinusitis think they need an antibiotic. Managing their expectations and providing instructions about supportive treatments are time consuming and may be difficult.

Nonetheless, we’re optimistic: We think that most patients today are aware of the problems associated with antibiotic resistance and wary of “superbugs,” and will therefore be receptive to this practice change. Physicians can help by reminding patients of the adverse effects of antibiotics and the natural course of rhino-sinusitis, as well as by offering symptomatic treatments.

Acknowledgement

The PURLs Surveillance System is supported in part by Grant Number UL1RR024999 from the National Center for Research Resources, a Clinical Translational Science Award to the University of Chicago. The content is solely the responsibility of the authors and does not necessarily represent the official views of the National Center for Research Resources or the National Institutes of Health.

References

1. Garbutt J, Banister C, Spitznagel E, et al. Amoxicillin for acute rhinosinusitis: a randomized controlled trial. JAMA. 2012;307:685-692.

2. Centers for Disease Control and Prevention. Summary health statistics for US adults: National Health Interview Survey 2010. January 2012. Available at: http://www.cdc.gov/nchs/data/series/sr_10/sr10_252.pdf. Accessed July 9, 2012.

3. Rosenfeld RM, Andes D, Bhattacharyya N, et al. Clinical practice guideline: adult sinusitis. Otolaryngol Head Neck Surg. 2007;137(3 suppl):S1-S31.

4. Hickner JM, Bartlett JG, Besser RE, et al. American Academy of Family Physians; American College of Physicians; American Society of Internal Medicine; Centers for Disease Control; Infectious Disease Society of America. Principles of appropriate antibiotic use for acute rhinosinusitis in adults; background. Ann Intern Med. 2001;134:498-505.

5. Ahovuo-Saloranta A, Borisenk OV, Kovanen N, et al. Antibiotics for acute maxillary sinusitis. Cochrane Database Syst Rev. 2008;(2):CD000243.-

6. Allen G, Kelsberg G, Jankowski TA. Do nasal decongestants relieve symptoms? J Fam Pract. 2003;52:714-724.

7. Chow AW, Benninger MS, Brook I, et al. IDSA clinical practice guideline for acute bacterial rhinosinusitis in children and adults. Clin Infect Dis. 2012;54:e72-e112.

8. Zalmanovici A, Yaphe J. Intranasal steroids for acute sinusitis. Cochrane Database Syst Rev. 2009;(4):CD005149.-

9. Schumann A, Hickner J. Patients insist on antibiotics for sinusitis? Here is a good reason to say “no”. J Fam Pract. Jul:2008;57:464-468.

10. Young J, De Sutter A, Merenstein D, et al. Antibiotics for adults with clinically diagnosed acute rhinosinusitis: a meta-analysis of individual patient data. Lancet. 2008;371:908-914.

11. Gill JM, Fleischut P, Haas S. Use of antibiotics for adult upper respiratory infections in outpatient settings: a national ambulatory network study. Fam Med. 2006;38:349-354.

12. Shehab N, Patel PR, Srinivasan A, et al. Emergency department visits for antibiotic-associated adverse events. Clin Infect Dis. 2008;47:735-743.

References

1. Garbutt J, Banister C, Spitznagel E, et al. Amoxicillin for acute rhinosinusitis: a randomized controlled trial. JAMA. 2012;307:685-692.

2. Centers for Disease Control and Prevention. Summary health statistics for US adults: National Health Interview Survey 2010. January 2012. Available at: http://www.cdc.gov/nchs/data/series/sr_10/sr10_252.pdf. Accessed July 9, 2012.

3. Rosenfeld RM, Andes D, Bhattacharyya N, et al. Clinical practice guideline: adult sinusitis. Otolaryngol Head Neck Surg. 2007;137(3 suppl):S1-S31.

4. Hickner JM, Bartlett JG, Besser RE, et al. American Academy of Family Physians; American College of Physicians; American Society of Internal Medicine; Centers for Disease Control; Infectious Disease Society of America. Principles of appropriate antibiotic use for acute rhinosinusitis in adults; background. Ann Intern Med. 2001;134:498-505.

5. Ahovuo-Saloranta A, Borisenk OV, Kovanen N, et al. Antibiotics for acute maxillary sinusitis. Cochrane Database Syst Rev. 2008;(2):CD000243.-

6. Allen G, Kelsberg G, Jankowski TA. Do nasal decongestants relieve symptoms? J Fam Pract. 2003;52:714-724.

7. Chow AW, Benninger MS, Brook I, et al. IDSA clinical practice guideline for acute bacterial rhinosinusitis in children and adults. Clin Infect Dis. 2012;54:e72-e112.

8. Zalmanovici A, Yaphe J. Intranasal steroids for acute sinusitis. Cochrane Database Syst Rev. 2009;(4):CD005149.-

9. Schumann A, Hickner J. Patients insist on antibiotics for sinusitis? Here is a good reason to say “no”. J Fam Pract. Jul:2008;57:464-468.

10. Young J, De Sutter A, Merenstein D, et al. Antibiotics for adults with clinically diagnosed acute rhinosinusitis: a meta-analysis of individual patient data. Lancet. 2008;371:908-914.

11. Gill JM, Fleischut P, Haas S. Use of antibiotics for adult upper respiratory infections in outpatient settings: a national ambulatory network study. Fam Med. 2006;38:349-354.

12. Shehab N, Patel PR, Srinivasan A, et al. Emergency department visits for antibiotic-associated adverse events. Clin Infect Dis. 2008;47:735-743.

Issue
The Journal of Family Practice - 61(10)
Issue
The Journal of Family Practice - 61(10)
Page Number
610-612
Page Number
610-612
Publications
Publications
Topics
Article Type
Display Headline
Rethinking antibiotics for sinusitis—again
Display Headline
Rethinking antibiotics for sinusitis—again
Legacy Keywords
Christopher Boisselle;MD; Kate Rowland;MD; acute rhinosinusitis; amoxicillin; OTC remedies; spontaneously resolve; decongestants; patient expectations; mucolytics; complications
Legacy Keywords
Christopher Boisselle;MD; Kate Rowland;MD; acute rhinosinusitis; amoxicillin; OTC remedies; spontaneously resolve; decongestants; patient expectations; mucolytics; complications
Sections
PURLs Copyright

Copyright © 2012 The Family Physicians Inquiries Network. All rights reserved.

Disallow All Ads
Alternative CME
Use ProPublica
Hide sidebar & use full width
render the right sidebar.
Conference Recap Checkbox
Not Conference Recap
Clinical Edge
Display the Slideshow in this Article
Article PDF Media
Media Files

Easing the discomfort of a speculum exam

Article Type
Changed
Thu, 07/16/2020 - 13:32
Display Headline
Easing the discomfort of a speculum exam

 

PRACTICE CHANGER

Put lubricating gel, not water, on the speculum every time you do a pelvic exam.1

STRENGTH OF RECOMMENDATION

B: Based on one good-quality, randomized controlled trial (RCT).

Hill DA, Lamvu G. Effect of lubricating gel on patient comfort during vaginal speculum examination. Obstet Gynecol. 2012;119(2 pt 1): 227-231.

ILLUSTRATIVE CASE

A 24-year-old woman comes in for an annual exam, including Pap smear and testing for sexually transmitted diseases (STD). She tells you how painful her previous speculum exam was and how worried she is about having another. Should you apply lubricating gel or water to the speculum before vaginal insertion to ease her discomfort?

Physiology teaches us that vaginal entry requires lubrication. But traditional teaching has held that lubricating gel on a speculum can interfere with the results of a Pap smear and chlamydia tesing.2 Pelvic exams performed without lubricating gel on the speculum can cause significant discomfort—possibly bad enough to prevent some women from undergoing the recommended screening tests.3

Until now, we’ve only evaluated gel’s impact on test results
Studies comparing lubricating gel and water have conclusively shown that a small amount of gel, used on the outside of the speculum blades, does not interfere with either Pap testing or detection of Chlamydia trachomatis.4,5 One liquid-based cytology manufacturer, however, discourages the use of lubricants with “carbomers” or “carbopol polymers,” but states that water-based lubricants have not been shown to interfere with Pap smear results.6 No studies have evaluated lubricants from a patient perspective—until now.

STUDY SUMMARY: Lubricating gel eases discomfort

The study by Hill and Lamvu was a 6-month, single-blind, randomized trial of women ages 18 to 50 years who sought care at an Orlando, Florida obstetrics and gynecology department for conditions requiring vaginal speculum examination.1 The study excluded women who might have an altered perception of pain during speculum insertion— those who were menopausal, pregnant, or within 6 weeks’ postpartum; had dyspareunia, vaginitis, vulvar pain, or vulvar lesions; were undergoing a procedure; or had never had vaginal intercourse. Women who were not fluent in English were excluded, as well.

The study included 120 women who underwent computer-generated randomization into 2 groups with no marked differences in demographics. A single examiner did all the speculum exams, using a standard protocol with a medium-size Graves speculum. The examiner applied 0.3 mL water-based lubricating gel to the speculum before insertion for the women in one group, and used 3 mL water for the other.

Immediately after the speculum was inserted and opened—before the examiner attempted to visualize the cervix—patients were given a visual analog scale and told to indicate the level of pain with insertion, using a scale of 0 (no pain) to 10 (the worst pain imaginable). The gel group had lower pain scores for speculum insertion compared with the water group (1.41±1.55 vs 2.15±1.93; P<.01), a statistically significant difference of 0.74. Twenty of the 59 patients in the gel group (33.9%) rated their pain as 0, compared with 6 of 60 (10%) in the water group (P=.002). Although pain, rather than sampling quality, was the primary outcome of the study, the authors also reported that all of the women who underwent Pap screening (73) had adequate cytology.

WHAT’S NEW?: It’s time for lubrication to become standard practice

This trial is the first to study speculum lubrication from this patient-oriented outcome, and to show that women experience less pain when lubricating gel is applied to the speculum, rather than water. This knowledge, combined with previous studies showing that a small amount of water-based lubricating gel does not interfere with liquid-based cytology or chlamydia test results, should make the use of lubricating gel standard practice when performing speculum examinations.

CAVEATS: We see no downside

The exclusion criteria of this study were meant to eliminate women who had an altered pain perception that could skew study results. Yet those who met the exclusion criteria may also benefit from a pelvic exam with gel lubrication. We see no harm in trying a small amount of lubricant when examining them, as well.

In addition, the study did not compare various types and sizes of specula. However, we see no reason why the benefit of a gel lubricant would be limited to the type of speculum used by the examiner.

Studies in emergency departments that have used visual analog scales to measure interventions that decrease pain have used a 0.9 mean difference as “clinically meaningful.”7,8 By that criteria, the 0.74 difference observed in this study does not rise to the level of clinical meaningfulness. However, one in 3 patients in the gel group marked 0 on the pain scale, indicating that they had no pain, vs only one in 10 in the water group. We believe that the higher proportion of women experiencing no pain and the mean difference of 0.74 on the pain scale (both statistically significant), combined with the lack of risk associated with the use of a water-based lubricant, makes this a clinically useful practice changer.

 

 

 

CHALLENGES TO IMPLEMENTATION: There aren’t any

Other than clinical inertia, we see no challenges to the implementation of this recommendation.

 

Acknowledgement

The PURLs Surveillance System is supported in part by Grant Number UL1RR024999 from the National center for Research Resources, a clinical Translational Science Award to the University of chicago. The content is solely the responsibility of the authors and does not necessarily represent the official views of the National center for Research Resources or the National Institutes of health.

Files
References

 

1. Hill DA, Lamvu G. Effect of lubricating gel on patient comfort during vaginal speculum examination. Obstet Gynecol. 2012;119 (2 pt 1):227-231.

2. Harmanli O, Jones KA. Using lubricant for speculum insertion. Obstet Gynecol. 2010;116:415-417.

3. Hoyo C, Yarnall KSH, Skinner CS, et al. Pain predicts non-adherence to pap smear screening among middle-aged African American women. Prev Med. 2005;41:439-445.

4. Amies AE, Miller L, Lee S, et al. The effect of vaginal speculum lubrication on the rate of unsatisfactory cervical cytology diagnosis. Obstet Gynecol. 2002;100:889-892.

5. Griffith WF, Stuart GS, Gluck KL, et al. Vaginal speculum lubrication and its effects on cervical cytology and microbiology. Contraception. 2005;72:60-604.

6. Evantash E. Lubricant use during Pap test collection. Bedford, Mass: Hologic, Inc; 2009. Available at: www.thinprep.com/pdfs/Lubricant_Letter_Rev_3.doc. Accessed June 8, 2012.

7. Bijur PE, Silver W, Gallagher EJ. Reliability of the visual analog scale for measurement of acute pain. Acad Emerg Med. 2001;8:1153-1157.

8. Kelly AM. Does the clinically significant difference in visual analog scale pain scores vary with gender, age, or cause of pain? Acad Emerg Med. 1998;5:1086-1090.

Article PDF
Author and Disclosure Information

 

Nicole VandenBerg, MD
North Memorial Family Medicine Residency, Department of Family Medicine and Community Health, University of Minnesota, Minneapolis

Shailendra Prasad, MBBS, MPH
North Memorial Family Medicine Residency, Department of Family Medicine and Community Health, University of Minnesota, Minneapolis

PURLs EDITOR
Bernard Ewigman, MD, MSPH
The University of Chicago

Issue
The Journal of Family Practice - 61(9)
Publications
Topics
Page Number
E1-E3
Sections
Files
Files
Author and Disclosure Information

 

Nicole VandenBerg, MD
North Memorial Family Medicine Residency, Department of Family Medicine and Community Health, University of Minnesota, Minneapolis

Shailendra Prasad, MBBS, MPH
North Memorial Family Medicine Residency, Department of Family Medicine and Community Health, University of Minnesota, Minneapolis

PURLs EDITOR
Bernard Ewigman, MD, MSPH
The University of Chicago

Author and Disclosure Information

 

Nicole VandenBerg, MD
North Memorial Family Medicine Residency, Department of Family Medicine and Community Health, University of Minnesota, Minneapolis

Shailendra Prasad, MBBS, MPH
North Memorial Family Medicine Residency, Department of Family Medicine and Community Health, University of Minnesota, Minneapolis

PURLs EDITOR
Bernard Ewigman, MD, MSPH
The University of Chicago

Article PDF
Article PDF

 

PRACTICE CHANGER

Put lubricating gel, not water, on the speculum every time you do a pelvic exam.1

STRENGTH OF RECOMMENDATION

B: Based on one good-quality, randomized controlled trial (RCT).

Hill DA, Lamvu G. Effect of lubricating gel on patient comfort during vaginal speculum examination. Obstet Gynecol. 2012;119(2 pt 1): 227-231.

ILLUSTRATIVE CASE

A 24-year-old woman comes in for an annual exam, including Pap smear and testing for sexually transmitted diseases (STD). She tells you how painful her previous speculum exam was and how worried she is about having another. Should you apply lubricating gel or water to the speculum before vaginal insertion to ease her discomfort?

Physiology teaches us that vaginal entry requires lubrication. But traditional teaching has held that lubricating gel on a speculum can interfere with the results of a Pap smear and chlamydia tesing.2 Pelvic exams performed without lubricating gel on the speculum can cause significant discomfort—possibly bad enough to prevent some women from undergoing the recommended screening tests.3

Until now, we’ve only evaluated gel’s impact on test results
Studies comparing lubricating gel and water have conclusively shown that a small amount of gel, used on the outside of the speculum blades, does not interfere with either Pap testing or detection of Chlamydia trachomatis.4,5 One liquid-based cytology manufacturer, however, discourages the use of lubricants with “carbomers” or “carbopol polymers,” but states that water-based lubricants have not been shown to interfere with Pap smear results.6 No studies have evaluated lubricants from a patient perspective—until now.

STUDY SUMMARY: Lubricating gel eases discomfort

The study by Hill and Lamvu was a 6-month, single-blind, randomized trial of women ages 18 to 50 years who sought care at an Orlando, Florida obstetrics and gynecology department for conditions requiring vaginal speculum examination.1 The study excluded women who might have an altered perception of pain during speculum insertion— those who were menopausal, pregnant, or within 6 weeks’ postpartum; had dyspareunia, vaginitis, vulvar pain, or vulvar lesions; were undergoing a procedure; or had never had vaginal intercourse. Women who were not fluent in English were excluded, as well.

The study included 120 women who underwent computer-generated randomization into 2 groups with no marked differences in demographics. A single examiner did all the speculum exams, using a standard protocol with a medium-size Graves speculum. The examiner applied 0.3 mL water-based lubricating gel to the speculum before insertion for the women in one group, and used 3 mL water for the other.

Immediately after the speculum was inserted and opened—before the examiner attempted to visualize the cervix—patients were given a visual analog scale and told to indicate the level of pain with insertion, using a scale of 0 (no pain) to 10 (the worst pain imaginable). The gel group had lower pain scores for speculum insertion compared with the water group (1.41±1.55 vs 2.15±1.93; P<.01), a statistically significant difference of 0.74. Twenty of the 59 patients in the gel group (33.9%) rated their pain as 0, compared with 6 of 60 (10%) in the water group (P=.002). Although pain, rather than sampling quality, was the primary outcome of the study, the authors also reported that all of the women who underwent Pap screening (73) had adequate cytology.

WHAT’S NEW?: It’s time for lubrication to become standard practice

This trial is the first to study speculum lubrication from this patient-oriented outcome, and to show that women experience less pain when lubricating gel is applied to the speculum, rather than water. This knowledge, combined with previous studies showing that a small amount of water-based lubricating gel does not interfere with liquid-based cytology or chlamydia test results, should make the use of lubricating gel standard practice when performing speculum examinations.

CAVEATS: We see no downside

The exclusion criteria of this study were meant to eliminate women who had an altered pain perception that could skew study results. Yet those who met the exclusion criteria may also benefit from a pelvic exam with gel lubrication. We see no harm in trying a small amount of lubricant when examining them, as well.

In addition, the study did not compare various types and sizes of specula. However, we see no reason why the benefit of a gel lubricant would be limited to the type of speculum used by the examiner.

Studies in emergency departments that have used visual analog scales to measure interventions that decrease pain have used a 0.9 mean difference as “clinically meaningful.”7,8 By that criteria, the 0.74 difference observed in this study does not rise to the level of clinical meaningfulness. However, one in 3 patients in the gel group marked 0 on the pain scale, indicating that they had no pain, vs only one in 10 in the water group. We believe that the higher proportion of women experiencing no pain and the mean difference of 0.74 on the pain scale (both statistically significant), combined with the lack of risk associated with the use of a water-based lubricant, makes this a clinically useful practice changer.

 

 

 

CHALLENGES TO IMPLEMENTATION: There aren’t any

Other than clinical inertia, we see no challenges to the implementation of this recommendation.

 

Acknowledgement

The PURLs Surveillance System is supported in part by Grant Number UL1RR024999 from the National center for Research Resources, a clinical Translational Science Award to the University of chicago. The content is solely the responsibility of the authors and does not necessarily represent the official views of the National center for Research Resources or the National Institutes of health.

 

PRACTICE CHANGER

Put lubricating gel, not water, on the speculum every time you do a pelvic exam.1

STRENGTH OF RECOMMENDATION

B: Based on one good-quality, randomized controlled trial (RCT).

Hill DA, Lamvu G. Effect of lubricating gel on patient comfort during vaginal speculum examination. Obstet Gynecol. 2012;119(2 pt 1): 227-231.

ILLUSTRATIVE CASE

A 24-year-old woman comes in for an annual exam, including Pap smear and testing for sexually transmitted diseases (STD). She tells you how painful her previous speculum exam was and how worried she is about having another. Should you apply lubricating gel or water to the speculum before vaginal insertion to ease her discomfort?

Physiology teaches us that vaginal entry requires lubrication. But traditional teaching has held that lubricating gel on a speculum can interfere with the results of a Pap smear and chlamydia tesing.2 Pelvic exams performed without lubricating gel on the speculum can cause significant discomfort—possibly bad enough to prevent some women from undergoing the recommended screening tests.3

Until now, we’ve only evaluated gel’s impact on test results
Studies comparing lubricating gel and water have conclusively shown that a small amount of gel, used on the outside of the speculum blades, does not interfere with either Pap testing or detection of Chlamydia trachomatis.4,5 One liquid-based cytology manufacturer, however, discourages the use of lubricants with “carbomers” or “carbopol polymers,” but states that water-based lubricants have not been shown to interfere with Pap smear results.6 No studies have evaluated lubricants from a patient perspective—until now.

STUDY SUMMARY: Lubricating gel eases discomfort

The study by Hill and Lamvu was a 6-month, single-blind, randomized trial of women ages 18 to 50 years who sought care at an Orlando, Florida obstetrics and gynecology department for conditions requiring vaginal speculum examination.1 The study excluded women who might have an altered perception of pain during speculum insertion— those who were menopausal, pregnant, or within 6 weeks’ postpartum; had dyspareunia, vaginitis, vulvar pain, or vulvar lesions; were undergoing a procedure; or had never had vaginal intercourse. Women who were not fluent in English were excluded, as well.

The study included 120 women who underwent computer-generated randomization into 2 groups with no marked differences in demographics. A single examiner did all the speculum exams, using a standard protocol with a medium-size Graves speculum. The examiner applied 0.3 mL water-based lubricating gel to the speculum before insertion for the women in one group, and used 3 mL water for the other.

Immediately after the speculum was inserted and opened—before the examiner attempted to visualize the cervix—patients were given a visual analog scale and told to indicate the level of pain with insertion, using a scale of 0 (no pain) to 10 (the worst pain imaginable). The gel group had lower pain scores for speculum insertion compared with the water group (1.41±1.55 vs 2.15±1.93; P<.01), a statistically significant difference of 0.74. Twenty of the 59 patients in the gel group (33.9%) rated their pain as 0, compared with 6 of 60 (10%) in the water group (P=.002). Although pain, rather than sampling quality, was the primary outcome of the study, the authors also reported that all of the women who underwent Pap screening (73) had adequate cytology.

WHAT’S NEW?: It’s time for lubrication to become standard practice

This trial is the first to study speculum lubrication from this patient-oriented outcome, and to show that women experience less pain when lubricating gel is applied to the speculum, rather than water. This knowledge, combined with previous studies showing that a small amount of water-based lubricating gel does not interfere with liquid-based cytology or chlamydia test results, should make the use of lubricating gel standard practice when performing speculum examinations.

CAVEATS: We see no downside

The exclusion criteria of this study were meant to eliminate women who had an altered pain perception that could skew study results. Yet those who met the exclusion criteria may also benefit from a pelvic exam with gel lubrication. We see no harm in trying a small amount of lubricant when examining them, as well.

In addition, the study did not compare various types and sizes of specula. However, we see no reason why the benefit of a gel lubricant would be limited to the type of speculum used by the examiner.

Studies in emergency departments that have used visual analog scales to measure interventions that decrease pain have used a 0.9 mean difference as “clinically meaningful.”7,8 By that criteria, the 0.74 difference observed in this study does not rise to the level of clinical meaningfulness. However, one in 3 patients in the gel group marked 0 on the pain scale, indicating that they had no pain, vs only one in 10 in the water group. We believe that the higher proportion of women experiencing no pain and the mean difference of 0.74 on the pain scale (both statistically significant), combined with the lack of risk associated with the use of a water-based lubricant, makes this a clinically useful practice changer.

 

 

 

CHALLENGES TO IMPLEMENTATION: There aren’t any

Other than clinical inertia, we see no challenges to the implementation of this recommendation.

 

Acknowledgement

The PURLs Surveillance System is supported in part by Grant Number UL1RR024999 from the National center for Research Resources, a clinical Translational Science Award to the University of chicago. The content is solely the responsibility of the authors and does not necessarily represent the official views of the National center for Research Resources or the National Institutes of health.

References

 

1. Hill DA, Lamvu G. Effect of lubricating gel on patient comfort during vaginal speculum examination. Obstet Gynecol. 2012;119 (2 pt 1):227-231.

2. Harmanli O, Jones KA. Using lubricant for speculum insertion. Obstet Gynecol. 2010;116:415-417.

3. Hoyo C, Yarnall KSH, Skinner CS, et al. Pain predicts non-adherence to pap smear screening among middle-aged African American women. Prev Med. 2005;41:439-445.

4. Amies AE, Miller L, Lee S, et al. The effect of vaginal speculum lubrication on the rate of unsatisfactory cervical cytology diagnosis. Obstet Gynecol. 2002;100:889-892.

5. Griffith WF, Stuart GS, Gluck KL, et al. Vaginal speculum lubrication and its effects on cervical cytology and microbiology. Contraception. 2005;72:60-604.

6. Evantash E. Lubricant use during Pap test collection. Bedford, Mass: Hologic, Inc; 2009. Available at: www.thinprep.com/pdfs/Lubricant_Letter_Rev_3.doc. Accessed June 8, 2012.

7. Bijur PE, Silver W, Gallagher EJ. Reliability of the visual analog scale for measurement of acute pain. Acad Emerg Med. 2001;8:1153-1157.

8. Kelly AM. Does the clinically significant difference in visual analog scale pain scores vary with gender, age, or cause of pain? Acad Emerg Med. 1998;5:1086-1090.

References

 

1. Hill DA, Lamvu G. Effect of lubricating gel on patient comfort during vaginal speculum examination. Obstet Gynecol. 2012;119 (2 pt 1):227-231.

2. Harmanli O, Jones KA. Using lubricant for speculum insertion. Obstet Gynecol. 2010;116:415-417.

3. Hoyo C, Yarnall KSH, Skinner CS, et al. Pain predicts non-adherence to pap smear screening among middle-aged African American women. Prev Med. 2005;41:439-445.

4. Amies AE, Miller L, Lee S, et al. The effect of vaginal speculum lubrication on the rate of unsatisfactory cervical cytology diagnosis. Obstet Gynecol. 2002;100:889-892.

5. Griffith WF, Stuart GS, Gluck KL, et al. Vaginal speculum lubrication and its effects on cervical cytology and microbiology. Contraception. 2005;72:60-604.

6. Evantash E. Lubricant use during Pap test collection. Bedford, Mass: Hologic, Inc; 2009. Available at: www.thinprep.com/pdfs/Lubricant_Letter_Rev_3.doc. Accessed June 8, 2012.

7. Bijur PE, Silver W, Gallagher EJ. Reliability of the visual analog scale for measurement of acute pain. Acad Emerg Med. 2001;8:1153-1157.

8. Kelly AM. Does the clinically significant difference in visual analog scale pain scores vary with gender, age, or cause of pain? Acad Emerg Med. 1998;5:1086-1090.

Issue
The Journal of Family Practice - 61(9)
Issue
The Journal of Family Practice - 61(9)
Page Number
E1-E3
Page Number
E1-E3
Publications
Publications
Topics
Article Type
Display Headline
Easing the discomfort of a speculum exam
Display Headline
Easing the discomfort of a speculum exam
Sections
PURLs Copyright

Copyright © 2012 The Family Physicians Inquiries Network. All rights reserved.

Disallow All Ads
Alternative CME
Use ProPublica
Hide sidebar & use full width
render the right sidebar.
Conference Recap Checkbox
Not Conference Recap
Clinical Edge
Display the Slideshow in this Article
Article PDF Media
Media Files

DEXA screening—are we doing too much?

Article Type
Changed
Thu, 07/16/2020 - 13:36
Display Headline
DEXA screening—are we doing too much?
PRACTICE CHANGER

Reconsider the intervals at which you recommend rescreening for osteoporosis; for post-menopausal women with a baseline of normal bone mineral density (BMD) or mild osteopenia, a 15-year interval is probably sufficient.1

STRENGTH OF RECOMMENDATION

B: Based on a single cohort study

Gourlay ML, Fine JP, Preisser JS, et al. Bone density testing interval and transition to osteoporosis in older women. N Engl J Med. 2012;366: 225-233

 

ILLUSTRATIVE CASE

A 67-year-old woman whose recent dual-energy x-ray absorptiometry (DEXA) scan showed mild osteopenia asks when she should have her next bone scan. What should you tell her?

One in 5 people who sustain a hip fracture die within a year,2 and as many as 36% die prematurely.3 Osteoporosis is the primary predictor of fracture risk and, in older white women in particular, low bone mineral density (BMD) increases the likelihood of fracture by 70% to 80%.4

Optimal screening frequency not known
The US Preventive Services Task Force (USPSTF) guideline for osteoporosis screening concludes that there is a lack of evidence about optimal rescreening intervals and states that intervals >2 years may be necessary to better predict fracture risk.5 In addition, the USPSTF cites a prospective study showing that repeat measurement of BMD after 8 years added little predictive value compared with baseline DEXA scan results.6

The prospective cohort study detailed below was undertaken to help guide decisions about how frequently to screen

STUDY SUMMARY: Longer intervals are reasonable for those at low risk

Gourlay et al followed 4957 women age ≥67 years with normal BMD or osteopenia and no history of hip or clinical vertebral fracture or osteoporosis treatment. The primary outcome was the estimated time it would take for 10% of the women to develop osteoporosis. The time until 2% of the women developed such a fracture was the secondary outcome

Participants had baseline DEXA scans, which were repeated at years 2, 6, 8, 10, and 16. The researchers followed the women until they were diagnosed with osteoporosis, started on medication for osteoporosis, or developed a hip or clinical vertebral fracture

After adjusting for multiple covariates (age, body mass index, smoking status, use of glucocorticoids, fracture after age 50, estrogen use, and rheumatoid arthritis), the intervals between baseline testing and the development of osteoporosis were:

  • 16.8 years (95% confidence interval [CI], 11.5-24.6) for women with normal BMD
  • 17.3 years (95% CI, 13.9-21.5) for women with mild osteopenia
  • 4.7 years (95% CI, 4.2-5.2) for women with moderate osteopenia
  • 1.1 year (95% CI, 1.0-1.3) for women with advanced osteopenia

Intervals until 2% of the cohort developed fractures were similar

Overall, the authors used a sensible approach to estimate reasonable intervals between DEXA screenings (TABLE)

TABLE
Suggested rescreening intervals based on DEXA scan results
1

DEXA result (T-score)Rescreening interval*
Normal/mild osteopenia (> -1.50)15 years
Moderate osteopenia (-1.50 to -1.99)5 years
Advanced osteopenia (-2.0 to -2.49)1 year
*Consider reducing these intervals by one-third for women older than 80 years.
 

 

 

WHAT’S NEW: Many DEXA scans can be eliminated

Rescreening all postmenopausal women every 2 years is unlikely to reduce osteoporotic fractures. This cohort study provides evidence that rescreening can often be delayed for many years, depending on the patient’s baseline risk. Changing practice based on these findings can reduce resource utilization without adversely affecting women’s health

CAVEATS: Questions about applicability may remain

This analysis was limited to women ≥67 years, so different results might be obtained from analyses that included younger postmenopausal women. In addition, 99% of the participants were white. Because the prevalence of osteoporosis of the hip among white women is equal to or slightly higher than it is among nonwhite women, it is likely that the suggested intervals are reasonable estimates for women of all races

In women >80 years, the interval between baseline testing and the development of osteoporosis was shorter than that of their younger counterparts. Thus, it might be reasonable to reduce rescreening intervals by a third for women in their 80s

CHALLENGES TO IMPLEMENTATION: Education needed for patients and docs

This study is the best so far to address the frequency of rescreening. In order to implement it, patients as well as clinicians will need to be educated. Effective long-term (>10 y) reminder systems would improve implementation

The recommendations of professional associations may also be a factor. The National Osteoporosis Foundation recommends assessing BMD every 2 years, but notes that more frequent testing may sometimes be warranted.7 The American College of Preventive Medicine recommends that screening for osteoporosis not occur more often than every 2 years.8

Acknowledgement

The PURLs Surveillance System is supported in part by Grant Number UL1RR024999 from the National Center for Research Resources, a Clinical Translational Science Award to the University of Chicago. The content is solely the responsibility of the authors and does not necessarily represent the official views of the National Center for Research Resources or the National Institutes of health.

Files
References

1. Gourlay ML, Fine JP, Preisser JS, et al. Bone-density testing interval and transition to osteoporosis in older women. N Engl J Med. 2012;366:225-233.

2. Leibson CL, Tosteson AN, Gabriel SE, et al. Mortality, disability, and nursing home use for persons with and without hip fracture. J Am Geriatr Soc. 2002;50:1644-1650.

3. Abrahamsen B, van Staa T, Ariely R, et al. Excess mortality following hip fracture: a systematic epidemiological review. Osteoporosis Int. 2009;20:1633-1650.

4. Smith J, Shoukri K. Diagnosis of osteoporosis. Clin Cornerstone. 2000;2:22-33.

5. US Preventive Services Task Force. Screening for osteoporosis: U.S. Preventive Services Task Force recommendation statement. Available at: http://www.uspreventiveservicestaskforce.org/uspstf10/osteoporosis/osteors.htm. Accessed June 15, 2012.

6. Hillier TA, Stone KL, Bauer DC, et al. Evaluating the value of repeat bone mineral density measurement and prediction of fractures in older women. Arch Intern Med. 2007;167:155-160.

7. National Osteoporosis Foundation. Clinician’s guide to prevention and treatment of osteoporosis. 2010. Available at: http://www.nof.org/sites/default/files/pdfs/NOF_ClinicianGuide2009_v7.pdf. Accessed June 30, 2012.

8. Lim LS, Hoeksema LJ, Sherin K. ACPM Prevention Practice Committee. Screening for osteoporosis in the adult US population: ACPM position statement on preventive practice. Am J Prev Med. 2009;36:366-375.

Article PDF
Author and Disclosure Information

Kevin W. Craig, MD, MSPH
Department of Family and Community Medicine, University of Missouri–Columbia

James J. Stevermer, MD, MSPH
Department of Family and Community Medicine, University of Missouri–Columbia

PURLs EDITOR
John Hickner, MD, MSc
Cleveland Clinic

Issue
The Journal of Family Practice - 61(9)
Publications
Topics
Page Number
555-556
Legacy Keywords
Nicole VandenBerg;MD; Shailendra Prasad;MBBS;MPH; speculum exam; lubricating gel; Pap smear; sexually transmitted diseases; vaginal entry; discomfort; pelvic exam
Sections
Files
Files
Author and Disclosure Information

Kevin W. Craig, MD, MSPH
Department of Family and Community Medicine, University of Missouri–Columbia

James J. Stevermer, MD, MSPH
Department of Family and Community Medicine, University of Missouri–Columbia

PURLs EDITOR
John Hickner, MD, MSc
Cleveland Clinic

Author and Disclosure Information

Kevin W. Craig, MD, MSPH
Department of Family and Community Medicine, University of Missouri–Columbia

James J. Stevermer, MD, MSPH
Department of Family and Community Medicine, University of Missouri–Columbia

PURLs EDITOR
John Hickner, MD, MSc
Cleveland Clinic

Article PDF
Article PDF
PRACTICE CHANGER

Reconsider the intervals at which you recommend rescreening for osteoporosis; for post-menopausal women with a baseline of normal bone mineral density (BMD) or mild osteopenia, a 15-year interval is probably sufficient.1

STRENGTH OF RECOMMENDATION

B: Based on a single cohort study

Gourlay ML, Fine JP, Preisser JS, et al. Bone density testing interval and transition to osteoporosis in older women. N Engl J Med. 2012;366: 225-233

 

ILLUSTRATIVE CASE

A 67-year-old woman whose recent dual-energy x-ray absorptiometry (DEXA) scan showed mild osteopenia asks when she should have her next bone scan. What should you tell her?

One in 5 people who sustain a hip fracture die within a year,2 and as many as 36% die prematurely.3 Osteoporosis is the primary predictor of fracture risk and, in older white women in particular, low bone mineral density (BMD) increases the likelihood of fracture by 70% to 80%.4

Optimal screening frequency not known
The US Preventive Services Task Force (USPSTF) guideline for osteoporosis screening concludes that there is a lack of evidence about optimal rescreening intervals and states that intervals >2 years may be necessary to better predict fracture risk.5 In addition, the USPSTF cites a prospective study showing that repeat measurement of BMD after 8 years added little predictive value compared with baseline DEXA scan results.6

The prospective cohort study detailed below was undertaken to help guide decisions about how frequently to screen

STUDY SUMMARY: Longer intervals are reasonable for those at low risk

Gourlay et al followed 4957 women age ≥67 years with normal BMD or osteopenia and no history of hip or clinical vertebral fracture or osteoporosis treatment. The primary outcome was the estimated time it would take for 10% of the women to develop osteoporosis. The time until 2% of the women developed such a fracture was the secondary outcome

Participants had baseline DEXA scans, which were repeated at years 2, 6, 8, 10, and 16. The researchers followed the women until they were diagnosed with osteoporosis, started on medication for osteoporosis, or developed a hip or clinical vertebral fracture

After adjusting for multiple covariates (age, body mass index, smoking status, use of glucocorticoids, fracture after age 50, estrogen use, and rheumatoid arthritis), the intervals between baseline testing and the development of osteoporosis were:

  • 16.8 years (95% confidence interval [CI], 11.5-24.6) for women with normal BMD
  • 17.3 years (95% CI, 13.9-21.5) for women with mild osteopenia
  • 4.7 years (95% CI, 4.2-5.2) for women with moderate osteopenia
  • 1.1 year (95% CI, 1.0-1.3) for women with advanced osteopenia

Intervals until 2% of the cohort developed fractures were similar

Overall, the authors used a sensible approach to estimate reasonable intervals between DEXA screenings (TABLE)

TABLE
Suggested rescreening intervals based on DEXA scan results
1

DEXA result (T-score)Rescreening interval*
Normal/mild osteopenia (> -1.50)15 years
Moderate osteopenia (-1.50 to -1.99)5 years
Advanced osteopenia (-2.0 to -2.49)1 year
*Consider reducing these intervals by one-third for women older than 80 years.
 

 

 

WHAT’S NEW: Many DEXA scans can be eliminated

Rescreening all postmenopausal women every 2 years is unlikely to reduce osteoporotic fractures. This cohort study provides evidence that rescreening can often be delayed for many years, depending on the patient’s baseline risk. Changing practice based on these findings can reduce resource utilization without adversely affecting women’s health

CAVEATS: Questions about applicability may remain

This analysis was limited to women ≥67 years, so different results might be obtained from analyses that included younger postmenopausal women. In addition, 99% of the participants were white. Because the prevalence of osteoporosis of the hip among white women is equal to or slightly higher than it is among nonwhite women, it is likely that the suggested intervals are reasonable estimates for women of all races

In women >80 years, the interval between baseline testing and the development of osteoporosis was shorter than that of their younger counterparts. Thus, it might be reasonable to reduce rescreening intervals by a third for women in their 80s

CHALLENGES TO IMPLEMENTATION: Education needed for patients and docs

This study is the best so far to address the frequency of rescreening. In order to implement it, patients as well as clinicians will need to be educated. Effective long-term (>10 y) reminder systems would improve implementation

The recommendations of professional associations may also be a factor. The National Osteoporosis Foundation recommends assessing BMD every 2 years, but notes that more frequent testing may sometimes be warranted.7 The American College of Preventive Medicine recommends that screening for osteoporosis not occur more often than every 2 years.8

Acknowledgement

The PURLs Surveillance System is supported in part by Grant Number UL1RR024999 from the National Center for Research Resources, a Clinical Translational Science Award to the University of Chicago. The content is solely the responsibility of the authors and does not necessarily represent the official views of the National Center for Research Resources or the National Institutes of health.

PRACTICE CHANGER

Reconsider the intervals at which you recommend rescreening for osteoporosis; for post-menopausal women with a baseline of normal bone mineral density (BMD) or mild osteopenia, a 15-year interval is probably sufficient.1

STRENGTH OF RECOMMENDATION

B: Based on a single cohort study

Gourlay ML, Fine JP, Preisser JS, et al. Bone density testing interval and transition to osteoporosis in older women. N Engl J Med. 2012;366: 225-233

 

ILLUSTRATIVE CASE

A 67-year-old woman whose recent dual-energy x-ray absorptiometry (DEXA) scan showed mild osteopenia asks when she should have her next bone scan. What should you tell her?

One in 5 people who sustain a hip fracture die within a year,2 and as many as 36% die prematurely.3 Osteoporosis is the primary predictor of fracture risk and, in older white women in particular, low bone mineral density (BMD) increases the likelihood of fracture by 70% to 80%.4

Optimal screening frequency not known
The US Preventive Services Task Force (USPSTF) guideline for osteoporosis screening concludes that there is a lack of evidence about optimal rescreening intervals and states that intervals >2 years may be necessary to better predict fracture risk.5 In addition, the USPSTF cites a prospective study showing that repeat measurement of BMD after 8 years added little predictive value compared with baseline DEXA scan results.6

The prospective cohort study detailed below was undertaken to help guide decisions about how frequently to screen

STUDY SUMMARY: Longer intervals are reasonable for those at low risk

Gourlay et al followed 4957 women age ≥67 years with normal BMD or osteopenia and no history of hip or clinical vertebral fracture or osteoporosis treatment. The primary outcome was the estimated time it would take for 10% of the women to develop osteoporosis. The time until 2% of the women developed such a fracture was the secondary outcome

Participants had baseline DEXA scans, which were repeated at years 2, 6, 8, 10, and 16. The researchers followed the women until they were diagnosed with osteoporosis, started on medication for osteoporosis, or developed a hip or clinical vertebral fracture

After adjusting for multiple covariates (age, body mass index, smoking status, use of glucocorticoids, fracture after age 50, estrogen use, and rheumatoid arthritis), the intervals between baseline testing and the development of osteoporosis were:

  • 16.8 years (95% confidence interval [CI], 11.5-24.6) for women with normal BMD
  • 17.3 years (95% CI, 13.9-21.5) for women with mild osteopenia
  • 4.7 years (95% CI, 4.2-5.2) for women with moderate osteopenia
  • 1.1 year (95% CI, 1.0-1.3) for women with advanced osteopenia

Intervals until 2% of the cohort developed fractures were similar

Overall, the authors used a sensible approach to estimate reasonable intervals between DEXA screenings (TABLE)

TABLE
Suggested rescreening intervals based on DEXA scan results
1

DEXA result (T-score)Rescreening interval*
Normal/mild osteopenia (> -1.50)15 years
Moderate osteopenia (-1.50 to -1.99)5 years
Advanced osteopenia (-2.0 to -2.49)1 year
*Consider reducing these intervals by one-third for women older than 80 years.
 

 

 

WHAT’S NEW: Many DEXA scans can be eliminated

Rescreening all postmenopausal women every 2 years is unlikely to reduce osteoporotic fractures. This cohort study provides evidence that rescreening can often be delayed for many years, depending on the patient’s baseline risk. Changing practice based on these findings can reduce resource utilization without adversely affecting women’s health

CAVEATS: Questions about applicability may remain

This analysis was limited to women ≥67 years, so different results might be obtained from analyses that included younger postmenopausal women. In addition, 99% of the participants were white. Because the prevalence of osteoporosis of the hip among white women is equal to or slightly higher than it is among nonwhite women, it is likely that the suggested intervals are reasonable estimates for women of all races

In women >80 years, the interval between baseline testing and the development of osteoporosis was shorter than that of their younger counterparts. Thus, it might be reasonable to reduce rescreening intervals by a third for women in their 80s

CHALLENGES TO IMPLEMENTATION: Education needed for patients and docs

This study is the best so far to address the frequency of rescreening. In order to implement it, patients as well as clinicians will need to be educated. Effective long-term (>10 y) reminder systems would improve implementation

The recommendations of professional associations may also be a factor. The National Osteoporosis Foundation recommends assessing BMD every 2 years, but notes that more frequent testing may sometimes be warranted.7 The American College of Preventive Medicine recommends that screening for osteoporosis not occur more often than every 2 years.8

Acknowledgement

The PURLs Surveillance System is supported in part by Grant Number UL1RR024999 from the National Center for Research Resources, a Clinical Translational Science Award to the University of Chicago. The content is solely the responsibility of the authors and does not necessarily represent the official views of the National Center for Research Resources or the National Institutes of health.

References

1. Gourlay ML, Fine JP, Preisser JS, et al. Bone-density testing interval and transition to osteoporosis in older women. N Engl J Med. 2012;366:225-233.

2. Leibson CL, Tosteson AN, Gabriel SE, et al. Mortality, disability, and nursing home use for persons with and without hip fracture. J Am Geriatr Soc. 2002;50:1644-1650.

3. Abrahamsen B, van Staa T, Ariely R, et al. Excess mortality following hip fracture: a systematic epidemiological review. Osteoporosis Int. 2009;20:1633-1650.

4. Smith J, Shoukri K. Diagnosis of osteoporosis. Clin Cornerstone. 2000;2:22-33.

5. US Preventive Services Task Force. Screening for osteoporosis: U.S. Preventive Services Task Force recommendation statement. Available at: http://www.uspreventiveservicestaskforce.org/uspstf10/osteoporosis/osteors.htm. Accessed June 15, 2012.

6. Hillier TA, Stone KL, Bauer DC, et al. Evaluating the value of repeat bone mineral density measurement and prediction of fractures in older women. Arch Intern Med. 2007;167:155-160.

7. National Osteoporosis Foundation. Clinician’s guide to prevention and treatment of osteoporosis. 2010. Available at: http://www.nof.org/sites/default/files/pdfs/NOF_ClinicianGuide2009_v7.pdf. Accessed June 30, 2012.

8. Lim LS, Hoeksema LJ, Sherin K. ACPM Prevention Practice Committee. Screening for osteoporosis in the adult US population: ACPM position statement on preventive practice. Am J Prev Med. 2009;36:366-375.

References

1. Gourlay ML, Fine JP, Preisser JS, et al. Bone-density testing interval and transition to osteoporosis in older women. N Engl J Med. 2012;366:225-233.

2. Leibson CL, Tosteson AN, Gabriel SE, et al. Mortality, disability, and nursing home use for persons with and without hip fracture. J Am Geriatr Soc. 2002;50:1644-1650.

3. Abrahamsen B, van Staa T, Ariely R, et al. Excess mortality following hip fracture: a systematic epidemiological review. Osteoporosis Int. 2009;20:1633-1650.

4. Smith J, Shoukri K. Diagnosis of osteoporosis. Clin Cornerstone. 2000;2:22-33.

5. US Preventive Services Task Force. Screening for osteoporosis: U.S. Preventive Services Task Force recommendation statement. Available at: http://www.uspreventiveservicestaskforce.org/uspstf10/osteoporosis/osteors.htm. Accessed June 15, 2012.

6. Hillier TA, Stone KL, Bauer DC, et al. Evaluating the value of repeat bone mineral density measurement and prediction of fractures in older women. Arch Intern Med. 2007;167:155-160.

7. National Osteoporosis Foundation. Clinician’s guide to prevention and treatment of osteoporosis. 2010. Available at: http://www.nof.org/sites/default/files/pdfs/NOF_ClinicianGuide2009_v7.pdf. Accessed June 30, 2012.

8. Lim LS, Hoeksema LJ, Sherin K. ACPM Prevention Practice Committee. Screening for osteoporosis in the adult US population: ACPM position statement on preventive practice. Am J Prev Med. 2009;36:366-375.

Issue
The Journal of Family Practice - 61(9)
Issue
The Journal of Family Practice - 61(9)
Page Number
555-556
Page Number
555-556
Publications
Publications
Topics
Article Type
Display Headline
DEXA screening—are we doing too much?
Display Headline
DEXA screening—are we doing too much?
Legacy Keywords
Nicole VandenBerg;MD; Shailendra Prasad;MBBS;MPH; speculum exam; lubricating gel; Pap smear; sexually transmitted diseases; vaginal entry; discomfort; pelvic exam
Legacy Keywords
Nicole VandenBerg;MD; Shailendra Prasad;MBBS;MPH; speculum exam; lubricating gel; Pap smear; sexually transmitted diseases; vaginal entry; discomfort; pelvic exam
Sections
PURLs Copyright

Copyright © 2012 The Family Physicians Inquiries Network. All rights reserved.

Disallow All Ads
Alternative CME
Use ProPublica
Hide sidebar & use full width
render the right sidebar.
Conference Recap Checkbox
Not Conference Recap
Clinical Edge
Display the Slideshow in this Article
Article PDF Media
Media Files

Injection may be the best bet for young athletes’ knee pain

Article Type
Changed
Mon, 01/14/2019 - 11:34
Display Headline
Injection may be the best bet for young athletes’ knee pain
PRACTICE CHANGER

Consider giving dextrose/lidocaine injections to adolescents with Osgood-Schlatter disease (OSD) that persists despite physical therapy.1

STRENGTH OF RECOMMENDATION

A: Based on one well-designed, randomized controlled trial (RCT).

Topol GA, Podesta LA, Reeves KD, et al. Hyperosmolar dextrose injection for recalcitrant Osgood-Schlatter disease. Pediatrics. 2011;128: e1121-e1128.

 

ILLUSTRATIVE CASE

A 13-year-old boy comes in to your office for follow-up of anterior knee pain from OSD that has not responded to 2 months of physical therapy. he is still unable to play on his recreational soccer team. What treatment can you offer to help him return to the sport he enjoys?

OSD is characterized by inflammation of the growth plate just below the knee, the result of repetitive strain on the secondary ossification center of the tibial tuberosity.2 Closure of the tibial growth plate is the definitive remedy for OSD, but the pain that some adolescents experience until that happens can be long-lasting and considerable. Nine years after diagnosis of OSD, one study found, up to 60% of patients who had received conservative treatment reported pain on kneeling and 18% had sports limitations.3

Inability to play may affect self-esteem
Adolescents whose recreational activities are limited due to OSD may experience a number of negative effects, including alienation from friends, altered peer group dynamics, and a decline in self-esteem. Surgery, which involves excision of the pain-producing ossicle with or without tuberculoplasty, relieves the pain and allows patients to return to their chosen sport in 90% to 95% of cases that have not responded to conservative treatment.4,5 For a self-limiting (although prolonged) condition like OSD, most physicians and patients would prefer to avoid surgery and opt for a more conservative approach.

Dextrose injections have been shown to be safe and effective when used for the treatment of tendon and ligamentous disorders such as Achilles tendonitis and lateral epicondylitis, although the mechanism of action is not clear.6,7 The study detailed in this PURL is the first prospective RCT of dextrose injections for the treatment of OSD.

STUDY SUMMARY: injections get adolescents back in the game

Topol et al1 sought to compare the efficacy of injections of dextrose and lidocaine with lidocaine-only injections or supervised usual care in treating OSD in young athletes. Sixty-six Argentinian boys and girls ages 9 to 17 years, all of whom had anterior knee pain and participated in kicking or jumping sports on organized teams, were considered for the study. The absence of either patellofemoral crepitus or proximal patellar tendon tenderness was a prerequisite for participation, as was reproduction of the anterior knee pain and localization of pain precisely to the tibial tuberosity during a single leg squat to confirm the OSD diagnosis.

After diagnosis, the patients completed ≥2 months of formal and gently progressive hamstring stretching, quads strengthening, and gradual reintroduction into their respective sports. Those who experienced pain during team play that persisted for ≥3 months—54 patients, all but 3 of whom were male, with a total of 65 knees requiring treatment—were included in the study. Participants were randomized to the usual care group or to one of the injection groups, which was blinded to patients, guardians, and physicians.

The injection groups received a solution of lidocaine 1%, alone or with 12.5% dextrose, at the start of the study and again at 1 and 2 months. Adequate injection was determined by complete pain relief during a single leg squat, which was also used to determine both proximal and distal points of tenderness. Both injection groups received 0.5-mL injections with a 27-gauge needle, repeated at approximately 1-cm intervals for a total of 3 to 4 midline injections. After 5 minutes, the leg squat was repeated to detect any remaining pain, and painful areas were injected until the patient could do a pain-free leg squat.

Because pain reduction may precede full healing, those in both the lidocaine-only and the dextrose-lidocaine groups received injections on all 3 occasions even if they were pain free. They were instructed to avoid running for a week after the initial treatment and then to run as tolerated. Subsequent treatments required a 3-day rest from running. Participants were able to return to their sport after the second injection and rest period.

Patients in all 3 groups received handouts explaining hamstring stretches and quadriceps strengthening exercises. The usual care group received individual instruction from a physical therapist. They were also given a video and returned at least once, both to ensure that they were performing the exercises correctly and to encourage compliance.

The primary outcome involved the Nirschl Pain Phase Scale (NPPS), a 7-point measure of sports-related symptoms and level of participation. Scores of 4 to 7 represent sports limitation resulting from pain. Scores <4 (which may involve soreness or pain but participation in the sport is unlimited) and 0 (asymptomatic participation) were the threshold goals for the study.

 

 

 

The groups were similar at baseline, and follow-up was 100%. At 3 months, NPPS scores improved more in dextrose-treated knees than in either the lidocaine-treated knees (3.9 vs 2.4; P=.004) or those who received usual care (3.9 vs 1.2; P=.001), and lidocaine alone was significantly better than usual care (2.4 vs 1.2; P=.024). More than 90% of participants in both injection groups achieved unlimited sports participation by 3 months. However, knees treated with dextrose were significantly more likely than lidocaine-treated knees to allow asymptomatic participation (NPPS=0), with 14 of 21 knees and 5 of 22 knees, respectively, being pain-free. After one year, more dextrose-treated knees than lidocaine-treated knees were asymptomatic with sports participation (32 of 38 vs 6 of 13; P=.024).

There were no reported adverse effects during this study and fewer than 10% of subjects required acetaminophen for postinjection pain control.

WHAT’S NEW: OSD can be safely and effectively treated

This study found dextrose injections to be safe, well tolerated, and effective in treating patients with intractable OSD symptoms. The results suggest that the duration of both the sports limitation and sports-related symptoms may be reduced with dextrose injections in adolescent athletes with recalcitrant OSD.

CAVEATS: Lack of validated measure, controls

NPPS is not an ideal measure of OSD symptoms because it has not been validated. The failure to use a validated measure of tendinopathy symptoms (eg, the Victorian Institute of Sport Assessment-Patella8) is a significant limitation of this trial. The athletes included in this study had already failed to respond to the usual treatment, which suggests that injections should be reserved for those who have tried hamstring stretching and quad-strengthening exercises.

CHALLENGES TO IMPLEMENTATION: Patient and provider comfort may be an issue

Although the injections in this study were well tolerated, there is a risk of infection, bleeding, and pain with any injection or invasive procedure. In addition, adolescents often have difficulty tolerating injections, especially repeated needlesticks like those called for in the proposed treatment. The nonviscous nature of dextrose allows 27- to 30-gauge needles to be used, which may make the injections easier for teens to tolerate. Some physicians may be hesitant to start these young patients on a new injectable therapy.

Click here to view PURL METHODOLOGY

References

1. Topol GA, Podesta LA, Reeves KD, et al. Hyperosmolar dextrose injection for recalcitrant Osgood-Schlatter disease. Pediatrics. 2011;128:e1121-e1128.

2. Gholve PA, Scher DM, Khakharia S, et al. Osgood-Schlatter syndrome. Curr Opin Pediatr. 2007;19:44-50.

3. Krause BL, Willimas JP, Caterall A. Natural history of OsgoodSchlatter disease. J Pediatr Orthop. 1990;10:65-68.

4. Nierenberg G, Falah M, Keren Y, et al. Surgical treatment of residual Osgood-Schlatter disease in young adults: role of the mobile osseous fragment. Orthopedics. 2011;34:176.-

5. El-Husseini TF, Abdelgawad AA. Results of surgical treatment of unresolved Osgood-Schlatter disease in adults. J Knee Surg. 2010;23:103-107.

6. Ryan M, Wong A, Taunton J. Favorable outcomes after sonographically guided intratendinous injection of hyperosmolar dextrose for chronic insertional and midportion achilles tendinosis. AJR Am J Roentgenol. 2010;194:1047-1053.

7. Scarpone M, Rabago D, Zgierska A, et al. The efficacy of prolotherapy for lateral epicondylitis: a pilot study. Clin J Sport Med. 2008;18:248-254.

8. Visentini PJ, Khan KM, Cook JL, et al. The VISA score: an index of severity of symptoms in patients with jumper’s knee (patellar tendinosis). Victorian Institute of Sport Tendon Study Group. J Sci Med Sport. 1998;1:22-28.

Article PDF
Author and Disclosure Information
Young athlete sidelined? Check to see how he’s coping
Samantha O’Connell, PhD

Mark Sakr, DO
Department of Family Medicine, University of North Carolina at Chapel Hill

Anne Mounsey, MD
Department of Family Medicine, University of North Carolina at Chapel Hill

PURLs EDITOR
John Hickner, MD, MSc
Cleveland Clinic

Issue
The Journal of Family Practice - 61(8)
Publications
Topics
Page Number
486-488
Sections
Author and Disclosure Information
Young athlete sidelined? Check to see how he’s coping
Samantha O’Connell, PhD

Mark Sakr, DO
Department of Family Medicine, University of North Carolina at Chapel Hill

Anne Mounsey, MD
Department of Family Medicine, University of North Carolina at Chapel Hill

PURLs EDITOR
John Hickner, MD, MSc
Cleveland Clinic

Author and Disclosure Information
Young athlete sidelined? Check to see how he’s coping
Samantha O’Connell, PhD

Mark Sakr, DO
Department of Family Medicine, University of North Carolina at Chapel Hill

Anne Mounsey, MD
Department of Family Medicine, University of North Carolina at Chapel Hill

PURLs EDITOR
John Hickner, MD, MSc
Cleveland Clinic

Article PDF
Article PDF
PRACTICE CHANGER

Consider giving dextrose/lidocaine injections to adolescents with Osgood-Schlatter disease (OSD) that persists despite physical therapy.1

STRENGTH OF RECOMMENDATION

A: Based on one well-designed, randomized controlled trial (RCT).

Topol GA, Podesta LA, Reeves KD, et al. Hyperosmolar dextrose injection for recalcitrant Osgood-Schlatter disease. Pediatrics. 2011;128: e1121-e1128.

 

ILLUSTRATIVE CASE

A 13-year-old boy comes in to your office for follow-up of anterior knee pain from OSD that has not responded to 2 months of physical therapy. he is still unable to play on his recreational soccer team. What treatment can you offer to help him return to the sport he enjoys?

OSD is characterized by inflammation of the growth plate just below the knee, the result of repetitive strain on the secondary ossification center of the tibial tuberosity.2 Closure of the tibial growth plate is the definitive remedy for OSD, but the pain that some adolescents experience until that happens can be long-lasting and considerable. Nine years after diagnosis of OSD, one study found, up to 60% of patients who had received conservative treatment reported pain on kneeling and 18% had sports limitations.3

Inability to play may affect self-esteem
Adolescents whose recreational activities are limited due to OSD may experience a number of negative effects, including alienation from friends, altered peer group dynamics, and a decline in self-esteem. Surgery, which involves excision of the pain-producing ossicle with or without tuberculoplasty, relieves the pain and allows patients to return to their chosen sport in 90% to 95% of cases that have not responded to conservative treatment.4,5 For a self-limiting (although prolonged) condition like OSD, most physicians and patients would prefer to avoid surgery and opt for a more conservative approach.

Dextrose injections have been shown to be safe and effective when used for the treatment of tendon and ligamentous disorders such as Achilles tendonitis and lateral epicondylitis, although the mechanism of action is not clear.6,7 The study detailed in this PURL is the first prospective RCT of dextrose injections for the treatment of OSD.

STUDY SUMMARY: injections get adolescents back in the game

Topol et al1 sought to compare the efficacy of injections of dextrose and lidocaine with lidocaine-only injections or supervised usual care in treating OSD in young athletes. Sixty-six Argentinian boys and girls ages 9 to 17 years, all of whom had anterior knee pain and participated in kicking or jumping sports on organized teams, were considered for the study. The absence of either patellofemoral crepitus or proximal patellar tendon tenderness was a prerequisite for participation, as was reproduction of the anterior knee pain and localization of pain precisely to the tibial tuberosity during a single leg squat to confirm the OSD diagnosis.

After diagnosis, the patients completed ≥2 months of formal and gently progressive hamstring stretching, quads strengthening, and gradual reintroduction into their respective sports. Those who experienced pain during team play that persisted for ≥3 months—54 patients, all but 3 of whom were male, with a total of 65 knees requiring treatment—were included in the study. Participants were randomized to the usual care group or to one of the injection groups, which was blinded to patients, guardians, and physicians.

The injection groups received a solution of lidocaine 1%, alone or with 12.5% dextrose, at the start of the study and again at 1 and 2 months. Adequate injection was determined by complete pain relief during a single leg squat, which was also used to determine both proximal and distal points of tenderness. Both injection groups received 0.5-mL injections with a 27-gauge needle, repeated at approximately 1-cm intervals for a total of 3 to 4 midline injections. After 5 minutes, the leg squat was repeated to detect any remaining pain, and painful areas were injected until the patient could do a pain-free leg squat.

Because pain reduction may precede full healing, those in both the lidocaine-only and the dextrose-lidocaine groups received injections on all 3 occasions even if they were pain free. They were instructed to avoid running for a week after the initial treatment and then to run as tolerated. Subsequent treatments required a 3-day rest from running. Participants were able to return to their sport after the second injection and rest period.

Patients in all 3 groups received handouts explaining hamstring stretches and quadriceps strengthening exercises. The usual care group received individual instruction from a physical therapist. They were also given a video and returned at least once, both to ensure that they were performing the exercises correctly and to encourage compliance.

The primary outcome involved the Nirschl Pain Phase Scale (NPPS), a 7-point measure of sports-related symptoms and level of participation. Scores of 4 to 7 represent sports limitation resulting from pain. Scores <4 (which may involve soreness or pain but participation in the sport is unlimited) and 0 (asymptomatic participation) were the threshold goals for the study.

 

 

 

The groups were similar at baseline, and follow-up was 100%. At 3 months, NPPS scores improved more in dextrose-treated knees than in either the lidocaine-treated knees (3.9 vs 2.4; P=.004) or those who received usual care (3.9 vs 1.2; P=.001), and lidocaine alone was significantly better than usual care (2.4 vs 1.2; P=.024). More than 90% of participants in both injection groups achieved unlimited sports participation by 3 months. However, knees treated with dextrose were significantly more likely than lidocaine-treated knees to allow asymptomatic participation (NPPS=0), with 14 of 21 knees and 5 of 22 knees, respectively, being pain-free. After one year, more dextrose-treated knees than lidocaine-treated knees were asymptomatic with sports participation (32 of 38 vs 6 of 13; P=.024).

There were no reported adverse effects during this study and fewer than 10% of subjects required acetaminophen for postinjection pain control.

WHAT’S NEW: OSD can be safely and effectively treated

This study found dextrose injections to be safe, well tolerated, and effective in treating patients with intractable OSD symptoms. The results suggest that the duration of both the sports limitation and sports-related symptoms may be reduced with dextrose injections in adolescent athletes with recalcitrant OSD.

CAVEATS: Lack of validated measure, controls

NPPS is not an ideal measure of OSD symptoms because it has not been validated. The failure to use a validated measure of tendinopathy symptoms (eg, the Victorian Institute of Sport Assessment-Patella8) is a significant limitation of this trial. The athletes included in this study had already failed to respond to the usual treatment, which suggests that injections should be reserved for those who have tried hamstring stretching and quad-strengthening exercises.

CHALLENGES TO IMPLEMENTATION: Patient and provider comfort may be an issue

Although the injections in this study were well tolerated, there is a risk of infection, bleeding, and pain with any injection or invasive procedure. In addition, adolescents often have difficulty tolerating injections, especially repeated needlesticks like those called for in the proposed treatment. The nonviscous nature of dextrose allows 27- to 30-gauge needles to be used, which may make the injections easier for teens to tolerate. Some physicians may be hesitant to start these young patients on a new injectable therapy.

Click here to view PURL METHODOLOGY

PRACTICE CHANGER

Consider giving dextrose/lidocaine injections to adolescents with Osgood-Schlatter disease (OSD) that persists despite physical therapy.1

STRENGTH OF RECOMMENDATION

A: Based on one well-designed, randomized controlled trial (RCT).

Topol GA, Podesta LA, Reeves KD, et al. Hyperosmolar dextrose injection for recalcitrant Osgood-Schlatter disease. Pediatrics. 2011;128: e1121-e1128.

 

ILLUSTRATIVE CASE

A 13-year-old boy comes in to your office for follow-up of anterior knee pain from OSD that has not responded to 2 months of physical therapy. he is still unable to play on his recreational soccer team. What treatment can you offer to help him return to the sport he enjoys?

OSD is characterized by inflammation of the growth plate just below the knee, the result of repetitive strain on the secondary ossification center of the tibial tuberosity.2 Closure of the tibial growth plate is the definitive remedy for OSD, but the pain that some adolescents experience until that happens can be long-lasting and considerable. Nine years after diagnosis of OSD, one study found, up to 60% of patients who had received conservative treatment reported pain on kneeling and 18% had sports limitations.3

Inability to play may affect self-esteem
Adolescents whose recreational activities are limited due to OSD may experience a number of negative effects, including alienation from friends, altered peer group dynamics, and a decline in self-esteem. Surgery, which involves excision of the pain-producing ossicle with or without tuberculoplasty, relieves the pain and allows patients to return to their chosen sport in 90% to 95% of cases that have not responded to conservative treatment.4,5 For a self-limiting (although prolonged) condition like OSD, most physicians and patients would prefer to avoid surgery and opt for a more conservative approach.

Dextrose injections have been shown to be safe and effective when used for the treatment of tendon and ligamentous disorders such as Achilles tendonitis and lateral epicondylitis, although the mechanism of action is not clear.6,7 The study detailed in this PURL is the first prospective RCT of dextrose injections for the treatment of OSD.

STUDY SUMMARY: injections get adolescents back in the game

Topol et al1 sought to compare the efficacy of injections of dextrose and lidocaine with lidocaine-only injections or supervised usual care in treating OSD in young athletes. Sixty-six Argentinian boys and girls ages 9 to 17 years, all of whom had anterior knee pain and participated in kicking or jumping sports on organized teams, were considered for the study. The absence of either patellofemoral crepitus or proximal patellar tendon tenderness was a prerequisite for participation, as was reproduction of the anterior knee pain and localization of pain precisely to the tibial tuberosity during a single leg squat to confirm the OSD diagnosis.

After diagnosis, the patients completed ≥2 months of formal and gently progressive hamstring stretching, quads strengthening, and gradual reintroduction into their respective sports. Those who experienced pain during team play that persisted for ≥3 months—54 patients, all but 3 of whom were male, with a total of 65 knees requiring treatment—were included in the study. Participants were randomized to the usual care group or to one of the injection groups, which was blinded to patients, guardians, and physicians.

The injection groups received a solution of lidocaine 1%, alone or with 12.5% dextrose, at the start of the study and again at 1 and 2 months. Adequate injection was determined by complete pain relief during a single leg squat, which was also used to determine both proximal and distal points of tenderness. Both injection groups received 0.5-mL injections with a 27-gauge needle, repeated at approximately 1-cm intervals for a total of 3 to 4 midline injections. After 5 minutes, the leg squat was repeated to detect any remaining pain, and painful areas were injected until the patient could do a pain-free leg squat.

Because pain reduction may precede full healing, those in both the lidocaine-only and the dextrose-lidocaine groups received injections on all 3 occasions even if they were pain free. They were instructed to avoid running for a week after the initial treatment and then to run as tolerated. Subsequent treatments required a 3-day rest from running. Participants were able to return to their sport after the second injection and rest period.

Patients in all 3 groups received handouts explaining hamstring stretches and quadriceps strengthening exercises. The usual care group received individual instruction from a physical therapist. They were also given a video and returned at least once, both to ensure that they were performing the exercises correctly and to encourage compliance.

The primary outcome involved the Nirschl Pain Phase Scale (NPPS), a 7-point measure of sports-related symptoms and level of participation. Scores of 4 to 7 represent sports limitation resulting from pain. Scores <4 (which may involve soreness or pain but participation in the sport is unlimited) and 0 (asymptomatic participation) were the threshold goals for the study.

 

 

 

The groups were similar at baseline, and follow-up was 100%. At 3 months, NPPS scores improved more in dextrose-treated knees than in either the lidocaine-treated knees (3.9 vs 2.4; P=.004) or those who received usual care (3.9 vs 1.2; P=.001), and lidocaine alone was significantly better than usual care (2.4 vs 1.2; P=.024). More than 90% of participants in both injection groups achieved unlimited sports participation by 3 months. However, knees treated with dextrose were significantly more likely than lidocaine-treated knees to allow asymptomatic participation (NPPS=0), with 14 of 21 knees and 5 of 22 knees, respectively, being pain-free. After one year, more dextrose-treated knees than lidocaine-treated knees were asymptomatic with sports participation (32 of 38 vs 6 of 13; P=.024).

There were no reported adverse effects during this study and fewer than 10% of subjects required acetaminophen for postinjection pain control.

WHAT’S NEW: OSD can be safely and effectively treated

This study found dextrose injections to be safe, well tolerated, and effective in treating patients with intractable OSD symptoms. The results suggest that the duration of both the sports limitation and sports-related symptoms may be reduced with dextrose injections in adolescent athletes with recalcitrant OSD.

CAVEATS: Lack of validated measure, controls

NPPS is not an ideal measure of OSD symptoms because it has not been validated. The failure to use a validated measure of tendinopathy symptoms (eg, the Victorian Institute of Sport Assessment-Patella8) is a significant limitation of this trial. The athletes included in this study had already failed to respond to the usual treatment, which suggests that injections should be reserved for those who have tried hamstring stretching and quad-strengthening exercises.

CHALLENGES TO IMPLEMENTATION: Patient and provider comfort may be an issue

Although the injections in this study were well tolerated, there is a risk of infection, bleeding, and pain with any injection or invasive procedure. In addition, adolescents often have difficulty tolerating injections, especially repeated needlesticks like those called for in the proposed treatment. The nonviscous nature of dextrose allows 27- to 30-gauge needles to be used, which may make the injections easier for teens to tolerate. Some physicians may be hesitant to start these young patients on a new injectable therapy.

Click here to view PURL METHODOLOGY

References

1. Topol GA, Podesta LA, Reeves KD, et al. Hyperosmolar dextrose injection for recalcitrant Osgood-Schlatter disease. Pediatrics. 2011;128:e1121-e1128.

2. Gholve PA, Scher DM, Khakharia S, et al. Osgood-Schlatter syndrome. Curr Opin Pediatr. 2007;19:44-50.

3. Krause BL, Willimas JP, Caterall A. Natural history of OsgoodSchlatter disease. J Pediatr Orthop. 1990;10:65-68.

4. Nierenberg G, Falah M, Keren Y, et al. Surgical treatment of residual Osgood-Schlatter disease in young adults: role of the mobile osseous fragment. Orthopedics. 2011;34:176.-

5. El-Husseini TF, Abdelgawad AA. Results of surgical treatment of unresolved Osgood-Schlatter disease in adults. J Knee Surg. 2010;23:103-107.

6. Ryan M, Wong A, Taunton J. Favorable outcomes after sonographically guided intratendinous injection of hyperosmolar dextrose for chronic insertional and midportion achilles tendinosis. AJR Am J Roentgenol. 2010;194:1047-1053.

7. Scarpone M, Rabago D, Zgierska A, et al. The efficacy of prolotherapy for lateral epicondylitis: a pilot study. Clin J Sport Med. 2008;18:248-254.

8. Visentini PJ, Khan KM, Cook JL, et al. The VISA score: an index of severity of symptoms in patients with jumper’s knee (patellar tendinosis). Victorian Institute of Sport Tendon Study Group. J Sci Med Sport. 1998;1:22-28.

References

1. Topol GA, Podesta LA, Reeves KD, et al. Hyperosmolar dextrose injection for recalcitrant Osgood-Schlatter disease. Pediatrics. 2011;128:e1121-e1128.

2. Gholve PA, Scher DM, Khakharia S, et al. Osgood-Schlatter syndrome. Curr Opin Pediatr. 2007;19:44-50.

3. Krause BL, Willimas JP, Caterall A. Natural history of OsgoodSchlatter disease. J Pediatr Orthop. 1990;10:65-68.

4. Nierenberg G, Falah M, Keren Y, et al. Surgical treatment of residual Osgood-Schlatter disease in young adults: role of the mobile osseous fragment. Orthopedics. 2011;34:176.-

5. El-Husseini TF, Abdelgawad AA. Results of surgical treatment of unresolved Osgood-Schlatter disease in adults. J Knee Surg. 2010;23:103-107.

6. Ryan M, Wong A, Taunton J. Favorable outcomes after sonographically guided intratendinous injection of hyperosmolar dextrose for chronic insertional and midportion achilles tendinosis. AJR Am J Roentgenol. 2010;194:1047-1053.

7. Scarpone M, Rabago D, Zgierska A, et al. The efficacy of prolotherapy for lateral epicondylitis: a pilot study. Clin J Sport Med. 2008;18:248-254.

8. Visentini PJ, Khan KM, Cook JL, et al. The VISA score: an index of severity of symptoms in patients with jumper’s knee (patellar tendinosis). Victorian Institute of Sport Tendon Study Group. J Sci Med Sport. 1998;1:22-28.

Issue
The Journal of Family Practice - 61(8)
Issue
The Journal of Family Practice - 61(8)
Page Number
486-488
Page Number
486-488
Publications
Publications
Topics
Article Type
Display Headline
Injection may be the best bet for young athletes’ knee pain
Display Headline
Injection may be the best bet for young athletes’ knee pain
Sections
PURLs Copyright

Copyright © 2012 The Family Physicians Inquiries Network. All rights reserved.

Disallow All Ads
Alternative CME
Article PDF Media

The shrinking case for saw palmetto

Article Type
Changed
Mon, 01/14/2019 - 11:34
Display Headline
The shrinking case for saw palmetto

PRACTICE CHANGER

Advise men with benign prostatic hyperplasia (BPH) not to take saw palmetto for urinary symptoms. Explain that it has not been found to alleviate symptoms, even at triple the standard dose.1

A: Based on evidence from a high-quality randomized controlled trial (RCT)1 and a 2009 meta-analysis.2

1. Barry MJ, Meleth S, Lee JY, et al. Effect of increasing doses of saw palmetto extract on lower urinary tract symptoms: a randomized trial. JAMA. 2011;306:1344-1351.

ILLUSTRATIVE CASE

A 66-year-old man comes to your office complaining of urinary frequency and straining to begin urination. He was recently diagnosed with BPh by a urologist, but is hesitant to begin taking a prescription drug. The patient, who is on a fixed income, asks you if saw palmetto extract might relieve his urinary symptoms. What should you tell him?

Roughly 40% of American men older than 60 years and nearly 90% of men older than 80 suffer from BPH and the troublesome lower urinary tract symptoms (LUTS) that it causes.3 Established medical and surgical options, as well as over-the-counter (OTC) plant-based products, are used for symptom relief. The OTC remedy most commonly used for BPH is Serenoa repens, derived from the saw palmetto dwarf palm tree. In a 2007 survey, 1.6 million US adults reported using saw palmetto extract, often as a treatment for BPH, in the 30 days prior to the survey.4

Until now, more questions than answers
As a family physician, you undoubtedly have many patients who are taking or considering taking saw palmetto for relief of BPH symptoms. The significant adverse effects of alpha-blockers and 5-alpha-reductase inhibitors, which are typically prescribed for LUTS—including decreased libido and dizziness—may help account for their interest in this alternative treatment.5,6

Until recently, evidence of saw palmetto’s efficacy has been limited and conflicting, despite widespread use of the extract. That has left many of us wondering whether we should recommend that men with BPH try saw palmetto despite the limited evidence; whether it is effective for some, but not all, BPH symptoms; and whether an increase in dose would increase its efficacy.

A 2002 Cochrane meta-analysis of 21 trials of saw palmetto extract for LUTS reported reduced nocturia, improved self-reported symptoms, and increased peak uroflow compared with placebo, without significant adverse effects.7 An updated Cochrane review published in 2009 included several more rigorous trials—and had very different results: This meta-analysis, which was based on 30 trials, found a reduction in nocturia, but failed to show improvement in other self-reported symptoms or peak uroflow.2

The largest trial included in the 2009 review was the Saw Palmetto Treatment for Enlarged Prostates (STEP) study,8 a one-year study with 225 participants. Its findings: no improvement in the treatment group compared with the placebo group in symptom scores or any secondary endpoints, and no important toxicity.8 Of note, the STEP study and most trials included in the 2009 Cochrane review used the standard saw palmetto extract dose of 160 mg twice daily.1,2

STUDY SUMMARY: Saw palmetto is ineffective, even at triple the dose

Barry et al conducted a 72-week double-blind, multicenter placebo-controlled trial to assess the effect of double (640 mg/d) and triple (960 mg/d) the standard dose of saw palmetto extract on BPH symptoms.1 The study included 369 men with moderate LUTS who had not recently received treatment for BPH. Exclusion criteria included a history of invasive BPH treatment, recent treatment with either an alpha-blocker or a 5-alpha-reductase inhibitor; recent phytotherapy, including saw palmetto; and a history of prostate or bladder cancer. Participants were randomized to receive either saw palmetto extract or an identical-looking placebo gel cap. Doses started at 320 mg/d and were increased to 640 mg/d at 24 weeks and 960 mg/d at 48 weeks.

The primary outcome was the change in the American Urological Association Symptom Index (AUASI) score from baseline to 72 weeks. AUASI, a scale of 0 to 35 in which higher numbers represent increased symptoms, is the same scoring tool used in both the Cochrane review and the STEP trial. Secondary measures included other symptom scales, peak uroflow, and poststudy satisfaction. The treatment and placebo groups had statistically identical baseline characteristics, and the sample size was large enough to detect clinically significant differences.

The AUASI score decreased by a mean of 2.20 points (95% confidence interval [CI], -3.04 to -0.36) in the group that received saw palmetto and by 2.99 points (95% CI, -3.81 to -2.17) in the placebo group—a mean difference of 0.79 in favor of the placebo group (P=.91). The proportion of participants achieving a 3-point reduction in AUASI score was statistically similar between the 2 groups (P=0.66). There was no significant dose response difference between the 2 groups, and saw palmetto proved to be no better than placebo for any of the secondary outcomes.

 

 

Subgroup analysis did not reveal any results that differed from the main outcomes. The only adverse events that were significantly different between the 2 groups related to physical injury or trauma, which were unlikely to be due to the intervention.1

By using the same symptom scoring system (AUASI) as many studies in the previous Cochrane reviews, Barry et al were able to compare their findings with those of other high-quality studies with similar methodologies and outcome measures. Despite using an even higher dose of the extract, the results of this trial are remarkably consistent with previous conclusions: Saw palmetto is not an effective treatment for symptoms associated with BPH. Moreover, this trial had a broad base of participants similar to the population in a primary care practice, including patients who would typically choose a natural remedy for LUTS.1

WHAT’S NEW: We now have answers to queries about saw palmetto

This trial is the first to compare higher doses of saw palmetto with placebo to assess a dosage threshold for effectiveness. While the study found no evidence of saw palmetto toxicity even at these higher doses, the extract did not outperform placebo for any measured outcome.1

This high-quality study confirmed the recent series of rigorous studies with negative outcomes by showing that the use of a standard dosage was not a study limitation and that saw palmetto extract is not effective for treating LUTS at any dosage. This trial should substantially affect future guideline recommendations that were limited by methodological concerns in the past.9-11

CAVEATS: In theory, individual preparations could work differently

It is not possible to be absolutely certain that these findings apply to all saw palmetto extract preparations, given the unknown active ingredients and unknown mechanism of action. However, the researchers used a high-quality preparation (a proprietary lipidic ethanolic extract) of saw palmetto at higher doses than the STEP trial and came to a similar conclusion, making it highly unlikely that another preparation would perform differently.

CHALLENGES TO IMPLEMENTATION: There are none

We see no challenges to implementation of this recommendation.

Acknowledgement

The PURLs Surveillance System is supported in part by Grant Number UL1RR024999 from the National Center for Research Resources, a Clinical Translational Science Award to the University of Chicago. The content is solely the responsibility of the authors and does not necessarily represent the official views of the National Center For Research Resources or the National Institutes of Health.

References

1. Barry MJ, Meleth S, Lee JY, et al. Effect of increasing doses of saw palmetto extract on lower urinary tract symptoms: a randomized trial. JAMA. 2011;306:1344-1351.

2. Tacklind J, MacDonald R, Rutks I, et al. Serenoa repens for benign prostatic hyperplasia. Cochrane Database Syst Rev. 2009;(2):CD001423.-

3. Roehrborn CG, McConnell JD. Etiology, Pathophysiology, Epidemiology, and Natural History of Benign Prostatic Hyperplasia. 8th ed. Philadelphia, Pa: Campbell’s Urology; 2002.

4. Barnes PM, Bloom B, Nahin RL. Complementary and alternative medicine use among adults and children: United States, 2007. Natl Health Stat Report. 2008;(12):1-23.

5. Traish AM, Hassani J, Guay AT, et al. Adverse side effects of 5-alpha-reductase inhibitors therapy: persistent diminished libido and erectile dysfunction and depression in a subset of patients. J Sex Med. 2011;8:872-884.

6. Clifford GM, Farmer RD. Medical therapy for benign prostatic hyperplasia: a review of the literature. Eur Urol. 2000;38:2-19.

7. Wilt T, Ishani A, MacDonald R. Serenoa repens for benign prostatic hyperplasia. Cochrane Database Syst Rev. 2002;(3):CD001423.-

8. Bent S, Kane C, Shinohara K, et al. Saw palmetto for benign prostatic hyperplasia. N Engl J Med. 2006;354:557-566.

9. Benign prostatic hyperplasia: treatment {saw palmetto}. In: DynaMed. Available at: http://www.DynamicMedical.com. Accessed April 16, 2012.

10. Saper R. Clinical use of saw palmetto {saw palmetto}. In: Basow DS, ed. UpToDate [database online]. Waltham, Mass: UpToDate 2012. Available at: http://www.uptodate.com. Accessed April 16, 2012.

11. Agency for Healthcare Research and Quality. Guidelines on the treatment of non-neurogenic male LUTS. Available at: http://www.guideline.gov/content.aspx?id=34066. Accessed June 17, 2012.

Article PDF
Author and Disclosure Information

Jason Ricco, MD, MPH
North Memorial Family Medicine Residency, University of Minnesota, Minneapolis

Shailendra Prasad, MBBS, MPH
North Memorial Family Medicine Residency, University of Minnesota, Minneapolis

PURLs EDITOR
John Hickner, MD, MSc

Cleveland Clinic

Issue
The Journal of Family Practice - 61(7)
Publications
Topics
Page Number
418-420
Sections
Author and Disclosure Information

Jason Ricco, MD, MPH
North Memorial Family Medicine Residency, University of Minnesota, Minneapolis

Shailendra Prasad, MBBS, MPH
North Memorial Family Medicine Residency, University of Minnesota, Minneapolis

PURLs EDITOR
John Hickner, MD, MSc

Cleveland Clinic

Author and Disclosure Information

Jason Ricco, MD, MPH
North Memorial Family Medicine Residency, University of Minnesota, Minneapolis

Shailendra Prasad, MBBS, MPH
North Memorial Family Medicine Residency, University of Minnesota, Minneapolis

PURLs EDITOR
John Hickner, MD, MSc

Cleveland Clinic

Article PDF
Article PDF

PRACTICE CHANGER

Advise men with benign prostatic hyperplasia (BPH) not to take saw palmetto for urinary symptoms. Explain that it has not been found to alleviate symptoms, even at triple the standard dose.1

A: Based on evidence from a high-quality randomized controlled trial (RCT)1 and a 2009 meta-analysis.2

1. Barry MJ, Meleth S, Lee JY, et al. Effect of increasing doses of saw palmetto extract on lower urinary tract symptoms: a randomized trial. JAMA. 2011;306:1344-1351.

ILLUSTRATIVE CASE

A 66-year-old man comes to your office complaining of urinary frequency and straining to begin urination. He was recently diagnosed with BPh by a urologist, but is hesitant to begin taking a prescription drug. The patient, who is on a fixed income, asks you if saw palmetto extract might relieve his urinary symptoms. What should you tell him?

Roughly 40% of American men older than 60 years and nearly 90% of men older than 80 suffer from BPH and the troublesome lower urinary tract symptoms (LUTS) that it causes.3 Established medical and surgical options, as well as over-the-counter (OTC) plant-based products, are used for symptom relief. The OTC remedy most commonly used for BPH is Serenoa repens, derived from the saw palmetto dwarf palm tree. In a 2007 survey, 1.6 million US adults reported using saw palmetto extract, often as a treatment for BPH, in the 30 days prior to the survey.4

Until now, more questions than answers
As a family physician, you undoubtedly have many patients who are taking or considering taking saw palmetto for relief of BPH symptoms. The significant adverse effects of alpha-blockers and 5-alpha-reductase inhibitors, which are typically prescribed for LUTS—including decreased libido and dizziness—may help account for their interest in this alternative treatment.5,6

Until recently, evidence of saw palmetto’s efficacy has been limited and conflicting, despite widespread use of the extract. That has left many of us wondering whether we should recommend that men with BPH try saw palmetto despite the limited evidence; whether it is effective for some, but not all, BPH symptoms; and whether an increase in dose would increase its efficacy.

A 2002 Cochrane meta-analysis of 21 trials of saw palmetto extract for LUTS reported reduced nocturia, improved self-reported symptoms, and increased peak uroflow compared with placebo, without significant adverse effects.7 An updated Cochrane review published in 2009 included several more rigorous trials—and had very different results: This meta-analysis, which was based on 30 trials, found a reduction in nocturia, but failed to show improvement in other self-reported symptoms or peak uroflow.2

The largest trial included in the 2009 review was the Saw Palmetto Treatment for Enlarged Prostates (STEP) study,8 a one-year study with 225 participants. Its findings: no improvement in the treatment group compared with the placebo group in symptom scores or any secondary endpoints, and no important toxicity.8 Of note, the STEP study and most trials included in the 2009 Cochrane review used the standard saw palmetto extract dose of 160 mg twice daily.1,2

STUDY SUMMARY: Saw palmetto is ineffective, even at triple the dose

Barry et al conducted a 72-week double-blind, multicenter placebo-controlled trial to assess the effect of double (640 mg/d) and triple (960 mg/d) the standard dose of saw palmetto extract on BPH symptoms.1 The study included 369 men with moderate LUTS who had not recently received treatment for BPH. Exclusion criteria included a history of invasive BPH treatment, recent treatment with either an alpha-blocker or a 5-alpha-reductase inhibitor; recent phytotherapy, including saw palmetto; and a history of prostate or bladder cancer. Participants were randomized to receive either saw palmetto extract or an identical-looking placebo gel cap. Doses started at 320 mg/d and were increased to 640 mg/d at 24 weeks and 960 mg/d at 48 weeks.

The primary outcome was the change in the American Urological Association Symptom Index (AUASI) score from baseline to 72 weeks. AUASI, a scale of 0 to 35 in which higher numbers represent increased symptoms, is the same scoring tool used in both the Cochrane review and the STEP trial. Secondary measures included other symptom scales, peak uroflow, and poststudy satisfaction. The treatment and placebo groups had statistically identical baseline characteristics, and the sample size was large enough to detect clinically significant differences.

The AUASI score decreased by a mean of 2.20 points (95% confidence interval [CI], -3.04 to -0.36) in the group that received saw palmetto and by 2.99 points (95% CI, -3.81 to -2.17) in the placebo group—a mean difference of 0.79 in favor of the placebo group (P=.91). The proportion of participants achieving a 3-point reduction in AUASI score was statistically similar between the 2 groups (P=0.66). There was no significant dose response difference between the 2 groups, and saw palmetto proved to be no better than placebo for any of the secondary outcomes.

 

 

Subgroup analysis did not reveal any results that differed from the main outcomes. The only adverse events that were significantly different between the 2 groups related to physical injury or trauma, which were unlikely to be due to the intervention.1

By using the same symptom scoring system (AUASI) as many studies in the previous Cochrane reviews, Barry et al were able to compare their findings with those of other high-quality studies with similar methodologies and outcome measures. Despite using an even higher dose of the extract, the results of this trial are remarkably consistent with previous conclusions: Saw palmetto is not an effective treatment for symptoms associated with BPH. Moreover, this trial had a broad base of participants similar to the population in a primary care practice, including patients who would typically choose a natural remedy for LUTS.1

WHAT’S NEW: We now have answers to queries about saw palmetto

This trial is the first to compare higher doses of saw palmetto with placebo to assess a dosage threshold for effectiveness. While the study found no evidence of saw palmetto toxicity even at these higher doses, the extract did not outperform placebo for any measured outcome.1

This high-quality study confirmed the recent series of rigorous studies with negative outcomes by showing that the use of a standard dosage was not a study limitation and that saw palmetto extract is not effective for treating LUTS at any dosage. This trial should substantially affect future guideline recommendations that were limited by methodological concerns in the past.9-11

CAVEATS: In theory, individual preparations could work differently

It is not possible to be absolutely certain that these findings apply to all saw palmetto extract preparations, given the unknown active ingredients and unknown mechanism of action. However, the researchers used a high-quality preparation (a proprietary lipidic ethanolic extract) of saw palmetto at higher doses than the STEP trial and came to a similar conclusion, making it highly unlikely that another preparation would perform differently.

CHALLENGES TO IMPLEMENTATION: There are none

We see no challenges to implementation of this recommendation.

Acknowledgement

The PURLs Surveillance System is supported in part by Grant Number UL1RR024999 from the National Center for Research Resources, a Clinical Translational Science Award to the University of Chicago. The content is solely the responsibility of the authors and does not necessarily represent the official views of the National Center For Research Resources or the National Institutes of Health.

PRACTICE CHANGER

Advise men with benign prostatic hyperplasia (BPH) not to take saw palmetto for urinary symptoms. Explain that it has not been found to alleviate symptoms, even at triple the standard dose.1

A: Based on evidence from a high-quality randomized controlled trial (RCT)1 and a 2009 meta-analysis.2

1. Barry MJ, Meleth S, Lee JY, et al. Effect of increasing doses of saw palmetto extract on lower urinary tract symptoms: a randomized trial. JAMA. 2011;306:1344-1351.

ILLUSTRATIVE CASE

A 66-year-old man comes to your office complaining of urinary frequency and straining to begin urination. He was recently diagnosed with BPh by a urologist, but is hesitant to begin taking a prescription drug. The patient, who is on a fixed income, asks you if saw palmetto extract might relieve his urinary symptoms. What should you tell him?

Roughly 40% of American men older than 60 years and nearly 90% of men older than 80 suffer from BPH and the troublesome lower urinary tract symptoms (LUTS) that it causes.3 Established medical and surgical options, as well as over-the-counter (OTC) plant-based products, are used for symptom relief. The OTC remedy most commonly used for BPH is Serenoa repens, derived from the saw palmetto dwarf palm tree. In a 2007 survey, 1.6 million US adults reported using saw palmetto extract, often as a treatment for BPH, in the 30 days prior to the survey.4

Until now, more questions than answers
As a family physician, you undoubtedly have many patients who are taking or considering taking saw palmetto for relief of BPH symptoms. The significant adverse effects of alpha-blockers and 5-alpha-reductase inhibitors, which are typically prescribed for LUTS—including decreased libido and dizziness—may help account for their interest in this alternative treatment.5,6

Until recently, evidence of saw palmetto’s efficacy has been limited and conflicting, despite widespread use of the extract. That has left many of us wondering whether we should recommend that men with BPH try saw palmetto despite the limited evidence; whether it is effective for some, but not all, BPH symptoms; and whether an increase in dose would increase its efficacy.

A 2002 Cochrane meta-analysis of 21 trials of saw palmetto extract for LUTS reported reduced nocturia, improved self-reported symptoms, and increased peak uroflow compared with placebo, without significant adverse effects.7 An updated Cochrane review published in 2009 included several more rigorous trials—and had very different results: This meta-analysis, which was based on 30 trials, found a reduction in nocturia, but failed to show improvement in other self-reported symptoms or peak uroflow.2

The largest trial included in the 2009 review was the Saw Palmetto Treatment for Enlarged Prostates (STEP) study,8 a one-year study with 225 participants. Its findings: no improvement in the treatment group compared with the placebo group in symptom scores or any secondary endpoints, and no important toxicity.8 Of note, the STEP study and most trials included in the 2009 Cochrane review used the standard saw palmetto extract dose of 160 mg twice daily.1,2

STUDY SUMMARY: Saw palmetto is ineffective, even at triple the dose

Barry et al conducted a 72-week double-blind, multicenter placebo-controlled trial to assess the effect of double (640 mg/d) and triple (960 mg/d) the standard dose of saw palmetto extract on BPH symptoms.1 The study included 369 men with moderate LUTS who had not recently received treatment for BPH. Exclusion criteria included a history of invasive BPH treatment, recent treatment with either an alpha-blocker or a 5-alpha-reductase inhibitor; recent phytotherapy, including saw palmetto; and a history of prostate or bladder cancer. Participants were randomized to receive either saw palmetto extract or an identical-looking placebo gel cap. Doses started at 320 mg/d and were increased to 640 mg/d at 24 weeks and 960 mg/d at 48 weeks.

The primary outcome was the change in the American Urological Association Symptom Index (AUASI) score from baseline to 72 weeks. AUASI, a scale of 0 to 35 in which higher numbers represent increased symptoms, is the same scoring tool used in both the Cochrane review and the STEP trial. Secondary measures included other symptom scales, peak uroflow, and poststudy satisfaction. The treatment and placebo groups had statistically identical baseline characteristics, and the sample size was large enough to detect clinically significant differences.

The AUASI score decreased by a mean of 2.20 points (95% confidence interval [CI], -3.04 to -0.36) in the group that received saw palmetto and by 2.99 points (95% CI, -3.81 to -2.17) in the placebo group—a mean difference of 0.79 in favor of the placebo group (P=.91). The proportion of participants achieving a 3-point reduction in AUASI score was statistically similar between the 2 groups (P=0.66). There was no significant dose response difference between the 2 groups, and saw palmetto proved to be no better than placebo for any of the secondary outcomes.

 

 

Subgroup analysis did not reveal any results that differed from the main outcomes. The only adverse events that were significantly different between the 2 groups related to physical injury or trauma, which were unlikely to be due to the intervention.1

By using the same symptom scoring system (AUASI) as many studies in the previous Cochrane reviews, Barry et al were able to compare their findings with those of other high-quality studies with similar methodologies and outcome measures. Despite using an even higher dose of the extract, the results of this trial are remarkably consistent with previous conclusions: Saw palmetto is not an effective treatment for symptoms associated with BPH. Moreover, this trial had a broad base of participants similar to the population in a primary care practice, including patients who would typically choose a natural remedy for LUTS.1

WHAT’S NEW: We now have answers to queries about saw palmetto

This trial is the first to compare higher doses of saw palmetto with placebo to assess a dosage threshold for effectiveness. While the study found no evidence of saw palmetto toxicity even at these higher doses, the extract did not outperform placebo for any measured outcome.1

This high-quality study confirmed the recent series of rigorous studies with negative outcomes by showing that the use of a standard dosage was not a study limitation and that saw palmetto extract is not effective for treating LUTS at any dosage. This trial should substantially affect future guideline recommendations that were limited by methodological concerns in the past.9-11

CAVEATS: In theory, individual preparations could work differently

It is not possible to be absolutely certain that these findings apply to all saw palmetto extract preparations, given the unknown active ingredients and unknown mechanism of action. However, the researchers used a high-quality preparation (a proprietary lipidic ethanolic extract) of saw palmetto at higher doses than the STEP trial and came to a similar conclusion, making it highly unlikely that another preparation would perform differently.

CHALLENGES TO IMPLEMENTATION: There are none

We see no challenges to implementation of this recommendation.

Acknowledgement

The PURLs Surveillance System is supported in part by Grant Number UL1RR024999 from the National Center for Research Resources, a Clinical Translational Science Award to the University of Chicago. The content is solely the responsibility of the authors and does not necessarily represent the official views of the National Center For Research Resources or the National Institutes of Health.

References

1. Barry MJ, Meleth S, Lee JY, et al. Effect of increasing doses of saw palmetto extract on lower urinary tract symptoms: a randomized trial. JAMA. 2011;306:1344-1351.

2. Tacklind J, MacDonald R, Rutks I, et al. Serenoa repens for benign prostatic hyperplasia. Cochrane Database Syst Rev. 2009;(2):CD001423.-

3. Roehrborn CG, McConnell JD. Etiology, Pathophysiology, Epidemiology, and Natural History of Benign Prostatic Hyperplasia. 8th ed. Philadelphia, Pa: Campbell’s Urology; 2002.

4. Barnes PM, Bloom B, Nahin RL. Complementary and alternative medicine use among adults and children: United States, 2007. Natl Health Stat Report. 2008;(12):1-23.

5. Traish AM, Hassani J, Guay AT, et al. Adverse side effects of 5-alpha-reductase inhibitors therapy: persistent diminished libido and erectile dysfunction and depression in a subset of patients. J Sex Med. 2011;8:872-884.

6. Clifford GM, Farmer RD. Medical therapy for benign prostatic hyperplasia: a review of the literature. Eur Urol. 2000;38:2-19.

7. Wilt T, Ishani A, MacDonald R. Serenoa repens for benign prostatic hyperplasia. Cochrane Database Syst Rev. 2002;(3):CD001423.-

8. Bent S, Kane C, Shinohara K, et al. Saw palmetto for benign prostatic hyperplasia. N Engl J Med. 2006;354:557-566.

9. Benign prostatic hyperplasia: treatment {saw palmetto}. In: DynaMed. Available at: http://www.DynamicMedical.com. Accessed April 16, 2012.

10. Saper R. Clinical use of saw palmetto {saw palmetto}. In: Basow DS, ed. UpToDate [database online]. Waltham, Mass: UpToDate 2012. Available at: http://www.uptodate.com. Accessed April 16, 2012.

11. Agency for Healthcare Research and Quality. Guidelines on the treatment of non-neurogenic male LUTS. Available at: http://www.guideline.gov/content.aspx?id=34066. Accessed June 17, 2012.

References

1. Barry MJ, Meleth S, Lee JY, et al. Effect of increasing doses of saw palmetto extract on lower urinary tract symptoms: a randomized trial. JAMA. 2011;306:1344-1351.

2. Tacklind J, MacDonald R, Rutks I, et al. Serenoa repens for benign prostatic hyperplasia. Cochrane Database Syst Rev. 2009;(2):CD001423.-

3. Roehrborn CG, McConnell JD. Etiology, Pathophysiology, Epidemiology, and Natural History of Benign Prostatic Hyperplasia. 8th ed. Philadelphia, Pa: Campbell’s Urology; 2002.

4. Barnes PM, Bloom B, Nahin RL. Complementary and alternative medicine use among adults and children: United States, 2007. Natl Health Stat Report. 2008;(12):1-23.

5. Traish AM, Hassani J, Guay AT, et al. Adverse side effects of 5-alpha-reductase inhibitors therapy: persistent diminished libido and erectile dysfunction and depression in a subset of patients. J Sex Med. 2011;8:872-884.

6. Clifford GM, Farmer RD. Medical therapy for benign prostatic hyperplasia: a review of the literature. Eur Urol. 2000;38:2-19.

7. Wilt T, Ishani A, MacDonald R. Serenoa repens for benign prostatic hyperplasia. Cochrane Database Syst Rev. 2002;(3):CD001423.-

8. Bent S, Kane C, Shinohara K, et al. Saw palmetto for benign prostatic hyperplasia. N Engl J Med. 2006;354:557-566.

9. Benign prostatic hyperplasia: treatment {saw palmetto}. In: DynaMed. Available at: http://www.DynamicMedical.com. Accessed April 16, 2012.

10. Saper R. Clinical use of saw palmetto {saw palmetto}. In: Basow DS, ed. UpToDate [database online]. Waltham, Mass: UpToDate 2012. Available at: http://www.uptodate.com. Accessed April 16, 2012.

11. Agency for Healthcare Research and Quality. Guidelines on the treatment of non-neurogenic male LUTS. Available at: http://www.guideline.gov/content.aspx?id=34066. Accessed June 17, 2012.

Issue
The Journal of Family Practice - 61(7)
Issue
The Journal of Family Practice - 61(7)
Page Number
418-420
Page Number
418-420
Publications
Publications
Topics
Article Type
Display Headline
The shrinking case for saw palmetto
Display Headline
The shrinking case for saw palmetto
Sections
Article Source

PURLs Copyright

Copyright © 2012 The Family Physicians Inquiries Network. All rights reserved.

Inside the Article

Article PDF Media

Consider adding this drug to fight COPD that’s severe

Article Type
Changed
Mon, 01/14/2019 - 11:34
Display Headline
Consider adding this drug to fight COPD that’s severe

PRACTICE CHANGER

Consider prescribing daily azithromycin for patients with chronic obstructive pulmonary disease (COPD) and a history of exacerbations—but do a careful risk-benefit analysis first.1

STRENGTH OF RECOMMENDATION

B: Based on one well-designed double-blind, randomized controlled trial (RCT).

Albert RK, Connett J, Bailey WC, et al. Azithromycin for prevention of exacerbations of COPD. N Engl J Med. 2011;365:689-698.

ILLUSTRATIVE CASE

A 65-year-old man with a history of moderate to severe COPD schedules an appointment soon after discharge from the hospital—his second hospitalization for COPD exacerbations in 4 months. The patient uses inhaled glucocorticoids, a long-acting beta-agonist (LABA), and a long-acting anticholinergic. Should you add a macrolide to his medication regimen?

Acute exacerbations of COPD—the third highest cause of death in the United States2—have a major effect on quality of life, often resulting in repeat trips to the emergency department (ED) and numerous hospitalizations, office visits, and days lost from work. According to a new study that used 2006 data, there were 1.25 million hospitalizations for COPD exacerbations that year, with health care costs of $11.9 billion.3 Preventing exacerbations and the associated morbidity and mortality is a major challenge that primary care physicians face.

Can a macrolide help?
Corticosteroids, long-acting beta-agonists (LABAs), and the anticholinergic tiotropium are known to reduce COPD exacerbations,4,5 but what about antibiotics? A Cochrane meta-analysis of 9 RCTs that assessed antibiotic use for COPD found that it did not decrease the number of exacerbations. Notably, however, macrolides were not used in any of the studies.6

Macrolides are known to have anti-inflammatory, antibacterial, and immunomodulatory properties that reduce pulmonary exacerbations in other chronic lung diseases. A recent meta-analysis found that patients with cystic fibrosis have fewer pulmonary exacerbations when they take azithromycin 3 times a week.7

Small studies of the effect of macrolides on the frequency of COPD exacerbations have had conflicting results.8,9 The larger study detailed here evaluated the ability of daily azithromycin therapy to reduce COPD exacerbations.

STUDY SUMMARY: Daily dose led to fewer exacerbations

This double-blind RCT included close to 1150 participants from 12 US academic health centers, randomly assigned to receive azithromycin 250 mg daily or placebo, in addition to their usual care. (About 10% of patients in both groups died, withdrew, or were lost to follow-up.)

To be included, patients had to be ≥40 years old and have a clinical diagnosis of COPD, defined as a smoking history of 10 pack-years or more, a decreased forced expiratory volume in one second/forced vital capacity (FEV1/FVC) ratio, and a decreased FEV1 after bronchodilation. In addition, participants had to be on long-term oxygen or have used systemic steroids within the previous year or have had an ED visit or hospital admission for COPD during that time frame. Exclusion criteria included a history of asthma, a resting heart rate >100 beats per minute, a prolonged corrected QT interval (QTc) on electrocardiogram or the use of a medication that might prolong QTc, and a documented hearing impairment.

At baseline, participants were similar in basic demographics, COPD severity, smoking history, and medication use: 49% of those in the azithromycin group and 46% of the placebo group were taking a combination of inhaled corticosteroids, LABAs, and a long-acting anticholinergic medication.

The primary outcome was the time to the first COPD exacerbation. This was defined as ≥3 days with 2 or more COPD symptoms—new onset or worsening cough, dyspnea, sputum production, chest tightness, or wheezing—for which antibiotics or steroids were required. Secondary outcomes were quality-of-life measurements on the St. George’s Respiratory Questionnaire (SGRQ) and the Medical Outcomes 36-item Short Form Health Survey (SF-36). Nasopharyngeal swabs were done every 3 months to check for colonization and resistance. Hearing was assessed with audiometry at the time of enrollment, and again at 3 and 12 months. All patients were followed for a year, with monthly telephone calls or clinic visits, to determine if an exacerbation had occurred in the previous month.

The median time to the first exacerbation in the azithromycin group was 266 days (95% confidence interval [CI], 227-313) vs 174 days (95% CI, 143-215) in the placebo group; P<.001. Frequency of acute exacerbations was 1.48 per patient-year for the azithromycin group compared with 1.83 for the placebo group (relative risk=0.83; 95% CI, 0.72–0.95; P=.01). The number needed to treat to prevent one acute exacerbation in a one-year period was 2.86.

There was no significant difference in the SGRQ and SF-36 scores for the azithromycin vs the placebo group. There was a small reduction in unscheduled office visits (0.11 per patient-year; P=.048) in the azithromycin group, and a decrease in hospitalization that was not statistically significant.

 

 

Azithromycin group had higher rates of adverse effects
Nasopharyngeal cultures from participants who became colonized during the course of the study found macrolide resistance in 81% of those in the azithromycin group vs 41% of the placebo group (P<.001). Twenty-five percent of patients in the azithromycin group developed measurable hearing loss, compared with 20% of those on placebo (P=0.04; number needed to harm=20).

WHAT’S NEW?: A better understanding of benefits and risks

This study shows that the addition of azithromycin (250 mg/d) to standard COPD treatment decreases the number of exacerbations, but does little to reduce hospital admissions. It also highlights the adverse effect profile of azithromycin and the importance of using the antibiotic only for carefully selected patients.

CAVEATS: Macrolide resistance is a key concern

Twenty-five percent of the azithromycin group had documented hearing loss—an additional one in 20 compared with patients in the placebo group. More importantly, there was an increase in the prevalence of macrolide-resistant respiratory pathogens in patients on daily azithromycin. The long-term impact of daily azithromycin on antibiotic resistance is unknown, both for patients themselves and the community at large.

Physicians will have to assess the benefit of a decrease in COPD exacerbations (approximately one every 3 years) vs the risk of an increase in hearing problems and macrolide resistance. A sensible approach would be to reserve daily use of azithromycin for patients with a history of multiple exacerbations, who potentially have more to gain.

CHALLENGES TO IMPLEMENTATION: There are none

There are no major challenges to implementation aside from the cost, which would be approximately $1200 per year (azithromycin 250 mg [30 tablets] at $98.99 per month).10

Acknowledgement

The PURLs Surveillance System is supported in part by Grant Number UL1RR024999 from the National Center for Research Resources, a Clinical Translational Science Award to the University of Chicago. The content is solely the responsibility of the authors and does not necessarily represent the official views of the National Center For Research Resources or the National Institutes of Health.

References

1. Albert RK, Connett J, Bailey WC, et al. Azithromycin for prevention of exacerbations of COPD. N Engl J Med. 2011;365:689-698.

2. Centers for Disease Control and Prevention. Injury prevention & control: data & statistics. Available at: http://www.cdc.gov/injury/wisqars/LeadingCauses.html. Accessed April 16, 2012.

3. Perera PN, Armstrong EP, Sherrill DL, et al. Acute exacerbations of COPD in the United States: inpatient burden and predictors of cost and mortality. COPD. 2012;9:131-141.

4. Jenkins CR, Jones PW, Calverley PM, et al. Efficacy of salmeterol/fluticasone propionate by GOLD stage of chronic obstructive pulmonary disease: analysis from the randomised, placebo-controlled TORCH study. Respir Res. 2009;10:59.-

5. Decramer M, Celli B, Kesten S, et al. Effect of tiotropium on outcomes in patients with moderate chronic obstructive pulmonary disease (UPLIFT); a prespecified subgroup analysis of a randomized controlled trial. Lancet. 2009;374:1171-1178.

6. Black PN, Staykova T, Chacko EE, et al. Prophylactic antibiotic therapy for chronic bronchitis. Cochrane Database Syst Rev. 2003;(1):CD004105.-

7. Southern KW, Barker PM, Solis-Moya A, et al. Macrolide antibiotics for cystic fibrosis. Cochrane Database Syst Rev. 2011;(12):CD002203.-

8. Seemungal TA, Wilkinson TM, Hurst JR, et al. Long-term erythromycin therapy is associated with decreased chronic obstructive pulmonary exacerbations. Am J Respir Crit Care Med. 2008;178:1139-1147.

9. Yamaya A, Azuma A, Tanaka H, et al. Inhibitory effects of macrolides on exacerbations and hospitalization in chronic obstructive pulmonary disease in Japan: a retrospective multicenter analysis. J Am Geriatri Soc. 2008;56:1358-1360.

10. www.Drugstore.com. Accessed March 28, 2012.

Article PDF
Author and Disclosure Information

Keia Hobbs, MD
Department of Family, Medicine, University of North Carolina at Chapel, Hill

Dionna Brown, MD
The University of Chicago

PURLs EDITOR
Anne Mounsey, MD
Department of Family, Medicine, University of North Carolina, at Chapel Hill

Issue
The Journal of Family Practice - 61(7)
Publications
Topics
Page Number
414-416
Sections
Author and Disclosure Information

Keia Hobbs, MD
Department of Family, Medicine, University of North Carolina at Chapel, Hill

Dionna Brown, MD
The University of Chicago

PURLs EDITOR
Anne Mounsey, MD
Department of Family, Medicine, University of North Carolina, at Chapel Hill

Author and Disclosure Information

Keia Hobbs, MD
Department of Family, Medicine, University of North Carolina at Chapel, Hill

Dionna Brown, MD
The University of Chicago

PURLs EDITOR
Anne Mounsey, MD
Department of Family, Medicine, University of North Carolina, at Chapel Hill

Article PDF
Article PDF

PRACTICE CHANGER

Consider prescribing daily azithromycin for patients with chronic obstructive pulmonary disease (COPD) and a history of exacerbations—but do a careful risk-benefit analysis first.1

STRENGTH OF RECOMMENDATION

B: Based on one well-designed double-blind, randomized controlled trial (RCT).

Albert RK, Connett J, Bailey WC, et al. Azithromycin for prevention of exacerbations of COPD. N Engl J Med. 2011;365:689-698.

ILLUSTRATIVE CASE

A 65-year-old man with a history of moderate to severe COPD schedules an appointment soon after discharge from the hospital—his second hospitalization for COPD exacerbations in 4 months. The patient uses inhaled glucocorticoids, a long-acting beta-agonist (LABA), and a long-acting anticholinergic. Should you add a macrolide to his medication regimen?

Acute exacerbations of COPD—the third highest cause of death in the United States2—have a major effect on quality of life, often resulting in repeat trips to the emergency department (ED) and numerous hospitalizations, office visits, and days lost from work. According to a new study that used 2006 data, there were 1.25 million hospitalizations for COPD exacerbations that year, with health care costs of $11.9 billion.3 Preventing exacerbations and the associated morbidity and mortality is a major challenge that primary care physicians face.

Can a macrolide help?
Corticosteroids, long-acting beta-agonists (LABAs), and the anticholinergic tiotropium are known to reduce COPD exacerbations,4,5 but what about antibiotics? A Cochrane meta-analysis of 9 RCTs that assessed antibiotic use for COPD found that it did not decrease the number of exacerbations. Notably, however, macrolides were not used in any of the studies.6

Macrolides are known to have anti-inflammatory, antibacterial, and immunomodulatory properties that reduce pulmonary exacerbations in other chronic lung diseases. A recent meta-analysis found that patients with cystic fibrosis have fewer pulmonary exacerbations when they take azithromycin 3 times a week.7

Small studies of the effect of macrolides on the frequency of COPD exacerbations have had conflicting results.8,9 The larger study detailed here evaluated the ability of daily azithromycin therapy to reduce COPD exacerbations.

STUDY SUMMARY: Daily dose led to fewer exacerbations

This double-blind RCT included close to 1150 participants from 12 US academic health centers, randomly assigned to receive azithromycin 250 mg daily or placebo, in addition to their usual care. (About 10% of patients in both groups died, withdrew, or were lost to follow-up.)

To be included, patients had to be ≥40 years old and have a clinical diagnosis of COPD, defined as a smoking history of 10 pack-years or more, a decreased forced expiratory volume in one second/forced vital capacity (FEV1/FVC) ratio, and a decreased FEV1 after bronchodilation. In addition, participants had to be on long-term oxygen or have used systemic steroids within the previous year or have had an ED visit or hospital admission for COPD during that time frame. Exclusion criteria included a history of asthma, a resting heart rate >100 beats per minute, a prolonged corrected QT interval (QTc) on electrocardiogram or the use of a medication that might prolong QTc, and a documented hearing impairment.

At baseline, participants were similar in basic demographics, COPD severity, smoking history, and medication use: 49% of those in the azithromycin group and 46% of the placebo group were taking a combination of inhaled corticosteroids, LABAs, and a long-acting anticholinergic medication.

The primary outcome was the time to the first COPD exacerbation. This was defined as ≥3 days with 2 or more COPD symptoms—new onset or worsening cough, dyspnea, sputum production, chest tightness, or wheezing—for which antibiotics or steroids were required. Secondary outcomes were quality-of-life measurements on the St. George’s Respiratory Questionnaire (SGRQ) and the Medical Outcomes 36-item Short Form Health Survey (SF-36). Nasopharyngeal swabs were done every 3 months to check for colonization and resistance. Hearing was assessed with audiometry at the time of enrollment, and again at 3 and 12 months. All patients were followed for a year, with monthly telephone calls or clinic visits, to determine if an exacerbation had occurred in the previous month.

The median time to the first exacerbation in the azithromycin group was 266 days (95% confidence interval [CI], 227-313) vs 174 days (95% CI, 143-215) in the placebo group; P<.001. Frequency of acute exacerbations was 1.48 per patient-year for the azithromycin group compared with 1.83 for the placebo group (relative risk=0.83; 95% CI, 0.72–0.95; P=.01). The number needed to treat to prevent one acute exacerbation in a one-year period was 2.86.

There was no significant difference in the SGRQ and SF-36 scores for the azithromycin vs the placebo group. There was a small reduction in unscheduled office visits (0.11 per patient-year; P=.048) in the azithromycin group, and a decrease in hospitalization that was not statistically significant.

 

 

Azithromycin group had higher rates of adverse effects
Nasopharyngeal cultures from participants who became colonized during the course of the study found macrolide resistance in 81% of those in the azithromycin group vs 41% of the placebo group (P<.001). Twenty-five percent of patients in the azithromycin group developed measurable hearing loss, compared with 20% of those on placebo (P=0.04; number needed to harm=20).

WHAT’S NEW?: A better understanding of benefits and risks

This study shows that the addition of azithromycin (250 mg/d) to standard COPD treatment decreases the number of exacerbations, but does little to reduce hospital admissions. It also highlights the adverse effect profile of azithromycin and the importance of using the antibiotic only for carefully selected patients.

CAVEATS: Macrolide resistance is a key concern

Twenty-five percent of the azithromycin group had documented hearing loss—an additional one in 20 compared with patients in the placebo group. More importantly, there was an increase in the prevalence of macrolide-resistant respiratory pathogens in patients on daily azithromycin. The long-term impact of daily azithromycin on antibiotic resistance is unknown, both for patients themselves and the community at large.

Physicians will have to assess the benefit of a decrease in COPD exacerbations (approximately one every 3 years) vs the risk of an increase in hearing problems and macrolide resistance. A sensible approach would be to reserve daily use of azithromycin for patients with a history of multiple exacerbations, who potentially have more to gain.

CHALLENGES TO IMPLEMENTATION: There are none

There are no major challenges to implementation aside from the cost, which would be approximately $1200 per year (azithromycin 250 mg [30 tablets] at $98.99 per month).10

Acknowledgement

The PURLs Surveillance System is supported in part by Grant Number UL1RR024999 from the National Center for Research Resources, a Clinical Translational Science Award to the University of Chicago. The content is solely the responsibility of the authors and does not necessarily represent the official views of the National Center For Research Resources or the National Institutes of Health.

PRACTICE CHANGER

Consider prescribing daily azithromycin for patients with chronic obstructive pulmonary disease (COPD) and a history of exacerbations—but do a careful risk-benefit analysis first.1

STRENGTH OF RECOMMENDATION

B: Based on one well-designed double-blind, randomized controlled trial (RCT).

Albert RK, Connett J, Bailey WC, et al. Azithromycin for prevention of exacerbations of COPD. N Engl J Med. 2011;365:689-698.

ILLUSTRATIVE CASE

A 65-year-old man with a history of moderate to severe COPD schedules an appointment soon after discharge from the hospital—his second hospitalization for COPD exacerbations in 4 months. The patient uses inhaled glucocorticoids, a long-acting beta-agonist (LABA), and a long-acting anticholinergic. Should you add a macrolide to his medication regimen?

Acute exacerbations of COPD—the third highest cause of death in the United States2—have a major effect on quality of life, often resulting in repeat trips to the emergency department (ED) and numerous hospitalizations, office visits, and days lost from work. According to a new study that used 2006 data, there were 1.25 million hospitalizations for COPD exacerbations that year, with health care costs of $11.9 billion.3 Preventing exacerbations and the associated morbidity and mortality is a major challenge that primary care physicians face.

Can a macrolide help?
Corticosteroids, long-acting beta-agonists (LABAs), and the anticholinergic tiotropium are known to reduce COPD exacerbations,4,5 but what about antibiotics? A Cochrane meta-analysis of 9 RCTs that assessed antibiotic use for COPD found that it did not decrease the number of exacerbations. Notably, however, macrolides were not used in any of the studies.6

Macrolides are known to have anti-inflammatory, antibacterial, and immunomodulatory properties that reduce pulmonary exacerbations in other chronic lung diseases. A recent meta-analysis found that patients with cystic fibrosis have fewer pulmonary exacerbations when they take azithromycin 3 times a week.7

Small studies of the effect of macrolides on the frequency of COPD exacerbations have had conflicting results.8,9 The larger study detailed here evaluated the ability of daily azithromycin therapy to reduce COPD exacerbations.

STUDY SUMMARY: Daily dose led to fewer exacerbations

This double-blind RCT included close to 1150 participants from 12 US academic health centers, randomly assigned to receive azithromycin 250 mg daily or placebo, in addition to their usual care. (About 10% of patients in both groups died, withdrew, or were lost to follow-up.)

To be included, patients had to be ≥40 years old and have a clinical diagnosis of COPD, defined as a smoking history of 10 pack-years or more, a decreased forced expiratory volume in one second/forced vital capacity (FEV1/FVC) ratio, and a decreased FEV1 after bronchodilation. In addition, participants had to be on long-term oxygen or have used systemic steroids within the previous year or have had an ED visit or hospital admission for COPD during that time frame. Exclusion criteria included a history of asthma, a resting heart rate >100 beats per minute, a prolonged corrected QT interval (QTc) on electrocardiogram or the use of a medication that might prolong QTc, and a documented hearing impairment.

At baseline, participants were similar in basic demographics, COPD severity, smoking history, and medication use: 49% of those in the azithromycin group and 46% of the placebo group were taking a combination of inhaled corticosteroids, LABAs, and a long-acting anticholinergic medication.

The primary outcome was the time to the first COPD exacerbation. This was defined as ≥3 days with 2 or more COPD symptoms—new onset or worsening cough, dyspnea, sputum production, chest tightness, or wheezing—for which antibiotics or steroids were required. Secondary outcomes were quality-of-life measurements on the St. George’s Respiratory Questionnaire (SGRQ) and the Medical Outcomes 36-item Short Form Health Survey (SF-36). Nasopharyngeal swabs were done every 3 months to check for colonization and resistance. Hearing was assessed with audiometry at the time of enrollment, and again at 3 and 12 months. All patients were followed for a year, with monthly telephone calls or clinic visits, to determine if an exacerbation had occurred in the previous month.

The median time to the first exacerbation in the azithromycin group was 266 days (95% confidence interval [CI], 227-313) vs 174 days (95% CI, 143-215) in the placebo group; P<.001. Frequency of acute exacerbations was 1.48 per patient-year for the azithromycin group compared with 1.83 for the placebo group (relative risk=0.83; 95% CI, 0.72–0.95; P=.01). The number needed to treat to prevent one acute exacerbation in a one-year period was 2.86.

There was no significant difference in the SGRQ and SF-36 scores for the azithromycin vs the placebo group. There was a small reduction in unscheduled office visits (0.11 per patient-year; P=.048) in the azithromycin group, and a decrease in hospitalization that was not statistically significant.

 

 

Azithromycin group had higher rates of adverse effects
Nasopharyngeal cultures from participants who became colonized during the course of the study found macrolide resistance in 81% of those in the azithromycin group vs 41% of the placebo group (P<.001). Twenty-five percent of patients in the azithromycin group developed measurable hearing loss, compared with 20% of those on placebo (P=0.04; number needed to harm=20).

WHAT’S NEW?: A better understanding of benefits and risks

This study shows that the addition of azithromycin (250 mg/d) to standard COPD treatment decreases the number of exacerbations, but does little to reduce hospital admissions. It also highlights the adverse effect profile of azithromycin and the importance of using the antibiotic only for carefully selected patients.

CAVEATS: Macrolide resistance is a key concern

Twenty-five percent of the azithromycin group had documented hearing loss—an additional one in 20 compared with patients in the placebo group. More importantly, there was an increase in the prevalence of macrolide-resistant respiratory pathogens in patients on daily azithromycin. The long-term impact of daily azithromycin on antibiotic resistance is unknown, both for patients themselves and the community at large.

Physicians will have to assess the benefit of a decrease in COPD exacerbations (approximately one every 3 years) vs the risk of an increase in hearing problems and macrolide resistance. A sensible approach would be to reserve daily use of azithromycin for patients with a history of multiple exacerbations, who potentially have more to gain.

CHALLENGES TO IMPLEMENTATION: There are none

There are no major challenges to implementation aside from the cost, which would be approximately $1200 per year (azithromycin 250 mg [30 tablets] at $98.99 per month).10

Acknowledgement

The PURLs Surveillance System is supported in part by Grant Number UL1RR024999 from the National Center for Research Resources, a Clinical Translational Science Award to the University of Chicago. The content is solely the responsibility of the authors and does not necessarily represent the official views of the National Center For Research Resources or the National Institutes of Health.

References

1. Albert RK, Connett J, Bailey WC, et al. Azithromycin for prevention of exacerbations of COPD. N Engl J Med. 2011;365:689-698.

2. Centers for Disease Control and Prevention. Injury prevention & control: data & statistics. Available at: http://www.cdc.gov/injury/wisqars/LeadingCauses.html. Accessed April 16, 2012.

3. Perera PN, Armstrong EP, Sherrill DL, et al. Acute exacerbations of COPD in the United States: inpatient burden and predictors of cost and mortality. COPD. 2012;9:131-141.

4. Jenkins CR, Jones PW, Calverley PM, et al. Efficacy of salmeterol/fluticasone propionate by GOLD stage of chronic obstructive pulmonary disease: analysis from the randomised, placebo-controlled TORCH study. Respir Res. 2009;10:59.-

5. Decramer M, Celli B, Kesten S, et al. Effect of tiotropium on outcomes in patients with moderate chronic obstructive pulmonary disease (UPLIFT); a prespecified subgroup analysis of a randomized controlled trial. Lancet. 2009;374:1171-1178.

6. Black PN, Staykova T, Chacko EE, et al. Prophylactic antibiotic therapy for chronic bronchitis. Cochrane Database Syst Rev. 2003;(1):CD004105.-

7. Southern KW, Barker PM, Solis-Moya A, et al. Macrolide antibiotics for cystic fibrosis. Cochrane Database Syst Rev. 2011;(12):CD002203.-

8. Seemungal TA, Wilkinson TM, Hurst JR, et al. Long-term erythromycin therapy is associated with decreased chronic obstructive pulmonary exacerbations. Am J Respir Crit Care Med. 2008;178:1139-1147.

9. Yamaya A, Azuma A, Tanaka H, et al. Inhibitory effects of macrolides on exacerbations and hospitalization in chronic obstructive pulmonary disease in Japan: a retrospective multicenter analysis. J Am Geriatri Soc. 2008;56:1358-1360.

10. www.Drugstore.com. Accessed March 28, 2012.

References

1. Albert RK, Connett J, Bailey WC, et al. Azithromycin for prevention of exacerbations of COPD. N Engl J Med. 2011;365:689-698.

2. Centers for Disease Control and Prevention. Injury prevention & control: data & statistics. Available at: http://www.cdc.gov/injury/wisqars/LeadingCauses.html. Accessed April 16, 2012.

3. Perera PN, Armstrong EP, Sherrill DL, et al. Acute exacerbations of COPD in the United States: inpatient burden and predictors of cost and mortality. COPD. 2012;9:131-141.

4. Jenkins CR, Jones PW, Calverley PM, et al. Efficacy of salmeterol/fluticasone propionate by GOLD stage of chronic obstructive pulmonary disease: analysis from the randomised, placebo-controlled TORCH study. Respir Res. 2009;10:59.-

5. Decramer M, Celli B, Kesten S, et al. Effect of tiotropium on outcomes in patients with moderate chronic obstructive pulmonary disease (UPLIFT); a prespecified subgroup analysis of a randomized controlled trial. Lancet. 2009;374:1171-1178.

6. Black PN, Staykova T, Chacko EE, et al. Prophylactic antibiotic therapy for chronic bronchitis. Cochrane Database Syst Rev. 2003;(1):CD004105.-

7. Southern KW, Barker PM, Solis-Moya A, et al. Macrolide antibiotics for cystic fibrosis. Cochrane Database Syst Rev. 2011;(12):CD002203.-

8. Seemungal TA, Wilkinson TM, Hurst JR, et al. Long-term erythromycin therapy is associated with decreased chronic obstructive pulmonary exacerbations. Am J Respir Crit Care Med. 2008;178:1139-1147.

9. Yamaya A, Azuma A, Tanaka H, et al. Inhibitory effects of macrolides on exacerbations and hospitalization in chronic obstructive pulmonary disease in Japan: a retrospective multicenter analysis. J Am Geriatri Soc. 2008;56:1358-1360.

10. www.Drugstore.com. Accessed March 28, 2012.

Issue
The Journal of Family Practice - 61(7)
Issue
The Journal of Family Practice - 61(7)
Page Number
414-416
Page Number
414-416
Publications
Publications
Topics
Article Type
Display Headline
Consider adding this drug to fight COPD that’s severe
Display Headline
Consider adding this drug to fight COPD that’s severe
Sections
Article Source

PURLs Copyright

Copyright © 2012 The Family Physicians Inquiries Network. All rights reserved.

Inside the Article

Article PDF Media

Treating pulmonary embolism at home?

Article Type
Changed
Mon, 01/14/2019 - 11:34
Display Headline
Treating pulmonary embolism at home?
PRACTICE CHANGER

Treat low-risk patients with pulmonary embolism (PE) with low-molecular-weight heparin (LMWH) in an outpatient setting.1

Aujesky D, Roy PM, Verschuren F, et al. Outpatient versus inpatient treatment for patients with acute pulmonary embolism: an international, open-label, randomised, non-inferiority trial. Lancet. 2011; 378:41-48.

STRENGTH OF RECOMMENDATION

B: Based on one good quality randomized controlled trial (RCT).

 

ILLUSTRATIVE CASE

Three months after undergoing surgical repair of an ankle fracture, a 50-year-old woman presents with acute onset dyspnea at rest and pleuritic chest pain. Her left calf is tender and swollen. The patient has a history of hypertension and smokes about 10 cigarettes per day. Her temperature is 37°C (99°F); pulse rate, 98; blood pressure, 135/85 mm Hg; respiratory rate, 25; and pulse oximetry, 92%.

You order a spiral CT, which reveals a contrast filling defect indicative of a PE. Her score on the Pulmonary Embolism Severity Index (PESI) is 50, an indication of low risk. She wants to know if she can be treated at home. What should you tell her?

In the past, intravenous unfractionated heparin, administered in an inpatient setting, was the recommended initial anticoagulation therapy for patients with venous thromboembolism (VTE). LMWH, which can be administered subcutaneously and does not require laboratory monitoring, has made it possible to treat VTE without hospitalization.

Outpatient PE care hindered by lack of evidence
Guidelines from the American College of Physicians, American Academy of Family Physicians, and British Thoracic Society recommend outpatient treatment of deep vein thrombosis with LMWH, which they find to be safe and cost effective for select patients.2,3 Until recently, the safety and efficacy of out-patient management of PE has been less clear.

The lack of an accurate prediction tool to identify patients who could be treated safely outside of the hospital was one barrier to the development of evidence-based recommendations for outpatient PE treatment. In 2005, the PESI,4 a validated tool that identifies patients with low risk of death from PE, was developed. Until recently, the absence of an RCT comparing inpatient and outpatient treatment for acute PE was another barrier.

STUDY SUMMARY: Outpatient treatment measures up

The Outpatient Treatment of Pulmonary Embolism (OTPE) study was a multinational, randomized, noninferiority trial comparing outpatient vs inpatient treatment of low-risk patients with acute PE. Participants had to be ≥18 years old, have acute symptomatic and objectively verified PE, and be at low risk of death based on the PESI score.4 In addition to excluding patients at moderate or high risk, the researchers identified 14 other exclusion criteria, including hypoxia, chest pain requiring opiates, and high risk for bleeding.

Patients were randomly assigned to the outpatient (n=171) or inpatient (n=168) group. Both groups received subcutaneous LMWH (enoxaparin, 1 mg/kg twice a day) for ≥5 days, followed by oral anticoagulation with a vitamin K antagonist for ≥90 days. Patients in the outpatient group were discharged from the emergency department (ED) within 24 hours of randomization, after being trained by a nurse to self-inject. Therapy after discharge was managed either by the patient’s primary care physician or the hospital’s anticoagulation staff.

The LMWH was discontinued in patients with an INR ≥2.0 for 2 consecutive days. All patients were followed for 90 days, and contacted by the study team daily for the first week and then at 14, 30, 60, and 90 days. On each occasion, participants were asked about symptoms of recurrent VTE, bleeding, and the use of health care resources.

The primary outcome was the recurrence of symptomatic, objectively confirmed VTE within the study period. Secondary outcomes were major bleeding and all-cause mortality. Outcomes were confirmed by clinicians who were unaware of treatment assignments.

Patients were also asked to rate both their overall satisfaction with their care and their treatment preference 14 days after randomization, using a 5-point Likert questionnaire. Prior to the trial, the investigators decided that outpatient treatment would be considered noninferior to inpatient care if the difference between rates of recurrent VTE did not exceed 4%, a measure used in previous studies comparing treatment regimens for VTE and outpatient vs inpatient treatment of DVT.5,6

Little difference in readmission rates, ED or office visits
One in 171 outpatients (0.6%) and none of the inpatients had recurrent VTE. Two out-patients (1.2%)—and no inpatients—developed major bleeding within 14 days, the result of intramuscular hematomas that occurred on Days 3 and 13. There was one additional bleeding event (menometrorrhagia) in the outpatient group on Day 50, but it was believed to be unrelated to the PE treatment. Per-protocol analysis, a more conservative measure used in noninferiority studies, found a difference in major bleeding rates of 3.8%. One person in each group died from non-VTE and nontreatment-related causes.

 

 

 

Almost all participants (99%) completed the satisfaction survey, which indicated that 92% of outpatients and 95% of inpatients were satisfied or very satisfied with their care. Hospital readmission rates, ED visits, and visits to primary care physicians were similar, with no significant differences between the 2 groups. The mean time spent in the hospital was 0.5 days (standard deviation [SD], 1.0) for outpatients and 3.9 days (SD, 3.1) for in-patients. Fourteen percent of outpatients and 6% of inpatients received home nursing visits for enoxaparin injection. The total number of home visits was higher among outpatients (348 vs 105). Because both groups had extreme outliers, however, this difference was not statistically significant.

WHAT’S NEW: It’s safe to keep low-risk patients at home

This is the first RCT comparing the safety and effectiveness of outpatient and inpatient treatment of acute, symptomatic PE. Results were statistically comparable, and patients were satisfied being treated at home. Outpatient treatment was less expensive because of the shorter length of stay (0.5 vs 3.9 days) and was associated with the same rates of hospital readmission, ED visits, and visits to primary care physicians. There were more home nursing visits in the outpatient treatment group. But even if you assume a cost of $200 per home visit, the additional cost would be about $282 per individual in the outpatient group—significantly less than the cost of the additional 3.4 days in the hospital for each individual in the inpatient group.

The study also confirmed that the PESI accurately identifies low-risk patients with PE who can be treated in an outpatient setting. Thirty percent of patients who were screened for the OTPE trial met the low-risk eligibility requirement.

CAVEATS: Use of risk assessment tool is essential

The average age of patients in this study was 47 years in the outpatient group and 49 years in the inpatient group. In addition, only 1% to 3% of the patients were diagnosed with cancer. Older patients who have both cancer and PE would be unlikely to qualify for outpatient care.

Physicians applying this practice changer should use the PESI to ensure that outpatient treatment for PE is used only for individuals at low risk.

CHALLENGES TO IMPLEMENTATION: ED coordination, training, and home care won’t be easy

This practice changer may be difficult for family physicians, who might not be included in emergency physicians’ decisions regarding the appropriate treatment for acute PE. In this study, primary care physicians were notified of the randomized treatment plan for their patients, and 17 potential participants were excluded from the trial because of their doctors’ opposition.

Outpatient management should be considered only if arrangements for adequate home nursing care can be made, if needed— and only for patients who are able to follow instructions and self-inject LMWH. Newer anticoagulation medications that are either injected once a day or taken orally might decrease the need for home nursing visits.

Acknowledgement

The PURLs Surveillance System is supported in part by Grant Number UL1RR024999 from the National Center for Research Resources, a Clinical Translational Science award to the University of Chicago. The content is solely the responsibility of the authors and does not necessarily represent the official views of the National Center for Research Resources or the National Institutes of Health.

Click here to view PURL METHODOLOGY

References

1. Aujesky D, Roy PM, Verschuren F, et al. Outpatient versus inpatient treatment for patients with acute pulmonary embolism: an international, open-label, randomised, non-inferiority trial. Lancet. 2011;378:41-48.

2. Snow V, Qaseem A, Barry P, et al. Management of venous thromboembolism: a clinical practice guideline from the American College of Physicians and the American Academy of Family Physicians. Ann Intern Med. 2007;146:204-210.

3. British Thoracic Society Standards of Care Committee Pulmonary Embolism Guideline Development Group. British Thoracic Society guidelines for the management of suspected acute pulmonary embolism. Thorax. 2003;58:470-483.

4. Aujesky D, Obrosky DS, Stone RA, et al. Derivation and validation of a prognostic model for pulmonary embolism. Am J Respir Crit Care Med. 2005;172:1041-1046.

5. Koopman MM, Prandoni P, Piovella F, et al. Treatment of venous thrombosis with intravenous unfractionated heparin administered in the hospital as compared with subcutaneous low-molecular-weight heparin administered at home. The Tasman Study Group. N Engl J Med. 1996;334:682-687.

6. Schulman S, Kearon C, Kakkar AK, et al. Dabigatran versus warfarin in the treatment of acute venous thromboembolism. N Engl J Med. 2009;361:2342-2352.

Article PDF
Author and Disclosure Information
Outpatient care of PE: tips from an emergency physician
Donald M. Yealy, MD

Mari Egan, MD
University of Chicago

Kate Rowland, MD
University of Chicago

PURLs EDITOR
John Hickner, MD, MSc
Cleveland Clinic

Issue
The Journal of Family Practice - 61(6)
Publications
Topics
Page Number
349-352
Legacy Keywords
Mari Egan ;MD; Kate Rowland ;MD; acute PE; low-molecular-weight heparin; LMWH; outpatient PE care; venous thromboembolism; OTPE study; treatment preference
Sections
Author and Disclosure Information
Outpatient care of PE: tips from an emergency physician
Donald M. Yealy, MD

Mari Egan, MD
University of Chicago

Kate Rowland, MD
University of Chicago

PURLs EDITOR
John Hickner, MD, MSc
Cleveland Clinic

Author and Disclosure Information
Outpatient care of PE: tips from an emergency physician
Donald M. Yealy, MD

Mari Egan, MD
University of Chicago

Kate Rowland, MD
University of Chicago

PURLs EDITOR
John Hickner, MD, MSc
Cleveland Clinic

Article PDF
Article PDF
PRACTICE CHANGER

Treat low-risk patients with pulmonary embolism (PE) with low-molecular-weight heparin (LMWH) in an outpatient setting.1

Aujesky D, Roy PM, Verschuren F, et al. Outpatient versus inpatient treatment for patients with acute pulmonary embolism: an international, open-label, randomised, non-inferiority trial. Lancet. 2011; 378:41-48.

STRENGTH OF RECOMMENDATION

B: Based on one good quality randomized controlled trial (RCT).

 

ILLUSTRATIVE CASE

Three months after undergoing surgical repair of an ankle fracture, a 50-year-old woman presents with acute onset dyspnea at rest and pleuritic chest pain. Her left calf is tender and swollen. The patient has a history of hypertension and smokes about 10 cigarettes per day. Her temperature is 37°C (99°F); pulse rate, 98; blood pressure, 135/85 mm Hg; respiratory rate, 25; and pulse oximetry, 92%.

You order a spiral CT, which reveals a contrast filling defect indicative of a PE. Her score on the Pulmonary Embolism Severity Index (PESI) is 50, an indication of low risk. She wants to know if she can be treated at home. What should you tell her?

In the past, intravenous unfractionated heparin, administered in an inpatient setting, was the recommended initial anticoagulation therapy for patients with venous thromboembolism (VTE). LMWH, which can be administered subcutaneously and does not require laboratory monitoring, has made it possible to treat VTE without hospitalization.

Outpatient PE care hindered by lack of evidence
Guidelines from the American College of Physicians, American Academy of Family Physicians, and British Thoracic Society recommend outpatient treatment of deep vein thrombosis with LMWH, which they find to be safe and cost effective for select patients.2,3 Until recently, the safety and efficacy of out-patient management of PE has been less clear.

The lack of an accurate prediction tool to identify patients who could be treated safely outside of the hospital was one barrier to the development of evidence-based recommendations for outpatient PE treatment. In 2005, the PESI,4 a validated tool that identifies patients with low risk of death from PE, was developed. Until recently, the absence of an RCT comparing inpatient and outpatient treatment for acute PE was another barrier.

STUDY SUMMARY: Outpatient treatment measures up

The Outpatient Treatment of Pulmonary Embolism (OTPE) study was a multinational, randomized, noninferiority trial comparing outpatient vs inpatient treatment of low-risk patients with acute PE. Participants had to be ≥18 years old, have acute symptomatic and objectively verified PE, and be at low risk of death based on the PESI score.4 In addition to excluding patients at moderate or high risk, the researchers identified 14 other exclusion criteria, including hypoxia, chest pain requiring opiates, and high risk for bleeding.

Patients were randomly assigned to the outpatient (n=171) or inpatient (n=168) group. Both groups received subcutaneous LMWH (enoxaparin, 1 mg/kg twice a day) for ≥5 days, followed by oral anticoagulation with a vitamin K antagonist for ≥90 days. Patients in the outpatient group were discharged from the emergency department (ED) within 24 hours of randomization, after being trained by a nurse to self-inject. Therapy after discharge was managed either by the patient’s primary care physician or the hospital’s anticoagulation staff.

The LMWH was discontinued in patients with an INR ≥2.0 for 2 consecutive days. All patients were followed for 90 days, and contacted by the study team daily for the first week and then at 14, 30, 60, and 90 days. On each occasion, participants were asked about symptoms of recurrent VTE, bleeding, and the use of health care resources.

The primary outcome was the recurrence of symptomatic, objectively confirmed VTE within the study period. Secondary outcomes were major bleeding and all-cause mortality. Outcomes were confirmed by clinicians who were unaware of treatment assignments.

Patients were also asked to rate both their overall satisfaction with their care and their treatment preference 14 days after randomization, using a 5-point Likert questionnaire. Prior to the trial, the investigators decided that outpatient treatment would be considered noninferior to inpatient care if the difference between rates of recurrent VTE did not exceed 4%, a measure used in previous studies comparing treatment regimens for VTE and outpatient vs inpatient treatment of DVT.5,6

Little difference in readmission rates, ED or office visits
One in 171 outpatients (0.6%) and none of the inpatients had recurrent VTE. Two out-patients (1.2%)—and no inpatients—developed major bleeding within 14 days, the result of intramuscular hematomas that occurred on Days 3 and 13. There was one additional bleeding event (menometrorrhagia) in the outpatient group on Day 50, but it was believed to be unrelated to the PE treatment. Per-protocol analysis, a more conservative measure used in noninferiority studies, found a difference in major bleeding rates of 3.8%. One person in each group died from non-VTE and nontreatment-related causes.

 

 

 

Almost all participants (99%) completed the satisfaction survey, which indicated that 92% of outpatients and 95% of inpatients were satisfied or very satisfied with their care. Hospital readmission rates, ED visits, and visits to primary care physicians were similar, with no significant differences between the 2 groups. The mean time spent in the hospital was 0.5 days (standard deviation [SD], 1.0) for outpatients and 3.9 days (SD, 3.1) for in-patients. Fourteen percent of outpatients and 6% of inpatients received home nursing visits for enoxaparin injection. The total number of home visits was higher among outpatients (348 vs 105). Because both groups had extreme outliers, however, this difference was not statistically significant.

WHAT’S NEW: It’s safe to keep low-risk patients at home

This is the first RCT comparing the safety and effectiveness of outpatient and inpatient treatment of acute, symptomatic PE. Results were statistically comparable, and patients were satisfied being treated at home. Outpatient treatment was less expensive because of the shorter length of stay (0.5 vs 3.9 days) and was associated with the same rates of hospital readmission, ED visits, and visits to primary care physicians. There were more home nursing visits in the outpatient treatment group. But even if you assume a cost of $200 per home visit, the additional cost would be about $282 per individual in the outpatient group—significantly less than the cost of the additional 3.4 days in the hospital for each individual in the inpatient group.

The study also confirmed that the PESI accurately identifies low-risk patients with PE who can be treated in an outpatient setting. Thirty percent of patients who were screened for the OTPE trial met the low-risk eligibility requirement.

CAVEATS: Use of risk assessment tool is essential

The average age of patients in this study was 47 years in the outpatient group and 49 years in the inpatient group. In addition, only 1% to 3% of the patients were diagnosed with cancer. Older patients who have both cancer and PE would be unlikely to qualify for outpatient care.

Physicians applying this practice changer should use the PESI to ensure that outpatient treatment for PE is used only for individuals at low risk.

CHALLENGES TO IMPLEMENTATION: ED coordination, training, and home care won’t be easy

This practice changer may be difficult for family physicians, who might not be included in emergency physicians’ decisions regarding the appropriate treatment for acute PE. In this study, primary care physicians were notified of the randomized treatment plan for their patients, and 17 potential participants were excluded from the trial because of their doctors’ opposition.

Outpatient management should be considered only if arrangements for adequate home nursing care can be made, if needed— and only for patients who are able to follow instructions and self-inject LMWH. Newer anticoagulation medications that are either injected once a day or taken orally might decrease the need for home nursing visits.

Acknowledgement

The PURLs Surveillance System is supported in part by Grant Number UL1RR024999 from the National Center for Research Resources, a Clinical Translational Science award to the University of Chicago. The content is solely the responsibility of the authors and does not necessarily represent the official views of the National Center for Research Resources or the National Institutes of Health.

Click here to view PURL METHODOLOGY

PRACTICE CHANGER

Treat low-risk patients with pulmonary embolism (PE) with low-molecular-weight heparin (LMWH) in an outpatient setting.1

Aujesky D, Roy PM, Verschuren F, et al. Outpatient versus inpatient treatment for patients with acute pulmonary embolism: an international, open-label, randomised, non-inferiority trial. Lancet. 2011; 378:41-48.

STRENGTH OF RECOMMENDATION

B: Based on one good quality randomized controlled trial (RCT).

 

ILLUSTRATIVE CASE

Three months after undergoing surgical repair of an ankle fracture, a 50-year-old woman presents with acute onset dyspnea at rest and pleuritic chest pain. Her left calf is tender and swollen. The patient has a history of hypertension and smokes about 10 cigarettes per day. Her temperature is 37°C (99°F); pulse rate, 98; blood pressure, 135/85 mm Hg; respiratory rate, 25; and pulse oximetry, 92%.

You order a spiral CT, which reveals a contrast filling defect indicative of a PE. Her score on the Pulmonary Embolism Severity Index (PESI) is 50, an indication of low risk. She wants to know if she can be treated at home. What should you tell her?

In the past, intravenous unfractionated heparin, administered in an inpatient setting, was the recommended initial anticoagulation therapy for patients with venous thromboembolism (VTE). LMWH, which can be administered subcutaneously and does not require laboratory monitoring, has made it possible to treat VTE without hospitalization.

Outpatient PE care hindered by lack of evidence
Guidelines from the American College of Physicians, American Academy of Family Physicians, and British Thoracic Society recommend outpatient treatment of deep vein thrombosis with LMWH, which they find to be safe and cost effective for select patients.2,3 Until recently, the safety and efficacy of out-patient management of PE has been less clear.

The lack of an accurate prediction tool to identify patients who could be treated safely outside of the hospital was one barrier to the development of evidence-based recommendations for outpatient PE treatment. In 2005, the PESI,4 a validated tool that identifies patients with low risk of death from PE, was developed. Until recently, the absence of an RCT comparing inpatient and outpatient treatment for acute PE was another barrier.

STUDY SUMMARY: Outpatient treatment measures up

The Outpatient Treatment of Pulmonary Embolism (OTPE) study was a multinational, randomized, noninferiority trial comparing outpatient vs inpatient treatment of low-risk patients with acute PE. Participants had to be ≥18 years old, have acute symptomatic and objectively verified PE, and be at low risk of death based on the PESI score.4 In addition to excluding patients at moderate or high risk, the researchers identified 14 other exclusion criteria, including hypoxia, chest pain requiring opiates, and high risk for bleeding.

Patients were randomly assigned to the outpatient (n=171) or inpatient (n=168) group. Both groups received subcutaneous LMWH (enoxaparin, 1 mg/kg twice a day) for ≥5 days, followed by oral anticoagulation with a vitamin K antagonist for ≥90 days. Patients in the outpatient group were discharged from the emergency department (ED) within 24 hours of randomization, after being trained by a nurse to self-inject. Therapy after discharge was managed either by the patient’s primary care physician or the hospital’s anticoagulation staff.

The LMWH was discontinued in patients with an INR ≥2.0 for 2 consecutive days. All patients were followed for 90 days, and contacted by the study team daily for the first week and then at 14, 30, 60, and 90 days. On each occasion, participants were asked about symptoms of recurrent VTE, bleeding, and the use of health care resources.

The primary outcome was the recurrence of symptomatic, objectively confirmed VTE within the study period. Secondary outcomes were major bleeding and all-cause mortality. Outcomes were confirmed by clinicians who were unaware of treatment assignments.

Patients were also asked to rate both their overall satisfaction with their care and their treatment preference 14 days after randomization, using a 5-point Likert questionnaire. Prior to the trial, the investigators decided that outpatient treatment would be considered noninferior to inpatient care if the difference between rates of recurrent VTE did not exceed 4%, a measure used in previous studies comparing treatment regimens for VTE and outpatient vs inpatient treatment of DVT.5,6

Little difference in readmission rates, ED or office visits
One in 171 outpatients (0.6%) and none of the inpatients had recurrent VTE. Two out-patients (1.2%)—and no inpatients—developed major bleeding within 14 days, the result of intramuscular hematomas that occurred on Days 3 and 13. There was one additional bleeding event (menometrorrhagia) in the outpatient group on Day 50, but it was believed to be unrelated to the PE treatment. Per-protocol analysis, a more conservative measure used in noninferiority studies, found a difference in major bleeding rates of 3.8%. One person in each group died from non-VTE and nontreatment-related causes.

 

 

 

Almost all participants (99%) completed the satisfaction survey, which indicated that 92% of outpatients and 95% of inpatients were satisfied or very satisfied with their care. Hospital readmission rates, ED visits, and visits to primary care physicians were similar, with no significant differences between the 2 groups. The mean time spent in the hospital was 0.5 days (standard deviation [SD], 1.0) for outpatients and 3.9 days (SD, 3.1) for in-patients. Fourteen percent of outpatients and 6% of inpatients received home nursing visits for enoxaparin injection. The total number of home visits was higher among outpatients (348 vs 105). Because both groups had extreme outliers, however, this difference was not statistically significant.

WHAT’S NEW: It’s safe to keep low-risk patients at home

This is the first RCT comparing the safety and effectiveness of outpatient and inpatient treatment of acute, symptomatic PE. Results were statistically comparable, and patients were satisfied being treated at home. Outpatient treatment was less expensive because of the shorter length of stay (0.5 vs 3.9 days) and was associated with the same rates of hospital readmission, ED visits, and visits to primary care physicians. There were more home nursing visits in the outpatient treatment group. But even if you assume a cost of $200 per home visit, the additional cost would be about $282 per individual in the outpatient group—significantly less than the cost of the additional 3.4 days in the hospital for each individual in the inpatient group.

The study also confirmed that the PESI accurately identifies low-risk patients with PE who can be treated in an outpatient setting. Thirty percent of patients who were screened for the OTPE trial met the low-risk eligibility requirement.

CAVEATS: Use of risk assessment tool is essential

The average age of patients in this study was 47 years in the outpatient group and 49 years in the inpatient group. In addition, only 1% to 3% of the patients were diagnosed with cancer. Older patients who have both cancer and PE would be unlikely to qualify for outpatient care.

Physicians applying this practice changer should use the PESI to ensure that outpatient treatment for PE is used only for individuals at low risk.

CHALLENGES TO IMPLEMENTATION: ED coordination, training, and home care won’t be easy

This practice changer may be difficult for family physicians, who might not be included in emergency physicians’ decisions regarding the appropriate treatment for acute PE. In this study, primary care physicians were notified of the randomized treatment plan for their patients, and 17 potential participants were excluded from the trial because of their doctors’ opposition.

Outpatient management should be considered only if arrangements for adequate home nursing care can be made, if needed— and only for patients who are able to follow instructions and self-inject LMWH. Newer anticoagulation medications that are either injected once a day or taken orally might decrease the need for home nursing visits.

Acknowledgement

The PURLs Surveillance System is supported in part by Grant Number UL1RR024999 from the National Center for Research Resources, a Clinical Translational Science award to the University of Chicago. The content is solely the responsibility of the authors and does not necessarily represent the official views of the National Center for Research Resources or the National Institutes of Health.

Click here to view PURL METHODOLOGY

References

1. Aujesky D, Roy PM, Verschuren F, et al. Outpatient versus inpatient treatment for patients with acute pulmonary embolism: an international, open-label, randomised, non-inferiority trial. Lancet. 2011;378:41-48.

2. Snow V, Qaseem A, Barry P, et al. Management of venous thromboembolism: a clinical practice guideline from the American College of Physicians and the American Academy of Family Physicians. Ann Intern Med. 2007;146:204-210.

3. British Thoracic Society Standards of Care Committee Pulmonary Embolism Guideline Development Group. British Thoracic Society guidelines for the management of suspected acute pulmonary embolism. Thorax. 2003;58:470-483.

4. Aujesky D, Obrosky DS, Stone RA, et al. Derivation and validation of a prognostic model for pulmonary embolism. Am J Respir Crit Care Med. 2005;172:1041-1046.

5. Koopman MM, Prandoni P, Piovella F, et al. Treatment of venous thrombosis with intravenous unfractionated heparin administered in the hospital as compared with subcutaneous low-molecular-weight heparin administered at home. The Tasman Study Group. N Engl J Med. 1996;334:682-687.

6. Schulman S, Kearon C, Kakkar AK, et al. Dabigatran versus warfarin in the treatment of acute venous thromboembolism. N Engl J Med. 2009;361:2342-2352.

References

1. Aujesky D, Roy PM, Verschuren F, et al. Outpatient versus inpatient treatment for patients with acute pulmonary embolism: an international, open-label, randomised, non-inferiority trial. Lancet. 2011;378:41-48.

2. Snow V, Qaseem A, Barry P, et al. Management of venous thromboembolism: a clinical practice guideline from the American College of Physicians and the American Academy of Family Physicians. Ann Intern Med. 2007;146:204-210.

3. British Thoracic Society Standards of Care Committee Pulmonary Embolism Guideline Development Group. British Thoracic Society guidelines for the management of suspected acute pulmonary embolism. Thorax. 2003;58:470-483.

4. Aujesky D, Obrosky DS, Stone RA, et al. Derivation and validation of a prognostic model for pulmonary embolism. Am J Respir Crit Care Med. 2005;172:1041-1046.

5. Koopman MM, Prandoni P, Piovella F, et al. Treatment of venous thrombosis with intravenous unfractionated heparin administered in the hospital as compared with subcutaneous low-molecular-weight heparin administered at home. The Tasman Study Group. N Engl J Med. 1996;334:682-687.

6. Schulman S, Kearon C, Kakkar AK, et al. Dabigatran versus warfarin in the treatment of acute venous thromboembolism. N Engl J Med. 2009;361:2342-2352.

Issue
The Journal of Family Practice - 61(6)
Issue
The Journal of Family Practice - 61(6)
Page Number
349-352
Page Number
349-352
Publications
Publications
Topics
Article Type
Display Headline
Treating pulmonary embolism at home?
Display Headline
Treating pulmonary embolism at home?
Legacy Keywords
Mari Egan ;MD; Kate Rowland ;MD; acute PE; low-molecular-weight heparin; LMWH; outpatient PE care; venous thromboembolism; OTPE study; treatment preference
Legacy Keywords
Mari Egan ;MD; Kate Rowland ;MD; acute PE; low-molecular-weight heparin; LMWH; outpatient PE care; venous thromboembolism; OTPE study; treatment preference
Sections
PURLs Copyright

Copyright © 2012 The Family Physicians Inquiries Network. All rights reserved.

Disallow All Ads
Alternative CME
Article PDF Media